Family Practice PA easy

Pataasin ang iyong marka sa homework at exams ngayon gamit ang Quizwiz!

A 22-year-old female presents to the primary care office with a complaint of heartburn. She has symptoms once every few weeks, and the heartburn occurs about 30 minutes after eating. The symptoms are mostly present when eating foods that contain tomato sauce. She has not tried any medication. She denies dysphagia, odynophagia, or weight loss. You are going to recommend a medication that works by inhibiting the proton pump of the gastric parietal cells. Which of the following medications has the mechanism of action listed above? A. Omeprazole B. Cimetidine C. Calcium carbonate D. Sucralfate E. Magnesium hydroxide

A The correct answer is omeprazole, which is a proton pump inhibitor. Cimetidine is a H2 receptor antagonis, which works by blocking the action of histamine on parietal cells in the stomach. Calcium carbonate and magnesium hydroxide are both antacids, which work by neutralizing the acid in the stomach. Sucralfate works by creating a paste-like material when it comes in contact with stomach acid, and serves as a buffer from the acid.

42-year-old female with history of severe rheumatoid arthritis requiring multiple medications follows up with her primary care physician assistant. She has been doing well on her current medication regimen but has been having some mild headaches. Her blood pressure today is 170/92 P = 82, T = 96.2˚F. She denies any previous history of hypertension. Which of the following medications that she is taking is most likely to be A. methotrexate B. cyclosporine C. acetaminophen D. hydrocodone E. fexofenadine

B The correct answer is (B). Cyclosporine, an immunosuppressant, is known to cause hypertension as a potential side effect and should be considered as a potential secondary cause of secondary hypertension. Methotrexate, acetaminophen, and fexofenadine are not known to cause secondary hypertension. Hydrocodone may actually cause hypotension as a potential adverse reaction.

A 60-year-old female presents for a routine physical exam. She has not seen a health care provider in over 30 years. She has no significant past medical history and is taking no medications. Her surgical history includes only carpal tunnel repair five years ago. Upon review of symptoms, she notes feeling somewhat tired. She attributes that to old age. Vital signs include BP 110/55, pulse 55 bpm, and respirations 16 per minute. Upon exam you note an enlarged, firm thyroid, thickening of her skin, and puffy facial features and pallor. A portion of blood tests that you ordered is shown below: Serum TSH = 4.4 uIU/mL Hemoglobin = 10.0 g/dL Hematocrit = 30% MCV = 101 Fasting plasma glucose = 105 mg/dL BUN = 10 mg/dL Creatinie = 0.6 ng/mL Which of the following is the most appropriate intervention? A. Lithium carbonate 300 mg PO twice daily B. Resection of the anterior pituitary C. Levothyroxin 50 to 100 ug PO daily D. Ferrous sulfate 325 mg PO three times daily E. Radioactive iodine protocol

C The correct choice is C, levothyroxin 50 to 100 ug PO daily. This patient has classic signs and symptoms of hypothyroidism and required thyroid hormone supplementation. The most common form of hypothyroidism is primary hypothyroidism (e.g. Hashimoto's thyroiditis), and the most common thyroid hormone supplementation is levothyroxine. Choice A, lithium carbonate 300 mg PO twice daily, is a medication used in patients with psychiatric disorders and is known to cause hypothyroidism. Choice B, resection of the anterior pituitary, is not indicated without evidence of a tumor or other pituitary pathology. Secondary hypothyroidism related to the anterior pituitary is quite rare. Choice D, ferrous sulfate 325 mg PO three times daily, is a common treatment protocol for patients with iron deficiency anemia. This patient's MCV is elevated, indicating large red blood cells, as seen in disorders such as vitamin B12 deficiency or folic acid deficiency. Patients with iron deficiency anemia present with microcytic hypochromic anemia. Choice E, radioactive iodine protocol, is a treatment used in patients with hyperthyroidism.

A 48 year-old male presents to the clinic with a history of productive cough for the past 3 days. Which of the following factors if present best predicts the diagnosis of pneumonia in this patient? A. Adenopathy B. Double sickening C. Dyspnea D. Fever E. Hoarseness

Cough, fever (D) and sputum production are classic symptoms of pneumonia. Adenopathy (A) and hoarseness (E) may coexist in some cases of viral pneumonia. Dyspnea (C) may develop as pneumonia progresses. Double sickening (B) is a clinical consideration in the differentiation of acute sinusitis from upper respiratory infection that typically occurs after approximately 7 days of illness.

A 34-year-old male presents to the primary care office with a complaint of heartburn. He has symptoms two to three times a week, and it occurs about 30 minutes after eating. He has tried over-the-counter antacids and they were helping to relieve his symptoms for a few months, but they are not working well now. He denies dysphagia, odynophagia, or weight loss. What is the next, most appropriate step in managing this patient? A. Upper endoscopy B. Increase the dose of the antacids C. Barium esophagography D. Treat empirically with a proton pump inhibitor E. Esophageal pH monitoring

D The correct answer is to treat empirically with a proton pump inhibitor. This is the treatment of choice in a patient with typical symptoms of heartburn and regurgitation. If the patient fails therapy with a proton pump inhibitor, or had any alarm symptoms present, then it would be reasonable to start with an upper endoscopy, barium esophagography, or esophageal pH monitoring. Increasing the dose of the antacids is not appropriate management for this patient.

The rotator cuff is comprised of which four muscles?

D Supraspinatus, infraspinatus, teres minor, subscapularis

A 65-year-old female presents to the primary care office with epigastric pain and nausea. Which of the following is a spiral gram-negative rod that resides beneath the gastric mucosa layer and causes gastric mucosal inflammation, with polymorphonuclear cells and lymphocytes? A. Treponema pallidum B. Borrelia burgdorferi C. Corynebacterium diphtheriae D. Helicobacter pylori E. Giardia lamblia

D The correct answer is helicobacter pylori. Treponema pallidum is a spirochete that is capable of infection in almost any organ or tissue in the body, and causes protean clinical manifestations. It is most commonly associated with syphilis. Borrelia burgdorferi is a spirochete and is associated with Lyme disease. Corynebacterium diphtheriae is a bacterium that infects the respiratory tract and is associated with diphtheria. Giardia lamblia is a protozoal infection of the upper small intestine and is associated with giardiasis.

An adult patient was recently diagnosed with type 2 diabetes mellitus. She met with her diabetic educator to discuss suggested changes to her diet and exercise regimens. What percent of her total daily calories should be in the form of carbohydrates? A. 5% to 15% B. 20% to 25% C. 35% to 40% D. 45% to 65% E. 70% to 80%

D The correct choice is D, 45% to 65%. The American Diabetes Association recommends a diet with 45% to 65% carbohydrates, 25% to 35% fats, and 10% to 35% protein.

A patient is being treated for hypothyroidism. His condition has been stable for the past year. What blood test should be ordered and monitored yearly in this patient? A. Total T4 B. T3 resin uptake C. Thyroid releasing hormone D. Thyroid stimulating hormone E. Free T3

D The correct choice is D, thyroid stimulating hormone. This test will help to monitor patient adherence with thyroid hormone supplementation, as well as to fine tune the dose so that the TSH remains within the reference range. Choices A, B, and E can be used in the work up of patients for primary hypothyroidism, but alone each test is not helpful to monitor chronic disease in patients. Choice C, serum thyroid releasing hormone, is used more commonly when investigating secondary hypothyroidism.

A diet high in nitrates is a significant risk factor for cancer of which of the following? A. oropharynx B. esophagus C. stomach D. pancreas E. liver

The Correct Answer is: C In addition to chronic H pylori infections, dietary nitrates are a significant risk factor for gastric cancer.

A female patient presents with a history of hypertension and low plasma HDL. She is asking you if she has metabolic syndrome. She does not have diabetes and she is not obese. Which of the following NCEP ATP III criteria would be needed to confirm that diagnosis? A. LDL > 70 mg/dL B. LDL > 90 mg/dL C. Triglycerides > 100 mg/dL D. Triglycerides ≥ 150 mg/dL E. Triglycerides > 300 mg/dL

The correct choice is D, triglycerides ≥ 150 mg/dL. The NCEP ATP III criteria for metabolic syndrome include three or more of the following: Central obesity with waist circumference > 102 cm in men and > 88 cm in women Hypertriglyceridemia with serum triglycerides ≥ 150 mg/dL, or the patient is taking medication for hypertriglyceridemia Low HDL level < 40 mg/dL in men and < 50 mg/dL in women, or the patient is taking medication for low HDL Hypertension with blood pressure ≥ 130 mm systolic or ≥ 85 mm diastolic, or the patient is taking medication for high blood pressure Fasting plasma glucose ≥ 100 mg/dL, or a positive diagnosis for diabetes mellitus, or the patient is taking medication for hyperglycemia

Which of the following pairs of conditions constitute 90% of the causes of adult hypercalcemia? A. Vitamin D intoxication and renal failure B. Lithium ingestion and hyperthyroidism C. Prolonged immobilization and thiazide use D. Hypothyroidism and hypoparathyroidism E. Primary hyperparathyroidism and malignancy

The correct choice is E, primary hyperparathyroidism and malignancy. Together, they constitute 90% of the causes of hypercalcemia. Choices A, B, and C can cause hypercalcemia, but at a lower rate. Choice D, hypothyroidism and hypoparathyroidism, do not cause hypercalcemia.

A 34-year-old male presents to the primary care office with a complaint of heartburn. He has symptoms two to three times a week, and it occurs about 30 minutes after eating. He has tried over-the-counter antacids, and they were helping relieve his symptoms for a few months, but they are not working well now. In discussing lifestyle modifications with this patient, to help reduce his symptoms, which of the following is recommended? A. Eating larger meals B. Increasing the intake of acidic foods C. Lying down within 30 minutes after eating D. Increasing the intake of fatty foods E. Elevating the head of the bed six inches

E The correct answer is elevating the head of the bed six inches. All of the other choices are known to exacerbate reflux symptoms.

In a boxer's fracture, the presence of how much angulation of the fifth metacarpal neck would require referral for reduction as opposed to treatment with a simple ulnar gutter splint? A. Less than 10 degrees B. 10-19 degrees C. 20-29 degrees D. 30-39 degrees E. Greater than 40 degrees

The Correct Answer is: E When there is greater than 40 degrees of angulation or an extensor lag (the patient cannot fully extend the affected finger) then a referral is required. Lesser degrees of angulation without an extensor lag can generally be handled conservatively with an ulnar gutter.

A 23-year-old patient who has recently been on a ski trip presents with pain to the right hand after sustaining a fall. It is difficult to move, and there is pain on flexion of the digit. Based on this history what ligament would the patient most likely have injured? A. 1st MCP joint ulnar collateral ligament B. 2nd MCP joint ulnar collateral ligament C. 3rd MCP joint ulnar collateral ligament D. 4th MCP joint ulnar collateral ligament E. 5th MCP joint ulnar collateral ligament

The Correct Answer is: A The ulnar collateral ligament at the base of the thumb, or 1st MCP joint, is often injured in forced abduction, such as a fall while skiing or during other sporting activities. An injury to this ligament has traditionally been called Gamekeeper's Thumb, but the origin of this term referred to a more chronic injury sustained by English gamekeepers as a result of the way they killed rabbits using their hands. Any of the MCP joint ulnar collateral ligaments could be injured in a fall if the mechanism of injury creates significant forces on the ligaments, but the 1st MCP joint is far more commonly injured than the others mentioned above.

A 23-year-old patient presents to the Emergency Department complaining about a bloody left eye that occurred after being struck with a tennis ball. The patient denies any decreased vision or photophobia. A photo of the left eye reveals the following (see below). Which of the following is the most appropriate management for this patient? A. CT scan of orbit B. Observation and reassurance C. Olopatadine 0.1% ophthalmic solution twice daily D. Refer to ophthalmology E. Sulfacetamide 10% ophthalmic solution three times a day for 5 days

The Correct Answer is: B The classic appearance of a bright red, flat collection of blood is consistent with a subconjunctival hemorrhage best managed with observation and reassurance (B) by the primary care provider. The mechanism of injury is inconsistent with orbital fracture (A), and the patient lacks infectious (E) or allergic (C) conjunctivitis.

An 11-year-old boy is being seen in the clinic for well-child care. His father inquires whether his son is starting to show physical signs of puberty. Which of the following is the first sign of puberty in males? A. change of voice B. scrotal and testicular enlargement C. gynecomastia D. pubic hair development

The Correct Answer is: B s crotal and testicular enlargement is the first sign of puberty in boys. This typically occurs at the average age of 10 to 12 years.

A 64-year-old male presents with right knee pain and stiffness while walking and going up and down stairs for the past six months. He states the pain persists all day long and is relieved with rest. There is no history of trauma and his past medical history is unremarkable. An x-ray of the right knee is performed and the following is observed: What is the suspected diagnosis based on the imaging above? A. Bursitis B. Osteoarthritis C. Torn posterior cruciate ligament D. Oblique tibia fracture E. Spiral femur fracture

The correct answer is (B). Osteoarthritis (OA) is the most common cause of chronic knee pain over 45 years of age and commonly seen in weight-bearing joints when walking or climbing stairs. As disease progresses, it becomes continuous and present at night. OA in the x-ray shows a narrowed joint space (white arrow) on the medial side of the knee (B); sclerosis of the bone in the medial compartment (black arrow), which is evidence of cortical thickening; and formation of osteophytes in the medial femur (white wedge). Soft tissue injuries including a PCL tear (C) and bursitis (A), would not usually be identifiable on x-ray and would need further imaging, such as a CT or MRI. No fractures, (D) and (E), are identified on this image.

A 78-year-old male returns to the FP office for a follow up of non-insulin-dependent diabetes mellitus (NIDDM) as a new patient to you, although he has been an office patient for the past year. He denies any problems this visit and says his blood sugars are in the 90-120 mg/dl range. He is currently taking the medications listed in the following choices. You receive his labs and note that his creatinine is 2.0 mg/dl and on the previous few labs this creatinine was also in the 1.8-2.0 mg/dl range. What medication should be discontinued? A. glipizide B. metformin C. omeprazole D. sitagliptin E. atenolol

The correct answer is (B). Metformin is contraindicated in this diabetic patient with chronic renal failure due to an increased risk of lactic acidosis and should be discontinued. Sitagliptin requires a dosing adjustment in renal failure but is not contraindicated in this patient. Glipizide (sulfonylurea), omeprazole (a PPI for GERD), and atenolol (a beta blocker for hypertension) are not contraindicated in this patient. Beta blockers should be used with caution in diabetics due to the potential of masking symptoms of hypoglycemia, but are not contraindicated.

A 36-year-old nurse states she will be traveling to Honduras for a medical mission and wants to ensure that she has adequate protection against hepatitis B. Which of the following laboratory values is consistent with immunity against hepatitis B from prior immunization? A. +HBsAg, +anti-HBc IgM B. +HBsAg, +anti-Hbc IgG C. +anti-HBs D. +anti-HBs, +anti-HBIgG E. +anti-HCV

The correct answer is (C). The presence of detectable anti-HBs indicates that she is immune from a previous hepatitis B vaccine. Booster immunizations are not indicated in her case. Choice (A) suggests acute hepatitis B. Choice (B) suggests she has highly infective chronic hepatitis B. Choice (D) suggests she had previous hepatitis B infection, recovered. Choice (E) suggests acute hepatitis C.

You are a physician assistant working in family practice. A 54-year-old male with a history of hypertension comes to your office complaining of fatigue. Which of the following medications is the most likely cause of the complaint and the EKG findings below? Source: (Gomella and Haist, 2007, Chapter 19) A. hydralazine B. losartan C. metoprolol D. captopril E. terazosin

The correct answer is (C). This patient has a sinus bradycardia with a heat rate of about 38 bpm. Metoprolol is a beta blocker known to cause bradycardia. ARBs (such as losartan) and ACE inhibitors (such as captopril) are unlikely to cause bradycardia. Hydralazine an alpha blocker that may cause symptomatic hypotension, but it is unlikely to cause bradycardia. Hydralazine, a direct vasodilator, is more likely to cause tachycardia. Terazosin does not have an effect on rate.

Consideration should be given to screening patients with type 1 diabetes mellitus should also be screened for which of the following: A- sarcoidosis B- Sheehan's syndrome C - Sjögren's Syndrome D- thyroid disease

The correct answer is (D). Type 1 diabetes mellitus (T1DM) is an autoimmune disease. As such, patients have a significantly higher risk of other autoimmune diseases, including celiac and thyroid disease. Most recommendations include screening for both diseases in patients with T1DM.

A 28-year-old female presents with a gradual progression of fatigue and pallor over the last few months. Initial CBC results show a hemoglobin of 10.4 mg/dL, hematocrit of 32%, an MCV of 112 fL, and a reticulocyte count of 3%. Which of the following is the most likely pathophysiologic mechanism responsible for her anemia? A. Chronic blood loss B. Defective bone marrow/stem cell function C. Defective DNA production D. Defective hemoglobin production E. Increased destruction of red blood cells

The correct answer is (E). The time course of the patient's presentation is consistent with multiple episodes of acute hemolysis. Defects in bone marrow (B) or red blood cell precursors (C and D) are refuted by the elevated reticulocyte count. Chronic blood loss (A) would have a more insidious, gradual onset and likely result in a decreased MCV.

Your patient is a 26-year-old male in whom you've diagnosed major depression. You wish to start him on pharmacologic therapy, but he expresses significant concern regarding sexual side effects. Which of the following depression medications is an option that will reduce the risk of sexual dysfunction? A. Bupropion B. Citalopram C. Fluoxetine D. Paroxetine E. Venlafaxine

The correct answer is bupropion (A). All the SSRIs, including citalopram (B), fluoxetine (C), and paroxetine (D), as well as venlafaxine (E), which is a combination serotonin and norepinephrine reuptake inhibitor (SNRI), have high rates of sexual side effects for men and women. Bupropion is a norepinephrine and dopamine reuptake inhibitor and can be helpful in averting or reducing both sexual side effects and weight gain. It is also indicated for smoking cessation.

A combination of which of the following medications may increase the risk of muscle and liver disease more than either drug used alone? A. Ezetimibe and HMG-CoA reductase inhibitor B. Low dose niacin and HMG-CoA reductase inhibitor C. Colestipol and gemfibrozil D. Niacin and cholestyramine E. Gemfibrozil and HMG-CoA reductase inhibitor

The correct choice is E, gemfibrozil and HMG-CoA reductase inhibitor. In combination, these can potentiate the risk of developing hepatic disease or myopathy. Choice A, Ezetimibe and HMG-CoA reductase inhibitor, is a synergistic treatment plan for patients with primary hypercholesterolemia. Choice B, low dose niacin and HMG-CoA reductase inhibitor, is a practical and effective treatment plan for patients with familial combined hyperproteinemia. Choice C, colestipol and gemfibrozil, is sometimes useful in patients with familial combined hyperlipidemia who are intolerant of niacin or HMG-CoA reductase inhibitors. Unfortunately, this combination may increase the risk of cholelithiasis. Choice D, niacin and cholestyramine, is useful in disorders with elevated VLDL and LDL, and useful in treating heterozygous familial hypercholesterolemia.

A 45-year-old patient came in to see his health care provider today, to discuss the results of his last annual assessment. He was told that he had developed type 2 diabetes mellitus. One of the recommendations from the physician assistant included a visit to an ophthalmologist. The physician assistant was concerned after seeing new capillaries, macular edema, and fibrous tissue within the retina during his funduscopic exam. What type of ocular complication does this patient most likely have at this time? A. Background retinopathy B. Closed angle glaucoma C. Macular degeneration D. Diabetic cataracts E. Proliferative retinopathy

The correct choice is E, proliferative retinopathy. The distinguishing factor in the patient's presentation, which signals this disorder, is the development of newly formed vessels. Proliferative retinopathy is the leading cause of blindness in the United States. Up to 20% of patients with type 2 diabetes have retinopathy at the time of diagnosis. Choice A, background retinopathy, or simple retinopathy includes retinal microaneurysms, hemorrhages, exudates, and edema, without new vessel formation. Choice B, closed angle glaucoma, is relatively uncommon in patients with diabetes, except after cataract extraction. Choice C, macular degeneration, is not associated with diabetes mellitus specifically. Choice D, diabetic cataracts, tends to occur in patients with diabetes earlier than the general population, and may correlate with the severity of the disease.

A 52-year-old woman presents with vaginal discharge that is white curd-like in appearance but is not malodorous. She has a 19-year history of obesity and poorly controlled type 2 diabetes mellitus. Microscopic examination of the discharge with 10% potassium hydroxide demonstrates filaments and spores. Which of the following is the most likely etiologic agent? A. Candida B. Gardnerella C. Lactobacillus D. Staphylococcus epidermidis E. Trichomonas vaginalis

The Correct Answer is: A This case has several clues pointing to a Candida infection, including the fact that diabetes mellitus can predispose patients to Candida infections and the presence of the white curd-like discharge that is not malodorous. In Trichomonas vaginalis, the discharge is malodorous and yellow-green in color. With Gardnerella, there is also a malodorous discharge. Lactobacillus is the predominant, normal microorganism of the vagina and keeps it slightly acidic to help reduce the growth of potentially harmful organisms. Staphylococcus epidermidis is also part of the natural flora of the vagina.

A 23-year-old male with a recent diagnosis of Non-Hodgkin's lymphoma presents complaining of swelling of the neck and face, cough, and dyspnea on exertion. On exam you note dilated neck veins. Which of the following is the most appropriate initial treatment for this patient? A. Beta-blockers B. Glucocorticoids C. IV fluids D. Trendelenburg position E. Vena cava stenting

The Correct Answer is: B The classic presentation signs and symptoms of superior vena cava syndrome (SVCS) are present in this patient. Patients with lymphoma often respond with a decrease in tumor mass and improvement in SVCS with glucocorticoid therapy. Beta-blockers (A) and vena cava stenting (E) aren't routinely used in the management of SVCS. Other initial symptomatic therapies include low salt diet, diuretics, head elevation, and diuretics. IV fluids (C) and Trendelenburg position (D) are contraindicated.

A 63 year-old woman presents to the clinic seeking counseling and medication to help quit smoking. Which of the following tobacco cessation agents binds with high affinity and selectivity at a4β2 neuronal nicotinic acetylcholine receptors? A. buproprion B. Clonidine C. nicotine replacement D. nortriptyline E. varenicline

The Correct Answer is: E Varenicline (E) is a selective partial agonist of the a4β2 neuronal nicotinic acetylcholine receptors and relieves withdrawal symptoms while also blocking the binding of nicotine (from continued tobacco use) to the receptor. Clonidine (B) is a centrally acting alpha-adrenergic agonist. Bupropion (A) and nortriptyline (D) are antidepressants that have shown efficacy in supporting smoking cessation efforts. Bupropion is FDA approved for smoking cessation, while clonidine and nortriptyline are not FDA approved for this indication.

A parent brings her child into the office with the concern of the way he stands. To the parent the child looks like they are "bowlegged." If the child is indeed bowlegged, what would be the best description of this orthopedic abnormality? A- Angulation of an extremity at a joint with the more distal part angled anteriorly. B- Angulation of an extremity at a joint with the more distal part angled away from the midline. C- Angulation of an extremity at a joint with the more distal part angled toward the midline. D- Angulation of an extremity at a joint with the more distal part angled posteriorly. E- Angulation of an extremity at a joint with the more proximal part angled away from the midline.

The correct answer is (C). A varus deformity involves angulation of an extremity at a joint with the more distal part angled toward the midline. This is the type of deformity found in someone who is described as "bow-legged."

A 30-year-old healthy female's labs return showing an HDL of 28 mg/dl, LDL of 132 mg/dl, and total cholesterol of 185mg/dl. She is concerned due to a family history of coronary artery disease (CAD) on her father's side in his 60s. She denies tobacco use. Her blood pressure is 108/50, P-64. She requests advice on what she can due to increase her HDL. What do you advise her? A. atorvastatin B. cholestyramine C. ezetimibe D. gemfibrozil E. exercise

The correct answer is (E). This patient is at very low risk of CAD based on the 10-year Framingham projections. Based on her < 1%, 10-year risk drug therapy is not indicated. Exercise has been shown to increase HDL. HDL is lower in patients who have a sedentary lifestyle or are obese. Her LDL may also be reduced with exercise, weight loss, and dietary modifications.

Your patient is a 26-year-old mother of two young children whose second child was just delivered two weeks ago and she is breast-feeding. She complains of insomnia and depression. She denies the use of any drugs or alcohol. You would like to hold off on pharmacologic therapy unless her symptoms progress. Which of the following will be your first line of treatment? A. Encourage more time with the baby B. Encourage her to begin planning to get back to work C. Promote adequate sleep D. Suggest she get out of the house more often E. Suggest her husband help more around the house

The correct answer is to promote adequate sleep (C). Postpartum "blues" are very common. Depression can occur in some cases, but most resolve without therapy. When required, SSRIs may be used even when a woman is breast feeding, though no studies have been done.

A mother expresses concern for her teen son after feeling a lump in his neck. He has no history of trauma to his neck. Surgical history is negative, and the patient does not take any medications. The mother tells you that thyroid problems run in the family. The patient has not been ill recently. Upon exam you feel a nontender, firm nodule on the right side of his thyroid with associated cervical lymphadenopathy. His serum TSH level is within the reference range. Radionuclide thyroid scanning demonstrates a "cold" nodule in the right side of the thyroid. What is the most appropriate next step in the work up of this patient? A. MRI of the anterior pituitary B. CT of the thyroid C. MRI of the thyroid D. Thyroid nodule fine needle aspiration E. Emergent thyroidectomy

The correct choice is D, thyroid nodule fine needle aspiration. With the advent of fine needle aspiration, it has become much easier, safer, and more reliable to obtain a specimen for biopsy. This patient has several characteristics that increase his risk of malignancy including his gender, young age, firmness of the nodule, and related lymphadenopathy. These, along with the ease of biopsy, suggest this path for diagnostic work up. Not enough information is known to warrant an emergent thyroidectomy, choice E. Choice A, MRI of the anterior pituitary, would be warranted if there was a suspicion of a pituitary cause of the thyroid nodules. Since the TSH is normal and the patient is not presenting with headaches or other pituitary related symptoms, this is not suggested. Choice B, CT of the thyroid, and choice C, MRI of the thyroid, would not provide any additional information after the thyroid scan. They may be helpful prior to any surgery if needed.

A 63 year-old woman complains of fatigue, loss of appetite, a sore-red tongue, paresthesias of her feet and hands, and unsteadiness of her gait. Which of the following tests is be used to confirm the patient's suspected diagnosis? A. Anti-intrinsic factor antibodies B. Antiparietal cell antibodies C. Coomb's test D. Schilling Test E. Serum folate levels

The Correct Answer is: A A diagnosis of pernicious anemia (PA) can be supported through the presence of anti-intrinsic factor antibodies. Antiparietal (B) cell antibodies aren't associated with the development of PA. The Coomb's test (C) is used in the evaluation of hemolytic anemias. Serum folate (D) levels are beneficial in evaluating macrocytic anemias, but will not establish a diagnosis of PA. The Schilling test (D) was once commonly used to diagnosis PA, but is no longer available due to lack of available radiolabeled human intrinsic factor.

A 20-year-old college football player presents with a chief complaint of a dull ache in his scrotum after prolonged standing on the sideline. It seems to get worse with vigorous activity and is relieved by lying down. Dilated veins in the left scrotum are observed on inspection, and both testicles are palpable and without masses. What is the most likely diagnosis? A. varicocele B. spermatocele C. hydrocele D. testicular mass

The Correct Answer is: A A varicocele can be recognized by the presence of scrotal enlargement caused by dilation of the pampiniform venous plexus. Varicoceles present as a "bag of worms" in the spermatic cord and are more prominent when the patient stands. More than 80% of the time, varicoceles occur on the left side. Hydroceles and spermatoceles are caused by fluid collection and are usually asymptomatic. Testicular masses must always be included in the differential diagnosis of scrotal masses, as they generally present as painless.

A 45-year-old male comes to your primary care office complaining of recurrent panic attacks that have led to a disabling fear of being in places from which escape might be difficult, such as a bus or a train. He now finds he can barely leave his home. You diagnose him with panic disorder and which of the following complications? A. Agoraphobia B. Generalized anxiety disorder C. Obsessive-compulsive disorder D. Posttraumatic stress disorder E. Social phobia

The Correct Answer is: A Agoraphobia (A) is a complication of panic disorder in which the attacks are associated with being in a crowd or around others. Generalized anxiety disorder (B) is a more overarching term, but does not necessarily include panic attacks. Obsessive-compulsive disorder (C) involves recurrent intrusive thoughts and rituals. Social phobia (E) is more focused on performance in normal social situations and post-traumatic stress (D) requires a traumatic trigger event.

A 58 year-old man with a hemoglobin of 11.1 mg/dL and hematocrit of 33% is noted to have a serum folic acid level of 0.2 ng/dL. Which of the following risk factors for folate deficiency should this patient be assessed for? A. Alcoholism B. Diabetes C. Down's syndrome D. Smoking E. Vegetarian diet

The Correct Answer is: A Alcoholism and the nutritional status that results is a common cause of folate deficiency. Diabetes (B) and smoking (D) are not directly associated with folate deficiency. Down's syndrome (C) risk can be reduced through the administration of prenatal folic acid. A vegetarian (E) diet provides excellent sources of folate.

A 20-year-old male presents to the emergency department complaining of pain to the right shoulder region while playing basketball. He states that his arm was pulled back and rotated while he was moving forward, and then felt a popping sensation in the shoulder. Since then he has not been able to move the shoulder at all due to pain and immobility. It is suspected that he has a dislocation. Given this scenario, what would be the most likely type? What type of shoulder dislocation did he most likely experience? A. Anterior B. Inferior C. Multidirectional D. Posterior E. Superior

The Correct Answer is: A All of the various types of dislocations mentioned above are possible, but anterior dislocations are by far the most common (>95%) and they are the most common of all joint dislocations. Most occur as a result of a fall or other traumatic event and they may become recurrent. The shoulder is most susceptible to an anterior dislocation when it is abducted and externally rotated. The shoulder joint is considered a very mobile joint, but this also renders it very susceptible to injury.

A 19-year-old male is brought by his mother to your primary care office. She is at her wits' end as he has just gotten out of juvenile detention but continues to engage in the behaviors that resulted in his incarceration. He lies, cheats, steals, and seems to disregard the rights and needs of others to the point of endangering their safety. This pattern has been present since early childhood, since at least age four. He does not feel that there is a problem, but his family and others around him are disturbed by his attitudes and behaviors. What is the most likely diagnosis? A. Antisocial personality disorder B. Borderline personality disorder C. Histrionic personality disorder D. Narcissistic personality disorder E. Schizoid personality disorder

The Correct Answer is: A All personality disorders involve a persistent pattern of behavior in which there is a disturbance in impulse control, interpersonal behavior, interpretation of people and events, and/or emotional response that begins no later than adolescence. Antisocial PD involves a reckless disregard for others, usually manifesting in lawlessness, lying, cheating, and a lack of remorse. Borderline PD (B) involves interpersonal reactivity and impulsivity often focused on perceived abandonment. Histrionic PD (C) involves a need to be the center of attention, often manifesting in behavior that is theatrical or seductive. Narcissistic PD (D) involves a lack of empathy and grandiosity. Schizoid PD (E) involves symptoms similar to schizophrenia.

A 43 year-old woman is brought to the emergency department in critical care secondary to a traumatic brain injury. Which of the following best describes her Cheyne-Stokes respiratory pattern? A. Alternating periods of shallow and deep breathing B. Difficult or labored breathing C. Difficult of labored breathing while supine D. Periods of absent breathing E. Sudden awakening due to shortness of breath

The Correct Answer is: A Cheyne-Stokes respirations are characterized by shallow breaths that increase in rate and depth followed by periods of apnea (A). Dyspnea is difficult or labored breathing (B) and if it occurs while supine (C) is termed orthopnea. Apnea is noted periods of absent breathing (D). Sudden awakening due to shortness of breath (E) is paroxysmal nocturnal dyspnea.

A 52-year-old male has a 200-pound file cabinet fall on his right leg. He comes to the emergency department complaining of pain and swelling to the right leg. He is also complaining of parasthesias to the leg also. Based on these findings on history, what part of the leg would be the most likely site of compartment syndrome? A. Anterior compartment of the leg B. Deep posterior compartment of the leg C. Dorsal compartment of the forearm D. Lateral compartment of the leg E. Superficial posterior compartment of the leg

The Correct Answer is: A Anterior compartment syndrome is most commonly found in the anterior compartment of the leg, with the volar compartment of the forearm also a common location. The mechanism is generally an acute crushing trauma to the affected area that causes an increase in pressure within the compartment that inhibits venous outflow and a decrease in arterial blood flow. This adversely affects tissue perfusion and ischemia of the involved tissues can occur. The symptoms can also occur with chronic exertion or when there is a dramatic increase in the amount of exercise being performed. Anterior compartment syndrome of the leg is likely most common due to its vulnerable location and susceptibility to injury in athletics and motor vehicle accidents. The rate of occurrence of acute anterior compartment syndrome is also likely due to the fact that the anterior compartment lies adjacent to the tibia and the tibia is the most frequently fractured long bone. Compartment syndrome could occur in any compartment of the upper or lower extremities if the necessary circumstances were in place (either acute trauma to the area or excessive use of the muscles that are associated with each of the compartments).

Out of all cervical vertebrae, which two are responsible for the greatest amount of rotation? A. C1 & C2 B. C2 & C3 C. C3 & C4 D. C4 & C5 E. C5 & C6

The Correct Answer is: A Approximately 50% of cervical rotation takes place between the C1 (atlas) and C2 (axis) vertebrae. These first two cervical vertebrae have a different shape from the other cervical vertebrae that allow for this greater range of motion. The remaining 50 % of cervical rotation is split fairly evenly between the remaining vertebrae. Approximately 50 % of flexion and extension occurs between the occiput at the base of the skull and C1 with the remaining 50% distributed fairly evenly between the remaining vertebrae with a slightly higher percentage occurring at the C5 & C6 level.

A 36-year-old female presents to your family practice office concerned about a breast lump she discovered in the shower last night. You obtain a detailed history and these are some of your findings: • G2P2 • LMP: 2 weeks ago, normal • Sister diagnosed with breast cancer at age 42. • Mother is currently being treated for ovarian cancer. • Father is of Ashkenazi Jewish ancestry. Which of the following is an appropriate recommendation for this patient? A. Enhanced surveillance, including mammography alternating with MRI, every six months. B. Genetic counseling and testing for BRCA1 and BRCA2 status. C. Referral to a breast surgeon for mastectomy D. Chemoprophylaxis with an aromatase inhibitor weekly.

The Correct Answer is: A Because of her ethnicity (Ashkenazi Jewish descent) and family history (two first degree relatives with breast and ovarian cancers), this patient is at a higher than normal risk for breast and ovarian cancers. With a physical finding of a palpable breast mass, this is even more ominous. She should be referred for an immediate mammogram but not necessarily MRI - and have continued enhanced surveillance annually and not every 6 months regardless of the result - as well as a referral to a breast surgeon for immediate consideration of a biopsy. A mastectomy is a possible consideration but not mandatory. She is certainly an appropriate candidate for consideration of referral to a genetic counselor and genetic testing to ascertain her BRCA1 and BRCA2 status. She is also an appropriate candidate for chemoprophylaxis.

Which of the following joints has the lowest occurrence rate of osteoarthritis? A. Elbows B. Hands C. Hips D. Knees E. Spine

The Correct Answer is: A Because the elbow is not a weight bearing bone, the rate of osteoarthritis in the elbow is considerably less that what is found in locations like the hips, knees and spine. The hands have one of the highest rates of occurrence of osteoarthritis, likely due to their near constant use and propensity for minor (or major) injury. When elbow arthritis does develop it is often post-traumatic osteoarthritis related to a significant injury in the past that disrupted joint surface integrity or as a result of rheumatoid arthritis, a systemic illness. Osteoarthritis of the elbow will generally present with pain, stiffness, and decreased range of motion. Osteophytes that form on the medial elbow might be implicated should neurological symptoms develop that correlate with ulnar nerve distribution as this nerve does pass in close proximity to the elbow on the medial side.

A 58-year-old male comes to your primary care office complaining of sadness, insomnia, loss of appetite, weight loss, and feelings of guilt or hopelessness for the past month. On further questioning you find that his wife of 30 years died of breast cancer shortly before the onset of symptoms. Which of the following is the most likely diagnosis? A. Bereavement B. Dysthymia C. Depressive episode D. Depressed mood E. Major depression

The Correct Answer is: A Bereavement (A) is the normal grief response experienced after a significant loss and includes all the symptoms of depression and, by definition, lasts no longer than two months, although many will suffer from some symptoms for longer. Dysthymia (B) is a longstanding depressed mood for at least two years but not meeting the full criteria for a depressive episode (C). Major depression (E) requires at least one depressive episode, which requires at least five of the symptoms, one of which is depressed mood (D).

A 64-year-old female presents for an annual examination. She is 5'0" and weighs 92 pounds; compared to her examination 3 years ago she has lost an inch in height. After performing a dual-energy x-ray absorptiometry (DXA) scan the diagnosis of osteoporosis is confirmed. Which of the following supplements should be recommended? A. Calcium carbonate 1,000 mg and vitamin D 800 international units daily B. Calcium carbonate 500 mg and vitamin D 2,500 international units daily C. Calcium carbonate 2,000 mg and vitamin D 500 international units daily D. Calcium carbonate 800 mg and vitamin D 100 international units daily

The Correct Answer is: A Calcium supplementation provides beneficial effects on bone mass throughout postmenopausal life and may reduce fracture rates up to 50%. Postmenopausal women receiving supplemental calcium over a 3-year period in a placebo-controlled, randomized clinical trial had stable total body calcium and BMD in the lumbar spine, femoral neck, and trochanter compared with the placebo group. Recommendations for calcium carbonate are 1,000-1,500 mg once daily orally. Vitamin D increases calcium absorption in the gastrointestinal tract, making calcium more available for reabsorption and circulation. Recommendations for Vitamin D are 800-2,000 international units once daily orally. Calcium Carbonate less than 1,000 mg per day is not sufficient (B and D). Vitamin D levels less than 800 IU per day is not sufficient (C and D).

A 42 year-old woman presents complaining of shortness of breath, three days of fever as high as 103F, and has a cough productive of green sputum. On physical examination, you hear crackles in her lungs. A chest radiograph reveals a consolidation in the left lower lobe. What do you expect to hear when you percuss this patient's left lower thorax? A. Dull B. Flat C. Hyperresonant D. Resonant E. Tympanic

The Correct Answer is: A Consolidation leads to dullness (A) to percussion. Flat sounds (B) are noted with organ percussion (i.e., hepatic flatness), hyperresonant (C) sounds are noted in pneumothorax, normal lung parenchyma results in resonance (D), and percussion of the gastric air bubble leads to tympanic sounds.

A 32-year-old female professional golfer presents with a 3-week history of pain along her thumb and down her wrist. She denies any trauma and states that it is aggravated with any movement of her wrist and thumb. Her physical examination is unremarkable. You perform the following test: This test requires her to cup her thumb in a closed fist and ulnar deviate, which reproduces her pain. Considering the suspected diagnosis based on the positive examination above, what are the affected anatomical structures? A. Extensor pollicis brevis and abductor pollicis longus B. Flexor pollicis longus and flexor digitorum profundus C. Extensor carpi radialis longus and brevis D. Ulnar collateral ligament

The Correct Answer is: A De Quervain tenosynovitis is a common condition that occurs in patients who have experienced excessive use of the thumb or wrist. This is a tenosynovitis of the extensor pollicis brevis and abductor pollicis tendons (A), where the tendons lie in the groove of the radial styloid. The diagnosis of De Quervain tenosynovitis is supported by a history of pain in this location along with a painful range of motion of the thumb and further confirmation may be provided by a positive Finkelstein test. The flexor pollicis longus and flexor digitorum profundus (B) are affected in carpal tunnel syndrome. Extensor carpi radialis longus and brevis (C) are extrinsic extensor muscles of the hand. The ulnar collateral ligament (D) is injured in Gamekeeper's thumb.

A father brings his son to the family practice clinic for a concern for coarctation of the aorta. The father was recently diagnosed with this disease, and he is concerned that his 5-year-old son may have it as well. What is the cardinal physical finding in coarctation of the aorta? A. Decreased or absent femoral pulses B. Systolic machinery-type murmur C. Holosystolic murmur at the lower left sternal border D. Thrill in the carotid arteries E. Cyanosis at birth

The Correct Answer is: A Decreased or absent femoral pulses is the correct answer. A systolic machinery-type murmur is classic for a patent ductus arteriosus. A holosystolic murmur at the lower left sternal border is most commonly seen with a ventricular septal defect. A thrill in the carotid arteries is associated with aortic stenosis. Cyanosis at birth can occur with many different congenital anomalies, but is not seen with coarctation of the aorta.

Patients in which of the following age groups are least likely to experience a dislocation or sprain when a significant stress is placed on their joints? A. 5-10 years old B. 15-20 years old C. 35-40 years old D. 55-60 years old E. 75-80 years old

The Correct Answer is: A Dislocations and ligamentous injuries are uncommon in prepubertal children as the ligaments and joints are quite strong as compared to the adjoining growth plates. Excessive force applied to a child's joint is more likely to cause a fracture through the growth plate than a dislocation or sprain.

You are getting ready to evaluate a patient who has a past history of gout. The chief complaint is that the patients gout has flared up again, causing pain. Based on your knowledge of gout, which joint is most commonly the site of an initial gout attack? A. 1st metatarsaphalangeal joint B. Knee C. Wrist D. Elbow E. Ankle

The Correct Answer is: A Eighty percent of gout attacks affect only one joint. The most common joint to be involved is the 1st metatarsal phalangeal joint. This common phenomenon is called podagra. Gout can affect other joints as well, including the knee, ankle and tarsal joints of the foot. Upper extremities are not commonly affected by gout. The knee is the most common joint to be affected by pseudogout followed by the wrist, MCP joints of the hands, hips, shoulders, elbows and spine.

A 40- year-old female patient comes to the office because she notices that she is easily fatigued but cannot pinpoint a direct cause of her fatigue. She has experienced muscle tenderness to the shoulders and other large muscle groups, sensitivity to touch these areas, and has also felt depressed lately. Based on this history and the vague physical exam findings, what is the most likely diagnosis? A. Fibromyalgia B. Polyarteritis nodosa C. Polymyositis D. Scleroderma E. Sjogren's

The Correct Answer is: A Fibromyalgia syndrome (FMS) primarily affects woman between the ages of 20 and 60 and is now the second most common condition seen in rheumatologists' offices behind only rheumatoid arthritis. This condition spares the joints, but causes tender areas throughout the soft tissue as well as generalized pain and fatigue. There are several common trigger points, both anteriorly and posteriorly that help confirm the diagnosis. Polyarteritis nodosa is a systemic necrotizing vasculitis that generally affects medium size muscular arteries. Patients can present with many of the same symptoms as those with fibromyalgia (fatigue, muscle pain, and others), but these patients often have skin manifestations, joint pain, and fever, along with renal, GI, and cardiovascular signs not seen in fibromyalgia. Polymyositis is an inflammatory condition of the muscles that is characterized by proximal muscle weakness, with much less pain than is seen in fibromyalgia. Despite the proximal muscle weakness there is generally not significant atrophy. Sjogren's syndrome does not generally present with any muscular manifestations. It is a chronic inflammatory disorder that affects the salivary and lacrimal glands resulting in dry eyes and a dry mouth. Scleroderma is classically associated with thickened and hardened skin, but it can have internal organ involvement as well affecting many different body systems. When it affects the musculoskeletal system it tends to affect joints and the areas where tendons cross joints. It can cause contractures, pain, and swelling as well as fatigue and weakness.

A 66-year-old female is admitted to the hospital with atrial fibrillation. Her past medical history has hypertension, diabetes mellitus type II, hypercholesterolemia, and rheumatoid arthritis. During her evaluation for work up it is noted that the patient has had recurring symptomatic episodes of atrial fibrillation for the last year, some resulting in the patient not being able to ambulate due to hypotensive events. Her current blood pressure is 146/83, and her heart rate is 87. Given this clinical scenario, what is the most appropriate procedure for this patient? A. Ablation therapy B. Cardiac catheterization C. Synchronized cardioversion D. Permanent pacemaker insertion E. Pacemaker and defibrillator insertion

The Correct Answer is: A The clinical scenario for patients who have recurrent symptomatic episodes related to atrial fibrillation is ablation therapy (A). Cardiac catheterization (B) is sometimes performed based on suspicion of coronary artery disease, but does not address the electrophysiological reasons for the patient's episodes. Cardioversion (C) would not address a patient who is having recurrent episodes, and there is no clinical indication for pacing or defibrillation needs (D and E).

A 45-year-old obese male presents with pain and swelling to his left knee. It is reddened and tender to touch. He states that this has happened before but the swelling had gone down over the course of several days. Anti-inflammatories have helped some in the past. An analysis of the arthrocentesis fluid reveals needle-shaped crystals. Based on the results of the fluid analysis found within the synovial fluid, what is the most likely diagnosis? A. Gout B. Lupus C. Osteoarthritis D. Pseudogout E. Rheumatoid arthritis

The Correct Answer is: A Gout is a condition found in people that have hyperuricemia (but not all people with hyperuricemia will develop the symptoms of gout). Hyperuricemia is defined as an elevated uric acid level in the blood. Uric acid is a product of purine metabolism. Gout occurs when needle-shaped monosodium urate crystals deposit in soft tissue, which leads to pain and inflammation at the deposition site. The diagnosis can be confirmed by analyzing joint aspirate to identify the classic negatively birefringent urate crystals that have a needle-like shape. Pseudogout can present with a similar clinical picture, but an analysis of joint aspirate shows calcium pyrophosphate crystals that are weakly positive, birefringent rhomboid shaped crystals. Lupus, osteoarthritis, and rheumatoid arthritis all can cause joint inflammation and pain, but none would show any significant crystal deposition during an analysis of a synovial fluid aspirate.

A 23-year-old college basketball player twists her ankle while practicing. She explains the injury that is consistent with an inversion mechanism. Based on this history, what ligament would you expect to be the most likely injured in the ankle of this patient? A. Anterior talofibular B. Anterior tibiofibular C. Calcaneofibular D. Deltoid E. Posterior talofibular ligament

The Correct Answer is: A Greater than 25,000 ankle sprains happen in the USA every day and the vast majority of those are inversion ankle sprains. The anterior talofibular ligament is the first, and often only, ligament damaged in inversion ankle sprains. As the force of the inversion increases, other lateral ankle ligaments can be involved. When the anterior tibiofibular ligament is involved, this is referred to as a high ankle sprain and such injuries generally have a prolonged recovery time. Calcaneofibular ligaments are generally the second most frequently injured of the lateral ankle ligaments and when injury occurs it is typically in combination with the anterior talofibular ligament. The deltoid ligament is a very strong ligament on the medial aspect of the ankle. Eversion stresses the deltoid ligament, but strong eversion forces are rare and when they do occur, an avulsion fracture of the medial malleolus is more likely than a significant ligament tear. The posterior talofibular ligament is one of the lateral ankle ligaments and can be injured in an inversion injury, but the rate of injury to this ligament lags far behind the anterior talofibular or calcaneofibular ligaments.

A 12 year-old African-American male with a history of sickle cell disease presents to the clinic for routine follow up. A peripheral smear is performed. Which of the following best describes the cause of the abnormality pictured in the patient's peripheral smear? (note photo taken from figure 29-12 in Harrison's) A. Absent or non-functioning spleen B. Failure of nuclear maturation C. Intravascular hemolysis D. Lead intoxication E. Presence of uremia

The Correct Answer is: A Howell-Jolly bodies are noted in the peripheral smear and result from the lack of removal of nuclear material due to an absent or non-functioning spleen. Failure of nuclear maturation (B) produces macrocytosis, intravascular hemolysis (C) leads to the creation of schizocytes, lead intoxication (D) and thalassemia cause basophilic stippling, and uremia (E) is associated with Burr cells.

A 57-year-old woman with a history of rheumatic fever is seen complaining of dyspnea while vacuuming her apartment, which has been worsening over the last few months. On physical exam, a possible opening snap, loud S 1 , and a very soft diastolic rumbling murmur is auscultated. When the patient is placed in the left lateral decubitus position, the murmur is accentuated, and heard best at the apex. With inspiration, the murmur does not increase in amplitude. On transthoracic echocardiogram, severe mitral valve stenosis and mitral regurgitation is noted. Which of the following is the most appropriate therapy or treatment for this patient? A. Cardiac catheterization, followed by mitral valve replacement B. Monitoring via repeat transthoracic echocardiogram in 6 months C. Monitoring via transesophageal echocardiogram in 6 months D. Treadmill exercise stress test E. Automatic internal cardiac defibrillator placement

The Correct Answer is: A In symptomatic patients demonstrating significant mitral valve stenosis, mitral valve replacement after cardiac catheterization,is recommended, to evaluate for associated valvular disease and coronary artery disease. This allows planning for possible concomitant coronary artery bypass surgery, with mitral valve replacement and or other valve replacement, if necessary. As the patient is symptomatic and demonstrates both severe mitral stenosis and mitral regurgitation, choice A is the most appropriate next step in management. Choices B and C are thus inappropriate, as the patient is already symptomatic. If the patient were not symptomatic, choice B would be a more viable choice compared to choice C, because it is less invasive than transesophageal echocardiogram. Choice D would be inappropriate, as strenuous physical activity should be avoided in patients with severe mitral valve stenosis. Choice E is appropriate therapy for patients at risk for ventricular tachycardia/fibrillation.

A 28-year-old male is concerned because he has a friend who was diagnosed with chronic hepatitis C and wants to know if he is at risk for this disease as well. Which of the following answers elicited in your history would make you the most suspicious? A. previous intravenous drug use B. blood transfusion last year C. monogamous relationship D. recent negative HIV test E. previous history of hepatitis

The Correct Answer is: A Intravenous drug abuse accounts for over 50% of hepatitis C cases. Recent blood transfusions are unlikely to cause hepatitis C. Hepatitis C is more likely to be contracted in patients with multiple sexual partners, although the incidence of transmission via sexual intercourse is considered low. Hepatitis A is transmitted via the fecal-oral route and does not increase the chances of hepatitis C.

A 23-year-old male presents to the clinic complaining of left anterior neck pain that developed over the past week following recovery from an acute upper respiratory infection. On physical exam a tender mass is felt anterior to the left sternocleidomastoid muscle from the mandible inferiorly to the level of the cricoid cartilage. Which of the following is the most likely diagnosis? A. Branchial cleft cyst B. Dermoid cyst C. Peritonsillar abcess D. Salivary gland tumor E. Thryoglossal duct cyst

The Correct Answer is: A The development of a neck mass in a young adult following URI is consistent with branchial cleft cyst (A) and thyroglossal duct cyst (E). The location of this mass away from the midline and anterior to the SCM is most consistent with branchial cleft cyst (A). The location of the mass and history are inconsistent with dermoid cysts (B), which are typically midline, peritonsillar abcesses (C), which would be located in the retropharyngeal space, and salivary gland tumors (D), which would be located in the parotid, submandibular, or submental salivary regions.

A 22-year-old male hurts his right knee while playing football on artificial turf. He states that he planted his foot and went to turn, but his leg didn't turn with his body. He instantly felt a popping sensation in the knee. A few hours later he develops an effusion. Based on the history of the injury, which knee structure was likely injured? A. Anterior cruciate ligament (ACL) B. Lateral collateral ligament (LCL) C. Medial collateral ligament (MCL) D. Posterior cruciate ligament (PCL) E. Quadriceps tendon

The Correct Answer is: A Non-impact rotational or hyperextension forces are the most common mechanisms for sustaining a tear of the ACL. One third of patients report hearing an audible popping sound as their ACL tear occurred. Because the ACL is a vascular structure, when it tears a rapid bloody effusion (hemarthrosis) usually develops which effects mobility of the joint. Lateral collateral ligaments are the least likely to be injured as the type of force necessary to cause injury would be a varus stress which is unlikely to occur in typical circumstances. Medial collateral ligament injuries are fairly common and produced by a valgus force that stresses the ligament. This can occur in many sporting events including those in which another competitor might fall on or dive into the lateral aspect of the knee. Trauma to a knee can result in tears of both the ACL and MCL in certain situations. A tear of the quadriceps tendon usually occurs when a person falls on a knee that is partially flexed. As the quadriceps muscle contract to prevent excessive flexion, the force and momentum of the fall may overwhelm the knee extension mechanism and cause the rupture. No such mechanism occurred in our scenario. Posterior cruciate injuries occur when the tibia is driven posterior in relation to the femur as may happen when a car dashboard is driven into the tibias during a major front impact collision. A powerful hyperextension force can result in both ACL and PCL tears (usually in that order). PCL tears are much more uncommon than ACL tears and don't generally occur with basic rotational forces as described in our patient scenario.

You are evaluating a patient with multiple articular complaints that are consistent with osteoarthritis. The patient is a 60-year-old female who complains of pain to her joints which worsen as the day progresses. On examination you notice that the patient has Heberdens nodes. Based on the history and physical exam findings, what structures become progressively damaged and eroded over time? A. Articular cartilage B. Fascia C. Ligaments D. Muscles E. Tendons

The Correct Answer is: A Osteoarthritis is a progressive, irreversible disease that leads to loss of articular cartilage in the joints. Osteoarthritis can affect the weight bearing joints in the lower extremities and spine, but can also be commonly found in the hands and other joints. The occurrence and severity of the condition can be affected by history of past trauma to a joint, long-term wear and tear on a joint, as well as obesity, genetics, and progressive aging. The condition can produce pain, swelling, stiffness, decreased range of motion, joint deformity, crepitus and decreased ability to do tasks of daily living. Osteoarthritis does tend to lead to direct damage to tissues like fascia (which is a fibrous connective tissue that surrounds muscles), muscles or tendons (tendons connect muscles to bones). As osteophytes form and joints become distorted, this could have a secondary impact on ligaments that are responsible for providing joint stability, but ligaments are not primarily affected by osteoarthritis.

What age group is most at risk to develop osteoid osteoma? A. Adolescents B. Elderly C. Middle-aged D. Neonates E. Toddlers

The Correct Answer is: A Osteoid osteoma is a benign bone forming tumor that usually develops during a patient's second decade of life. This type of tumor is much more common in boys than girls and typically affects the lower extremities (femur and tibia primarily) and spine more than other areas of the body. Patients typically present with gradually progressive bone pain that is worse at night and does not correlate with activity level. The tumor produces high levels of prostaglandins, so symptoms usually improve in 20-25 minutes if the patient takes a medication like ibuprofen, ASA or other NSAIDS that are prostaglandin inhibitors. A lack of improvement in symptoms with these medications should lead health care providers to consider a different diagnosis. The pain of this condition may cause those afflicted in a leg to limp and have swelling, muscle atrophy or contractures and exquisite point tenderness. The condition usually resolves on its own over time, but symptomatic patients may require surgical resection or radioablation of the tumor.

A 32-year-old man presents to the urgent care center with a concern of scrotal tenderness that began 3 days ago and has now worsened. Physical examination reveals a temperature of 100.7°F, positive tenderness in the posterolateral aspect of the right testis with swelling, spermatic cord tenderness with palpation, and no transillumination. What is this patient's most likely diagnosis? A. epididymitis B. orchitis C. epididymo-orchitis D. testicular torsion

The Correct Answer is: A Pain and swelling are prominent features of epididymitis; fever and abdominal pain may also be present. Epididymitis is caused by an ascending infection that without treatment will continue to the testicles, causing a significant swelling that will make it difficult for the clinician to distinguish between the epididymis and the testicles (epididymo-orchitis). Orchitis alone is most commonly viral (mumps) and observed in prepubertal boys. In men younger than 30 years, epididymitis can be confused with torsion.

A 43 year-old woman presents to the clinic due to the presence of a bulls eye rash on her right upper back. She is prescribed tetracycline. What is the mechanism of tetracycline? A. Binds to the 30S ribosomal subunit to inhibit protein synthesis B. Binds to the 50S ribosomal subunit to inhibit protein synthesis C. Inhibits dihydropteroate synthase and folate production D. Inhibits DNA replication by binding to DNA gyrase and topoisomerase IV E. Inhibits the transpeptidation reaction

The Correct Answer is: A Tetracycline binds to the 30S ribosomal subunit to inhibit protein synthesis (A). Macrolide antibiotics bind to the 50S ribosomal subunit to inhibit protein synthesis (B). Sulfamethoxazole inhibits dihydropteroate synthase and folate production (C). Ciprofloxacin inhibits DNA replication by binding to DNA gyrase and topoisomerase IV (D). Beta lactam antibiotics inhibit the transpeptidation reaction leading to cell wall destruction (E).

: Shoulder dislocation commonly involves injury to which nerve? A. Axillary B. Median C. Peroneal D. Radial E. Ulnar

The Correct Answer is: A The axillary nerve is in close proximity to the glenohumeral joint, thus making it vulnerable to injury during a shoulder dislocation. The median, radial, ulnar nerves are more distal in the upper extremity and and the peroneal nerve is in the leg.

Which of the following diseases is associated with the development of nasal polyps? A. Amyloidosis B. Allergic rhinitis C. Chronic sinusitis D. Chronic obstructive pulmonary disease E. Wegener's granulomatosis

The Correct Answer is: B Nasal polyps are most commonly idiopathic although they may develop secondary to allergic rhinitis (B), or cystic fibrosis. Chronic sinusitis (C) may result from obstruction of the sinus drainage secondary to a polyp. Amyloidosis (A) and Wegener's granulomatosis (E) lead to the development of lesions with histology and appearance different than a benign nasal polyp.

A 26-year-old woman who is nursing presents to clinic complaining of 2 to 3 days of increasing pain and redness in her left breast. She continues to feed her infant and has good milk supply. She is 4 weeks postpartum. She complains of some general fatigue, but no headaches, body aches, or fever. On physical exam, results are as follows: P 80, T 99.1°F, and BP 120/70, CV RRR, and lungs CTA. Her left breast has a 5-cm area of induration, in the upper outer quadrant a 2-cm mass is noted, which is tender to the touch. In addition, milk expressed is non-purulent. What is the most likely cause of the infection? A. Staphylococcus aureus B. Streptococcus viridians C. Blastomycosis D. Sporotrichosis E. Streptococcus pyogenes

The Correct Answer is: A The discrete nodules indicate a breast abscess, which can commonly occur in nursing women. Fungal infections can occur, but are rare, and Streptococcal infections are not as likely as staph infections.

Upon testing a patient for function of the hip extensors, which muscle is considered the primary muscle responsible for most extension? A. Gluteus maximus B. Pectineus C. Semimembranosus D. Semitendinosus E. Vastus lateralis

The Correct Answer is: A The gluteus maximus is a large muscle that is partially responsible for giving shape to the buttocks. It is the dominant muscle responsible for hip extension. It is easily palpable with a patient in the prone position with buttocks squeezed together or with the hip extended and the knee flexed. The pectineus muscle is considered a secondary hip adductor. The Semimembranosus and Semitendinosus are two of the three hamstring muscles. They are primary movers in knee flexion, but only secondary contributors to hip extension. Vastus lateralis is one of the four quadriceps muscles and plays a role in knee extension, but not hip extension.

A 58 year-old male presents with a history of dyspnea on exertion and chronic cough worse with arising in the mornings. He has a 60-pack year history of cigarette use. On examination there is increased AP diameter and decreased breath sounds with a prolonged expiratory phase. Pulse oximetry reveals an oxygen saturation of 93% on room air. Current medications include varenicline and ipratropium. Which of the following is the most appropriate intervention at this time? A. Influenza and pneumococcal vaccine B. Montelukast C. Oxygen therapy D. Prednisone E. Prophylactic antibiotic therapy

The Correct Answer is: A The patient has chronic obstructive pulmonary disease being treated with bronchodilators (ipratropium) and he is undergoing tobacco cessation therapy (varenicline). Additional health maintenance requirements include administration of influenza and pneumococcal vaccine (A). Antibiotics (E) are beneficial during acute COPD exacerbations. Oxygen therapy (C) is not required as evidenced by his pulse oximetry of 93%. Prednisone (D) is used in later stage disease that fails to respond to additional inhaled medications (long-acting beta agonists, corticosteroids). Montelukast (B) is used in the management of asthma.

A 73-year-old, male with a 30 pack-year smoking history presents to the clinic with complaints of headache and dizziness. Labs reveal a hemoglobin of 21.3 g/dl, hematocrit of 63%, and platelet count of 498,000. He tests positive for the JAK2 mutation. Which of the following should be recommended to this patient to prevent secondary complications from his diagnosis? A. Aspirin 81 mg daily B. Ferrous Sulfate 325 mg twice daily C. Eat a diet rich in vitamin B12 D. Eat a diet rich in folate E. Maintain stable consumption of foods that contain vitamin K

The Correct Answer is: A The patient most likely has polycythemia vera and is at risk for thrombotic events that can be reduced through the use of daily aspirin. In addition, he should be counseled on smoking cessation if he is still currently smoking. The patient should not take iron supplements (B) as they compete with phlebotomy therapy that seeks to reduce iron stores. Patient's with PCV similarly do not require vitamin B 12 (C) or folate (D) supplementation, and are not impacted by fluctuations in vitamin K consumption (E) unless they are being treated with warfarin for a current/recent thrombotic event.

23-year-old man presents to the outpatient clinic for follow-up from a recent urgent care visit. He complains of sore throat, fever, fatigue, myalgias, and a rash that started 5 days ago, and have worsened since he was seen in the urgent care 3 days ago. The patient appears non-toxic with a temperature of 39.4 degrees Celsius. Physical exam reveals pharyngeal and tonsillar erythema without exudates, generalized lymphadenopathy, a morbilliform rash on his trunk, and no hepatosplenomegaly. A rapid strep screen and Monospot performed at the local urgent care were reportedly negative. Which of the following prevention strategies should be recommended to this patient? A- Abstain from sexual activity B- Avoid aspirin C-avoid contact sports and rest D- bedrest and increased fluids E- Take the full course of antibiotics

The Correct Answer is: A The patient presentation is consistent with acute retroviral syndrome. The patient is highly contagious and should be counseled on strategies to prevent transmission of HIV to others (A). Aspirin use in viral syndromes (B) is associated with Reye's syndrome, but most often occurs in children with influenza or varicella. Avoiding contact sports (C) is appropriate patient education for a patient with infectious mononucleosis, and patients with group A strep pharyngitis should be instructed to take the full course of their antibiotics (E).

A 36-year-old man presents to the emergency department with a tight bandage around his chest to help reduce pain from a chest wall injury on his right side that occurred during mixed martial arts sparing. Physical exam reveals dullness to percussion, dry crackles and diminished breath sounds over the right lower lobe. Chest x-ray shows elevation of the right hemi-diaphragm. What is the most likely diagnosis? A. Atelectasis B. Bronchiectasis C. Pleural Effusion D. Pneumothorax E. Pulmonary edema

The Correct Answer is: A The patient's injury places him at risk of atelectasis, pneumothorax, or other traumatic injuries. The physical exam and chest x-ray findings are classic for atelectasis (A). Pleural effusion (C) would present with fluid in the costophrenic angle on chest x-ray. Pneumothroax (D) would typically present with findings in the upper lung fields including hyperresonance to percussion. Pulmonary edema (E) would present with increased vascular markings and evidence of fluid within the alveolar space on chest x-ray.

A 14-month-old African American boy is diagnosed with sickle cell anemia. Neither parent has the disease. If you are counseling the parents of this child on the likelihood of another one of their children having sickle cell disease, what percentage would you tell them? A. 0% B. 25% C. 50% D. 75% E. 100%

The Correct Answer is: B Sickle cell anemia is an autosomal recessive genetic disorder. The presence of two defective genes is required for sickle cell anemia to occur. If each parent carries one sickle hemoglobin gene and one normal gene, each child has a 25% chance of inheriting two defective genes and developing sickle cell, a 25% chance of inheriting two normal genes and not having the disease, and a 50% chance of being an unaffected carrier.

A 49-year-old male presents to the clinic with symptoms of nausea, occasional vomiting, vague epigastric pain, fatigue, and weight loss of 35 lbs. in the past few months. On exam, you find a palpable abdominal mass. What is his most likely diagnosis? A. Gastric adenocarcinoma B. Crohn's disease C. Peptic ulcer disease D. Nonfamilial adenmatous polyposis E. Anorexia nervosa

The Correct Answer is: A The patient's signs and symptoms in this question correlate most closely with gastric adenocarcinoma. With Crohn's disease, the patient may have weight loss and their signs and symptoms often include intermittent bouts of low-grade fever, diarrhea, right lower quadrant abdominal pain, and right lower quadrant tenderness on physical exam. Peptic ulcer disease typically presents with dyspepsia, but you would not see the associated weight loss or abdominal mass. Nonfamilial adenomatous polyposis consists of polyps in the colon, which are most typically asymptomatic; however, the most common symptom would be intermittent hematochezia if the polyp was large and ulcerated. Patients with anorexia nervosa have a disturbance of body image and weight loss, but they would not have the other symptoms present (the finding of an abdominal mass).

A 43-year-old female presents to the outpatient clinic complaining of itching and irritation of her right eye. She denies decreased vision or photophobia. On physical exam the patient's eye has the following appearance: Which of the following is the most likely diagnosis for this patient? A. Anterior blepharitis B. Dacrocystitis C. Posterior belpharitis D. Preseptal cellulitis E. Psoriasis

The Correct Answer is: A The patient's symptoms of itching and irritation of the lid margin with an inflamed eyelid and eyelash scaling is consistent with anterior blepharitis (A). Dacrocystitis (B) is an infection of the lacrimal sac and would be isolated to that location. Posterior blepharitis (C) affects the inner lid margins and meibomian glands. The symptoms and physical exam findings aren't consistent with preseptal cellulitis (D) or psoriasis (E); however, seborrhea is commonly associated with anterior blepharitis.

A 43-year-old female presents to the outpatient clinic complaining of itching and irritation of her right eye. She denies decreased vision or photophobia. On physical exam the patient's eye has the following appearance: Which of the following is the most appropriate management for this patient's condition? A. Daily lid cleansing and application of bacitracin ophthalmic ointment 500 units/g B. Doxycycline 100 mg by mouth once daily C. Incision and drainage D. Referral to an ophthalmologist E. Warm compresses every 3 hours until resolution

The Correct Answer is: A The patient's symptoms of itching and irritation of the lid margin with an inflamed eyelid and eyelash scaling is consistent with mild anterior blepharitis, which is initially treated with cleansing and the potential addition of a topical antistaphlococcal antibiotic (A). Answers (C) and (E) are appropriate treatments for a hordeolum. Doxycycline once daily (B) can be used as a long-term treatment for posterior blepharitis. The condition doesn't warrant referral (D).

A 28-year-old male presents to the outpatient clinic with complaints of a painful, red right eye, with profuse drainage that started 2 days ago. He has increased tearing and photosensitivity. Physical exam reveals the following appearance of the upper tarsal conjunctiva. Which of the following is the most likely causative agent for this patient's conjunctivitis? A. Chlamydia trachomatis B. Herpes simplex virus C. Neisseria gonorrhea D. Pseudomonas E. Streptococcus pneumoniae

The Correct Answer is: A The presence of acute follicular conjunctivitis is most consistent with inclusion conjunctivitis resulting from chlamydial infection (A). Follicles are most often seen in viral conjunctivitis and some forms of parasitic conjunctivitis, but not typically seen in bacterial conjunctivitis (C, D, and E).

A 23-year-old female presents to the outpatient clinic with complaints of a painful, red right eye, with profuse drainage that started 2 days ago. She has increased tearing and photosensitivity. Physical exam reveals the following appearance of the upper tarsal conjunctiva. Which of the following is the most appropriate treatment for this patient? A. Azithromycin 1 gram by mouth B. Ceftriaxone 1 gram IM C. Olopatadine 0.1% ophthalmic solution twice daily D. Sulfacetamide 10% ophthalmic solution three times a day for 5 days E. Valacyclovir 500 mg by mouth twice daily for 10 days

The Correct Answer is: A The presence of acute follicular conjunctivitis is most consistent with inclusion conjunctivitis resulting from chlamydial infection. Follicles are most often seen in viral conjunctivitis and some forms of parasitic conjunctivitis, but not typically seen in bacterial conjunctivitis. The first-line treatment for inclusion conjunctivitis is azithromycin 1 gram orally (A).

A 58-year-old female presents to the outpatient clinic complaining of 1 week of rhinorrhea, nonproductive cough, and hoarseness. On physical exam she is noted to have erythematous nasal mucosa and decreased phonation without significant nasal discharge, sinus tenderness, pharyngeal erythema, or lymphadenopathy. Which of the following is the most likely diagnosis? A. Laryngitis B. Laryngeal cancer C. Vocal cord hemorrhage D. Vocal cord paralysis E. Vocal cord polyp

The Correct Answer is: A The presence of acute hoarseness associated with an upper respiratory infection is consistent with laryngitis (A).

A 56-year-old male patient is diagnosed with prostatitis. Which of the following is the least appropriate antibiotic to prescribe in the family practice setting? A. ceftriaxone B. doxycycline C. levofloxacin D. trimethoprim-sulfamethoxazole (TMP-SMX) E. All are appropriate antibiotics for this patient in this setting.

The Correct Answer is: A The response to antibiotics in acute bacterial prostatis is usually prompt, perhaps because drugs penetrate readily into the acutely inflamed prostate Antibiotic selection should be guided by results of urine cultures and susceptibility results. Appropriate empiric antibiotics include a fluroquinolone (i.e.levofloxacin 500 mg once daily) or TMP/SMX (one double-strength tablet every 12 hours). Patients who are too ill for oral therapy or are septic on presentation should be hospitalized for initial parenteral treatment (intravenous quinolones with or without an aminoglycoside). Ceftriaxone would not be recommended as first-line.

An 8-year-old male presents to his primary care provider with the onset of a new rash, consisting of small, oval, discrete scaling plaques on his trunk, and a large red plaque with overlying thin, silvery scales in the gluteal cleft. Which of the following is a potentially important historical finding in this patient? A. a recent history of group A strep infection B. a family history of atopy C. exposure to nickel in clothing D. a personal history of allergies

The Correct Answer is: A This is a classic guttate psoriasis. An acute strep infection is a known precipitating factor of guttate psoriasis. All patients need to be checked and treated for a strep infection. Atopy has no correlation with guttate psoriasis. It is not caused by contact with an allergen or irritant. It is also not caused by an allergic reaction.

In your family practice, you perform an ankle brachial index (ABI) on your 66-year-old diabetic who smokes with the results being 0.71 on the left and 0.68 on the right. Which of the following is the most appropriate next step? A. Begin the patient on aspirin 81 mg and clopidogrel 75 mg daily. B. Begin the patient on prasugrel 60 mg loading dose followed by 10 mg daily and refer to cardiology. C. Begin the patient on a low-molecular weight heparinoid (LMWH) and refer to a vascular surgeon for further evaluation. D. Begin the patient on warfarin 5 mg daily and titrate to an INR of 2.0 to 3.0.

The Correct Answer is: A This patient has peripheral arterial disease (PAD). This can be treated with antiplatelet agents, including aspirin and/or clopidogrel. Warfarin is an anticoagulant and is not FDA-approved for use in PAD. Your other consideration is referral to a cardiologist and/or vascular surgeon for further evaluation, depending upon the degree of symptoms. Patients with PAD have approximately one in six chance of have significant atherosclerosis in at least one other vascular bed, including carotids and coronary, and with a diabetic history, aggressive treatment of all co-morbidities could be life-saving. Effient, LMWH, and warfarin are not FDA-approved for treatment of PAD.

A 51-year-old male patient presents to your family practice office complaining of genital discomfort with dysuria. His digital rectal exam reveals an enlarged, tender prostate. His prostate-specific antigen (PSA) returns elevated with a value of 11.1 mg/mL, which you fractionate, and this reveals approximately 75% free PSA. His urinalysis reveals moderate white cells and trace blood. What would be your next step in treating this patient? A. Begin him on 6 weeks of doxycycline to treat his prostatitis and when resolved, repeat his PSA level. B. Immediately refer him to a urologist for prostate biopsy to rule out prostate cancer. C. Immediately refer him to a urologist for cystoscopy to rule out bladder cancer and perform a computed tomography (CT) scan of the abdomen and pelvis in the interim. D. Order a stat testicular sonogram to rule out torsion. E. Order a CT scan of the abdomen and pelvis.

The Correct Answer is: A This patient has signs and symptoms consistent with prostatitis. Additionally, while his PSA is elevated, this is common in prostatits as well as prostate cancer, and his free PSA is of a percentage that prostate cancer is unlikely. However, it would be prudent to recheck his PSA after treatment and resolution of his symptoms to confirm that an underlying cancer is not smoldering.

A 64-year-old African American female presents to the clinic for evaluation of her hypertension, which she has had for several years. In the past she had been taking hydrochlorothiazide and lisinopril, with little effect on her blood pressure management. At today's visit, she has no complaints and feels well. Her vitals show T m 96.6, P 85, R 18, BP 191/99. She has no jugular venous distention seen on the neck exam, her lungs are clear, and cardiac exam has a regular rate and rhythm without murmur or gallop. Her abdomen is soft, non-tender, and a bruit is appreciated at the mid-abdomen just a few centimeters below the epigastric region. There is no fullness or enlargement of the abdominal aorta on palpation. Based on the history and clinical findings, what is the most likely diagnosis? A. Renal artery stenosis B. Abdominal aortic aneurysm C. Mitral stenosis radiating murmur D. Tricuspid stenosis E. Coarctation of the aorta

The Correct Answer is: A This patient is exhibiting hypertension that appears to be quite high, along with evidence of a bruit on abdominal exam. Because there are no appreciable murmurs on cardiac exam, it would be less likely that a cardiac valvular murmur (C) is the reason. The abdominal aorta is of normal size, and most coarctation murmurs (E) are better appreciated in the back of the chest than on abdominal exam. Because the patient has uncontrolled hypertension despite being on two antihypertensive medications, the most likely reason for the bruit is renal artery stenosis (A). An abdominal aortic aneurysm (B) is less likely given the normal size of the aorta on exam, and tricuspid stenosis (D) would not present with these symptoms.

A 33-year-old male is brought by rescue to the emergency department after he had a collision on the side of the road on his motorcycle. He is complaining of pain and swelling of the right ankle, with decreased range of motion and pain to the extremity. You decide to order a radiograph of his right ankle (with results shown below). Based on the radiography provided, what is the nature of this injury? A. Bimalleolar ankle fracture involving the distal tibia and fibula B. Distal fibula fracture only C. Distal tibia fracture only D. Lisfranc fracture - dislocation E. Talus fracture

The Correct Answer is: A This radiograph clearly shows fractures of both the distal fibula and tibia (see arrows below). Bimalleolar ankle fractures can also present with a clear fibular fracture and signs of deltoid ligament disruption on the medial side (a palpable gap on the medial side, deltoid ligament tenderness or laxity with an eversion-type stress). This type of injury is considered unstable and generally requires surgical management. Choices "B" and "C" were incorrect since there is clear evidence of fractures on both the lateral and medial aspect of the joint pictured. A Lisfranc fracture - dislocation would seem unlikely as the tarsometarsal joints appear intact, but this is not an ideal view to rule out that injury conclusively. This x-ray does not have any evidence of a talus fracture, but additional views would be needed to be sure no such fracture is present.

What diagnosis should be given to a patient who has nonbizarre delusions for at least a month and no other symptoms? A. schizoaffective disorder B. delusional disorder C. brief psychotic disorder D. schizophreniform disorder

The Correct Answer is: B A delusional disorder presents with nonbizarre delusions for at least a month. The disorder does not present with any other symptoms related to schizophrenia or a mood disorder. A brief psychotic disorder has symptoms that last for 1 day to 1 month. Schizophreniform has symptoms that last at least a month, but no longer than 6 months. In schizoaffective disorders, depression or mania develop along with schizophrenic symptoms.

A 29-year-old female presents with a "bump" on her wrist for the past week. She denies any pain or history of trauma. Her past medical history is unremarkable. On physical examination you discover a 2.0 cm nontender, firm cystic lesion on the dorsum of the wrist. Considering your suspected diagnosis what is the best treatment option? A. IV antibiotics B. Aspiration and injection of a corticosteroid C. Physical therapy D. Hold pressure on the cyst until it ruptures

The Correct Answer is: B A ganglion cyst is a cystic collection of synovial fluid within a joint or tendon sheath. Treatment options include cyst aspiration and corticosteroid injection (B), or surgical excision. However, none are typically performed in the ED. Surgery is generally effective but ganglion cysts may reoccur. Referral to a hand surgeon is indicated for persistent or recurrent pain or cosmetic deformity. IV antibiotics (A) would not help since ganglion cysts are not known to be infectious. Physical therapy (C) would aggravate the cyst and would not reduce the size typically. Rupturing the cyst (D) would temporarily reduce the size but recurrence is high.

What type of fracture is not related to an acute bony trauma? A. Greenstick B. Stress C. Oblique D. Comminuted E. Spiral

The Correct Answer is: B A stress or fatigue fracture is caused by small, repetitive forces that usually involve the metatarsal shafts, the distal tibia, and the femoral neck (though many other bones may be affected). These fractures may not be seen on initial radiographs. A greenstick fracture is an incomplete traumatic fracture with angular deformity seen in children. An oblique fracture is a traumatic fracture with an angulated fracture line. A comminuted fracture is a traumatic fracture in which there are more than two fracture segments. A spiral fracture is a traumatic fracture that has a multiplanar and complex fracture line usually caused by an excessive rotational force on a bone.

A parent brings her child into the office with the concern of the way he stands. To the parent the child looks like they are knocking knees. If the child is indeed presenting this way, what would be the best description of this orthopedic abnormality? A. Angulation of an extremity at a joint with the more distal part angled anteriorly. B. Angulation of an extremity at a joint with the more distal part angled away from the midline. C. Angulation of an extremity at a joint with the more distal part angled toward the midline. D. Angulation of an extremity at a joint with the more distal part angled posteriorly. E. Angulation of an extremity at a joint with the more proximal part angled away from the midline.

The Correct Answer is: B A valgus deformity involves angulation of an extremity at a joint with the more distal part angled away from the midline. When looking at the anterior aspect of a patient in the anatomic position, a valgus deformity of the left knee or left elbow would look like the letter L.

A 64-year-old female presents to your family practice office with a complaint of shortness of breath, dizziness, and fatigue over the past seven days. She states that she has experienced a similar episode twice in the past seven months, but it has resolved spontaneously in the past. However, this time she states it is different and is concerned about her condition. Her vital signs are stable and ECG is below. What is your diagnosis and which of the following is the appropriate initial treatment? A. paroxysmal supraventricular tachycardia; start her on metoprolol ER 50 mg once daily and refer her to cardiology B. persistent atrial fibrillation with a controlled rate; start her on warfarin 5.0 mg and check her INR in 5 days C. sinus arrhythmia; reassure her that this is perfectly normal, associated with respirations, and that nothing further needs to be done D. tell her that her ECG shows some irregularity; order a 24-hour holter monitor E. she is having an anterior-inferior wall myocardial infarction; give her a full-dose aspirin and call 911

The Correct Answer is: B Atrial fibrillation is the most common chronic arrhythmia, with an incidence and prevalence that rises with age, so that it affects approximately 10% of individuals over 80 years old. It occurs in rheumatic and other forms of valvular heart disease, dilated cardiomyopathy, atrial septal defect (ASD), hypertension, and coronary heart disease. It also occurs in patients with no apparent cardiac disease; it may be the initial presenting sign in thyrotoxicosis, and this condition should be excluded with the initial episode. The heart rate may range from quite slow to extremely rapid, but is uniformly irregular unless there is a underlying complete heart block and a permanent ventricular pacemaker is in place. The surface ECG typically demonstrates erratic, disorganized atrial activity between discrete QRS complexes occurring in an irregular pattern. The atrial activity may be very fine and difficult to detect on the ECG, or quite course and often mistaken for atrial flutter. Atrial fibrillation often appears paroxysmally before becoming the established rhythm.

A 23 year-old man presents to the clinic complaining of fever, productive cough and fatigue for 3 days. Physical exam reveals decreased breath sounds in the left lower lobe. He is prescribed azithromycin. Which of the following statements best describes the mechanism of action of azithromycin? A. Binds to the 30S ribosomal subunit to inhibit protein synthesis B. Binds to the 50S ribosomal subunit to inhibit protein synthesis C. Inhibits dihydropteroate synthase and folate production D. Inhibits DNA replication by binding to DNA gyrase and topoisomerase IV E. Inhibits the transpeptidation reaction

The Correct Answer is: B Azithromycin (B) is a macrolide antibiotic that binds to the 50S ribosomal subunit to inhibit protein synthesis. Beta lactam antibiotics (A) inhibit the transpeptidation reaction leading to cell wall destruction, Ciprofloxacin (C) inhibits DNA replication by binding to DNA gyrase and topoisomerase IV, tetracycline (E) binds to the 30S ribosomal subunit to inhibit protein synthesis, and sulfamethoxazole (D) inhibits dihydropteroate synthase and folate production.

A 4-month-old, full-term infant female presents to the clinic for a well-child visit. The mother states the baby is nursing well and her growth chart progression is within normal limits. Which of the following supplements is most important for this infant to receive? A. Fluoride B. Iron C. Omega-3 fatty acids D. Protein E. Vitamin K

The Correct Answer is: B Breastfed infants require additional dietary iron, either through supplements or through consumption of iron rich foods (i.e., fortified cereal). Without supplementation, the infant will deplete her iron stores by about 5-6 months of age and develop iron deficiency anemia. Fluoride (B) supplementation should begin at 6 months of age. Omega-3 fatty acids (C) and protein supplementation (D) are less important in this full-term infant, and vitamin K (E) supplementation is used in the presence of specific diseases (i.e., chronic liver disease).

A 35-year-old male presents with pain and decreased range of motion after sustaining a fall in which the patient tried to grab onto a bar which pulled his entire arm in the process. Given this clinical scenario, at what cervical motor neuron level would the biceps reflex be testing? A. C4 B. C5 C. C6 D. C7 E. C8

The Correct Answer is: B C5 is the primary motor neuron being tested in a biceps reflex. C6 contributes to the brachioradialis reflex primarily, but does have a small role in the biceps reflex. The C7 motor neuron is primarily involved in the triceps reflex. C4 and C8 do not contribute to any primary reflexes.

Which of the following is a risk factor for cholesteatoma formation? A- Acute otitis media B- Chronic tympanic membrane retraction C- Diabetes mellitus D- Hyperlipidemia E- Smoking

The Correct Answer is: B Cholesteatomas result from negative pressure in the middle ear chronically retracting the pars flaccida (B). Acute otitis media (A) may lead to short-term TM retraction or perforation, but without perforation or the placement of PE tubes it is unlikely to lead to cholesteatoma formation. Diabetes (C), hyperlipidemia (D), and smoking (E) are not associated with this localized pathology.

A 52-year-old female with a past medical history of dyslipidemia, GERD, hypothyroidism, and osteoarthritis presents with a gradual progression of fatigue and pallor over the last few months. Initial CBC results show a hemoglobin of 10.7 mg/dL, hematocrit of 33%, an MCV of 117 fL, and a reticulocyte count of 0.0%. Which of the following medications should be considered as a potential cause of her anemia? A- Celecoxib B- Cholestyramine C- Levothyroxine D- Omeprazole E- Ranitidine

The Correct Answer is: B Cholestyramine is associated with folate malabsorption and may rarely be a medication cause for macrocytic anemia. Celecoxib (A), levothyroxine (C), omeprazole (D), and ranitidine (E) are not strongly associated with vitamin B 12 or folate deficiency leading to macrocytic anemia

A 32-year-old female professional golfer presents with a 3-week history of pain along her thumb and down her wrist. She denies any trauma and states that it is aggravated with any movement of her wrist and thumb. Her physical examination and x-rays are unremarkable. You perform the following test: This test requires her to cup her thumb in a closed fist and ulnar deviate, which reproduces her pain. What is the most likely diagnosis based on this exam finding? A. Colle's fracture B. De Quervain tenosynovitis C. Carpal tunnel syndrome D. Ganglion cyst E. Gout

The Correct Answer is: B De Quervain tenosynovitis (B) commonly occurs in patients who have experienced excessive use of the thumb or wrist. Often, no plausible cause can be found. This is a tenosynovitis of the tendons that lie in the groove of the radial styloid. The patient usually presents with pain along the radial aspect of the wrist that may radiate to the thumb or extend into the forearm. The diagnosis of De Quervain tenosynovitis is supported by a history of pain in this location along with a painful range of motion of the thumb and local tenderness over the distal portion of the radial styloid. Further confirmation of the diagnosis may be provided by a positive Finkelstein test, in which the patient grasps the thumb in the palm of the hand and the examiner ulnar deviates the thumb and hand. This stretches the tendons over the radial styloid and produces sharp pain along the involved tendons. A ganglion cyst (D) and carpal tunnel syndrome (C) would not appear on physical examination in this presentation. Gout (E) would be painful and involve more of the joint. A Colle's (A) fracture would show on her x-ray.

A 43 year-old woman undergoes pulmonary function tests (PFTS) to evaluate progressive dyspnea and cough. The diffusion capacity is noted to be significant reduced. What is the most likely type of lung disease this patient is experiencing? A. Congenital B. Interstitial C. Obstructive D. Restrictive E. Fixed extra-thoracic obstruction

The Correct Answer is: B Diffusion capacity is reduced in conditions that effect alveolar gas exchange such as, interstitial lung disease (B), atelectasis, pneumonia pulmonary vasculature disease, and late stage emphysema (due to destruction of alveoli, not obstructive lung disease).

A 67 year-old with chronic kidney disease presents to the clinic with a hemoglobin of 8.6 mg/dl and hematocrit of 24%. Which of the following agents, if administered, can further increase this patient's risk of thrombotic vascular events? A. Cyanocobalamin B. Erythropoietin C. Ferrous sulfate D. Folic acid E. Niacin

The Correct Answer is: B Erythropoietin (B) increases red blood cell mass and toxicity may lead to increased blood viscosity and potential thrombotic events. Cyanocobalamin (A), ferrous sulfate (C), folic acid (D), and niacin (E) don't increase thrombotic risk.

A 22-year-old non-obese female presents to the clinic with sign and symptoms of deep vein thrombosis (DVT). She denies recent trauma, prolonged immobilization, smoking, use of oral contraceptives, or changes in her health. Her last menstrual period was one week ago. Which of the following is the most likely diagnosis that contributed to the development of her DVT? A. Antithrombin deficiency B. Factor V Leiden C. Protein C deficiency D. Protein S deficiency E. Hyperhomocysteinemia

The Correct Answer is: B Factor V Leiden mutation is one of the most common genetically inherited prothrombotic states. Deficiencies of antithrombin (A), protein C (C) and protein S (D) when homozygous in nature typically present as fetal death or severe thrombosis at birth. When heterozygous, they carry a much lesser risk of developing thrombosis as compared to Factor V Leidin. Hyperhomocysteinemia (E) also carries a much lesser risk of developing thrombosis.

A 30-year-old female presents with a year history of chronic aching pain and stiffness throughout her entire body. She denies any trauma, new activities, or change in symptoms throughout the day. Her past medical history is significant for trouble sleeping at night and irritable bowel symptoms. On physical examination, the only finding is pain produced on palpation of the trapezius, medial knee, and lateral epicondyle of the elbow bilaterally. Her vitals are normal. All laboratory work including a thyroid panel, complete metabolic panel, erythrocyte sedimentation rate, and rheumatoid factor are within normal limits. What is the most likely diagnosis based on these findings? A. Polymyositis B. Fibromyalgia C. Polymyalgia rheumatica D. Rheumatoid arthritis E. Systemic lupus erythematosus

The Correct Answer is: B Fibromyalgia (B) is a diagnosis of exclusion most frequent in women aged 20-50 years. The patient with fibromyalgia complains of chronic aching pain and stiffness, frequently involving the entire body but with prominence of pain around the neck, shoulders, low back, and hips. Fatigue, sleep disorders, subjective numbness, chronic headaches, and irritable bowel symptoms are common. Objective signs of inflammation are absent and laboratory studies are normal. Physical examination is normal except for "trigger points" of pain produced by palpation of various areas such as the trapezius, the medial fat pad of the knee, and the lateral epicondyle of the elbow. Polymyositis (A) produces weakness rather than pain. Polymyalgia rheumatica (C) produces shoulder and pelvic pain and is associated with an elevated ESR, occurring after age 50. Rheumatoid arthritis (D) and systemic lupus erythematosus (SLE), (E), present with objective physical findings or abnormalities on routine testing.

Your patient is a 22-year-old male who has experienced significant dysfunction for the past eight months. He exhibits only one symptom of schizophrenia, but that one symptom has been present for a significant part of each day. Which of the following would qualify the patient to be diagnosed with schizophrenia according to the DSM-IV TR? A. A firm belief that someone is following him B. An auditory hallucination in which two or more voices are conversing C. A sensation that bugs are crawling on his skin D. Refusal to go out without a hat, certain he will become ill without one E. Speaking entirely in rhymes that are related but mostly nonsensical

The Correct Answer is: B Generally, two or more characteristic symptoms of schizophrenia are required in order to make a diagnosis unless the one symptom present is a bizarre delusion or an auditory hallucination (B) that consists of either a running commentary or two voices conversing. Positive symptoms of schizophrenia also include disorganized or catatonic behavior and disorganized speech as well as delusions and hallucinations. Negative symptoms include affective flattening, alogia and avolition. (A) and (D) are both potentially delusions, but neither is particularly bizarre. A sensation of bugs on the skin (C, fomication) is a type of hallucination and speaking in rhyme (E) qualifies as disorganized speech. Each of these might count as one of the two required symptoms but would not be sufficient without a second symptom.

According to the ATP III revised guidelines, which of the following should be your primary focus of treatment in patients with dyslipidemia? A. lowering apolipoprotein B B. lowering LDL cholesterol C. Lowering non-HDL cholesterol D. lowering triglycerides E. raising HDL cholesterol

The Correct Answer is: B Given their proven efficacy, ease of administration, and enhanced patient compliance over other classes of medications, statin agents are the drugs of first choice for most patients. In particular, patients with diabetes or those in the highest risk category derive special benefits from their use due to their innate anti-inflammatory effects. Myopathy and elevated liver enzymes are the main potential side effects from statin agents. An increase of serum aminotransferase levels to more than three times normal occurs in 1% of patients taking high doses of statins. Monitoring of liver function tests at six weeks, 12 weeks, six months, and annually thereafter can help identify patients with hepatic side effects and facilitate prompt discontinuation of the agents. Rhabdomyolysis occurs in less than 0.1% of cases. It can be prevented by the prompt discontinuation of the agent when muscle pain and elevated muscle enzymes occur. Unexplained pain in large muscle groups should prompt investigation for myopathy; however, routine monitoring of muscle enzymes is not supported by any evidence. Side effects from statins may not be class specific. Therefore, a side effect with one agent should not prevent a trial with another statin agent. Prior concerns about statins causing cataracts or cancer have been alleviated by the release of two large meta-analyses in 2001.

A 76 year-old woman with steroid dependent chronic obstructive pulmonary disease is hospitalized with fever, chills, and a productive cough. The sputum gram stain shows many WBCs and small, pleomorphic gram-negative rods. Which of the following is the most likely causative agent? A. Chlamydia pneumoniae B. Haemophilus influenzae C. Mycoplasma pneumoniae D. Staph aureus E. Strep pneumoniae

The Correct Answer is: B Haemophilus influenzae (B) is a gram-negative pleomorphic coccobacillus. Strep pneumonia (E) and Staph aureus (D) are gram positive organisms. Mycoplasma pneumonia (C) and Chlamydia pneumoniae (A) aren't visible on gram stain.

A 65-year-old woman presents with a complaint of blood in her urine, intermittently for the last month. The patient denies any fever, chills, flank pain, or dysuria. Social history is positive for tobacco use (45 pack years), but patient reports stopping her tobacco use last year. What is the most likely cause of her hematuria? A. urinary tract infection (UTI) B. bladder cancer C. renal calculi D. pyelonephritis

The Correct Answer is: B Hematuria in women older than 60 years is consistent with a bladder malignancy. Bladder cancer causes episodic, gross hematuria that is usually painless. Cigarette smoking is a risk factor that also increases the incidence of bladder cancer. Painful hematuria associated with suprapubic discomfort or dysuria (or both) is more indicative of cystitis or calculi. Pyelonephritis is associated with chills, fever, and flank pain.

A 34-year-old male presents to the primary care office with a complaint of heartburn that has been present for three months. He has symptoms two to three times a week, which occurs about 30 minutes after eating. He has tried over-the-counter antacids and they were helping to relieve his symptoms for a few months, but they are not working well now. He denies dysphagia, odynophagia, or weight loss. You decide to treat him with a proton pump inhibitor at this visit, and he achieves good symptomatic relief with this therapy. What length of therapy is appropriate in this patient? A. Two to four weeks B. Eight to twelve weeks C. Four to six months D. One year E. Continue indefinitely

The Correct Answer is: B If a patient achieves good symptomatic relief with a course of an empiric, once-daily proton pump inhibitor, therapy may be discontinued after eight to twelve weeks.

What is the most common joint dislocation in children? A. Ankle B. Elbow C. Finger D. Knee E. Shoulder

The Correct Answer is: B In children the elbow is the most commonly dislocated joint and it is the third most common joint dislocation for adults. The shoulder and finger are dislocated more frequently than the elbow in adults. Knee dislocations at the femur-tibia joint are rare (patellofemoral dislocations are more common) and ankle dislocations are also relatively rare.

A 23 year-old vegan presents to the clinic complaining of fatigue. Initial CBC reveals a hemoglobin of 11.1 mg/dL and an MCV of 113 fL. Which of the following best describes the cause of the abnormality pictured in the patient's peripheral smear? (note photo taken from figure 57-5- in Harrison's) A. Absent or non-functioning spleen B. Failure of nuclear maturation C. Intravascular hemolysis D. Lead intoxication E. Presence of uremia

The Correct Answer is: B Macrocytes are present on the peripheral smear and result from failure of nuclear maturation commonly secondary to vitamin B 12 or folate deficiency. Howell-Jolly bodies result from the lack of removal of nuclear material due to an absent or non-functioning spleen (A). Intravascular hemolysis (C) creates schizocytes, lead intoxication (D) and thalassemia cause basophilic stippling, and uremia (E) is associated with Burr cells.

A 1 day-old boy develops progressing abdominal distension, bilious vomiting and failure to pass a meconium stool. Abdominal radiographs show dilated loops of small bowel. Which of the following genetic mutations should the patient be evaluated for? A. ΔF508 B. G551D C. Q1412X D. R117H E. W1282X

The Correct Answer is: B Mutations in CFTR protein function resulting from genotype G551D mutations are amenable to treatment with ivacaftor (B). Approximately 5% of CF patients have the G551D mutation and all patients should be assessed for potential ivacaftor therapy. ΔF 508 (A) is the most common genotype occurring in 60-66% of all CF patients and is not amenable to ivacaftor therapy.

A 16-year-old girl presents to the clinic complaining of strong desires to sleep at inappropriate times. She is very concerned because she "felt paralyzed" while falling asleep on the couch last night. Which of the following is the best diagnostic test to confirm this patient's diagnosis? A. CT of the head B. multiple sleep latency test C. Tensilon test D. thyroid stimulating hormone E. polysomnography

The Correct Answer is: B Narcolepsy is characterized by hypersomnolence, loss of muscle tone prior to sleep, hallucinations upon initiating or arising from sleep, and episodes of sleep paralysis. The diagnostic test that is used in conjunction with clinical history to establish the diagnosis is the multiple sleep latency test. The Tensilon test is utilized to assess for the presence of myasthenia gravis. Polysomnography can be useful in excluding other sleep disorders, but it does not assess sleep latency time necessary to support the diagnosis of narcolepsy.

You are evaluating a 67-year-old female who has history and physical exam findings consistent with osteoarthritis. Based on your knowledge of the disease, which joints are most commonly affected? A. Carpometacarpal (CMC) B. Distal interphalangeal (DIP) C. Metacarpophalangeal (MCP) D. Proximal interphalangeal (PIP) E. Radiocarpal

The Correct Answer is: B Osteoarthritis can affect all of the joints mentioned, but generally has the highest prevalence in the DIP joints, especially the second DIP joint. Osteophyte formation at the DIP joints produce enlargements referred to has Heberden's nodes. The first carpometacarpal joint is likely the second most commonly affected joint. This painful and potentially debilitating condition at the base of the thumb can make grasping activities difficult and be exacerbated by prolonged or strenuous use of the thumb. While metacarpophalangeal and proximal interphalangeal joints can definitely be damaged in osteoarthritis, they are the more classic locations for rheumatoid arthritis changes to be manifested. Radiocarpal joints at the wrist are susceptible to osteoarthritis, but not at the same high frequency as the DIP joints.

How many types of primary osteoporosis are there? A. 1 B. 2 C. 3 D. 4 E. 5

The Correct Answer is: B Osteoporosis is a condition characterized by low bone mass, which increases the fragility of bones and leads to an increased risk of fracture. Osteoporosis is defined as being either primary or secondary and primary osteoporosis is further broken down into Type 1 and Type 2. Type 1 is related to decreased hormone levels - estrogen in women and testosterone in men, and is sometimes referred to as "postmenopausal osteoporosis." It is six times more common in women and results in loss of trabecular bone. Type 1 primary osteoporosis often presents with vertebral compression fractures or fractures of the distal radius after a fall. Type 2 primary osteoporosis is sometimes referred to as "senile osteoporosis" and generally occurs in patients over 70 years of age. It is twice as common in women as men and occurs due to a diminished capacity to make new bone. The most common types of fractures found in this type of osteoarthritis are hip and pelvic fractures. Secondary osteoporosis occurs at a somewhat higher rate in men versus women and is caused by some other medical condition that produces bone loss. Common causes include long-term steroid use, various endocrine abnormalities, and neoplastic diseases such as multiple myeloma.

A 22-year-old non-obese female presents to the clinic with sign and symptoms of deep vein thrombosis (DVT). She denies recent trauma, prolonged immobilization, smoking, use of oral contraceptives, or changes in her health. Her last menstrual period was one week ago. She should avoid exposure to which of the following medications? A. Aspirin B. Combined oral contraceptives C. Enoxaparin D. Progesterone-based contraceptives E. Warfarin

The Correct Answer is: B Patients with factor V Leiden mutation are at risk for thrombotic events when exposed to combined oral contraceptives. Aspirin (A) is not effective for the treatment of DVT and increases the risk of bleeding if co-administered with enoxaparin or warfarin. A patient may be treated with progesterone only-based contraceptives (D) as a contraception option, and may require enoxaparin (C) or warfarin (E) to manage thrombotic risk or events.

A 72-year-old female presents with a 4-month history of pain and stiffness in her shoulders and hips. She identifies the pain being worse in the morning and aggravated with getting in and out of the car along with difficulty brushing her hair. She also reports malaise and a 10-pound weight loss over the past few months. Her blood work shows an erythrocyte sedimentation rate (ESR) of 74 mm/h. What is the best treatment for the suspected diagnosis? A. Clindamycin B. Prednisone C. Acetaminophen D. Tramadol E. Oxycodone

The Correct Answer is: B Patients with polymyalgia rheumatica (a clinical diagnosis based on pain and stiffness of the shoulder and pelvic girdle areas, frequently in association with fever, malaise, and weight loss) are treated with prednisone 10-20 mg/day orally. Usually after 2-4 weeks of treatment, slow tapering of the prednisone can be attempted. Most patients require some dose of prednisone for a minimum of approximately 1 year; 6 months is too short in most cases. Clindamycin (A) is an antibiotic that will not help polymyalgia rheumatic. Acetaminophen (C) alone has not been shown as an effective treatment. Tramadol (D) and oxycodone (E) are pain medications that will not help with the specific condition.

A 6-week-old male with sickle cell disease presents to the pediatric office for his well-child visit. When should this child begin taking daily prophylactic penicillin? A- Now B - 2 months C - 6 months D- 12 months E- 5 years

The Correct Answer is: B Patients with sickle cell disease develop functional asplenia as early as 3 months of age and should begin treatment with prophylactic penicillin at 2 months of age to prevent infection by encapsulated organisms (i.e., pneumococcus). Discontinuation of therapy can be considered beginning about age 5 (E).

A 19-year-old female patient presents to her family practice office for her annual Pap test and her first dose of the quadravalent human papillomavirus (HPV) vaccine. As her physician assistant, when would you schedule her to come in for her second dose of the HPV vaccine? A. two weeks B. two months C. three months D. four months E. six months

The Correct Answer is: B People are frequently confused by differences between the quadravalent and bivalent versions of this vaccine. But dosing schedules are not one of the confusing issues. The Centers for Disease Control and Prevention (CDC), Advisory Committee on Immunization Practices (ACIP), and the manufacturers of both the HPV4 and the HPV2 vaccines all agree: The dosing and administration schedules are the same for HPV4 and HPV2. Each dose is 0.5 mL, administered intramuscularly, preferably in a deltoid muscle. The vaccines are administered in a three-dose schedule. The second dose is administered one to two months after the first dose, and the third dose is administered six months after the first dose.

A 16-year-old boy presents to the office with complaints of a rash, low-grade fever, headache, and malaise. Symptoms began yesterday after he spent most of his free time in the past 4 days deer hunting in the woods around his house. He reports that he does check himself for ticks every night. He often finds them but has not noticed any this season that were latched on to his skin. On examination, his temperature is noted to be 99.9°F, his HEENT is unremarkable, and he has 1 to 2 mm red macules over wrists and ankles with remainder of skin clear. Heart, lungs, and abdomen unremarkable. The most likely diagnosis in this patient is A. Lyme disease B. Rocky Mountain spotted fever C. ehrlichiosis D. Q fever

The Correct Answer is: B Rocky Mountain spotted fever (RMSF) is a rickettsial infection caused by Ricketsia ricketsii. The organism is transmitted to humans through the bite of the dog tick and is more common among those who spend time outdoors in a wooded area. The illness begins with generalized symptoms of fever, headache, nausea, vomiting, malaise, and myalgias. The rash of RMSF begins as a macular rash and progresses to nonblanching petechiae. The rash begins over the wrists and ankles and progresses to the arms, legs, and trunk. Untreated, it can progress to respiratory failure and/or central nervous system involvement. Serologic confirmation is not usually valid until 7 to 10 days after clinical symptoms begin so treatment is often begun empirically. Drug of choice is doxycycline 100 mg po bid until the patient is afebrile and clinically better for 2 to 3 days. Lyme disease is distinguished from RMSF by the pattern of the rash. Lyme disease is characterized by the classic erythema chronicum migrans rash, usually on the trunk. Ehrlichiosis usually does not manifest with a rash. It begins with the same general symptoms but can progress to a toxic shock syndrome. Q fever can be transmitted by ticks, but it is often acquired through contact with sheep, cattle, and goats. It has similar generalized symptoms but can progress to a cough and pneumonia. It is usually without rash. All of these illnesses respond to doxycycline.

What term is used to describe the forward movement of one vertebral body on the vertebra below it, as shown in this figure? A. Spina bifida B. Spondylolisthesis C. Spondylolysis D. Sprengel's deformity E. Sustentaculum tali

The Correct Answer is: B Spondylolisthesis is the forward movement of one vertebral body on the vertebra below. This most commonly occurs with L5 on S1 (more than 85% of cases) or L4 on L5. It is often due to a spondylosis (a bony defect) in the pars articularis as pictured above which is an acquired condition that may develop as a result of a stress fracture and it is not uncommon in children or adolescents. A small number of spondylolithesis cases are congenital. Spina bifida refers to a non-union of the vertebral arch at the spinous process, which is a congenital condition. Sprengel's deformity refers to a scapula that only partially descends from the neck to the thorax. The higher than normal position of the scapula results in a shortened appearance of the neck. The sustentaculum tali refers to an anatomical landmark in the foot that supports the talus and serves as an attachment for the spring ligament. It is part of the calcaneus bone.

A 43 year-old woman recently diagnosed with sarcoidosis returns to the clinic for follow-up. Her chest x-ray demonstrates bilateral hilar lymphadenopathy. What stage of disease is this finding most consistent with? A. Stage 0 B. Stage 1 C. Stage 2 D. Stage 3 E. Stage 4

The Correct Answer is: B Stage 1 (B) disease consists of bilateral hilar and/or mediastinal adenopathy without pulmonary parenchymal abnormalities. Stage 0 (A) is a normal chest x-ray. Stage 2 (C) includes adenopathy and lung parenchymal abnormalities. Stage 3 (D) includes diffuse lung parenchymal disease without nodal enlargement, and Stage 4 (E) is end-stage lung disease with pulmonary fibrosis and honeycombing.

A man who is sexually active with men develops an ulcer on the rim of the anus. This ulcer has a clean, erythematous base and firm, raised margins. He says it is not painful, but he noticed it while wiping himself following a bowel movement several days ago. He has no other symptoms, but does have rubbery nontender lymphadenopathy in the genital area. Of the following, which of the following is the most appropriate test to order at this stage to confirm the suspected diagnosis of syphilis? A- Darkfield microscopic examination B- IgG enzyme immunoassay (EIA) C- Polymerase chain reaction (PCR) D- Rapid Plasma Reagin (RPR) test E- Venereal Disease Research Laboratory (VDRL) test

The Correct Answer is: B The EIA or CIA (chemoluminescence assay) is faster and less expensive than traditional syphilis testing, so current algorithms suggest beginning with one of these tests. The darkfield examination of fresh exudate from a lesion is usually only available in specialized laboratories as it requires expertise in obtaining and examining specimens. (A) PCR testing is increasingly used, but not widely commercially available in the U.S. (C) The RPR and VDRL tests (D, E) do not become positive until 1-3 weeks following the appearance of a chancre.

Which bone is the most susceptible and most often fractured at birth? A. Calcaneus B. Clavicle C. Femur D. Humerus E Patella

The Correct Answer is: B The clavicle is the most common bone broken during childbirth. It often is associated with shoulder dystocia, but clavicular fractures can occur in uncomplicated pregnancies. They are usually of the greenstick variety and heal without complications. Calcaneal and patellar fractures are highly unlikely to occur since they are not long bones which are much more vulnerable to fracture. Fractures of the humerus and femur are possible during childbirth, but generally only in traumatic births. Humerus and femur fractures are much less common than clavicular fractures.

A disk herniation that is putting pressure on the L5 nerve root may present with weakness of what muscle(s)? A. Anterior tibialis B. Extensor hallucis longus C. Gastrocnemius-soleus D. Iliopsoas E. Peroneus longus and brevis

The Correct Answer is: B The extensor hallucis longus muscle's motor function is associated the L5 motor neuron, which also supplies the gluteus medius and extensor digitorum longus and brevis muscles. The anterior tibialis muscle is supplied by the L4 motor neuron. Nerves emanating from T12, L1, L2 and L3 supply the iliopsoas. Gastrocnemius, soleus and peroneus longus and brevis are all supplied by nerves coming from the S1 area. The plantar flexing gastrocnemius and soleus muscles also are supplied by S2.

A 36 year-old smoker with a past medical history of asthma presents to the clinic for follow-up care. What is the first step that should be performed to promote smoking cessation? A. Assess how long after awakening until their first cigarette B. Ask about their smoking and obtain permission to discuss the behavior C. Determine their prior quit attempts and results D. Prescribe varencline E. Share information about the harms of smoking

The Correct Answer is: B The first step in promoting positive behavioral change is to ask if the patient if they smoke and obtain permission to discuss the behavior (B). Assessing the time from awakening to first cigarette (A) is helpful in determining the severity of their nicotine dependence and understanding prior attempts and results (C) forms a basis for assisting smokers interested in quitting. Sharing the harms of smoking (E) should be re-framed positively for patients through advisement on the benefits of quitting. Varnecline (D) is one option available to assist smokers in their attempt to quit.

A 5-year-old male is being evaluated for an acute injury to the right ankle. On the x-ray of the ankle there is a distal tibia fracture that involves the separation of the epiphysis, as well as a small non-displaced chip fracture of the metaphysis of the tibia. Based on these findings, what type of Salter-Harris fracture does this child have? A. I B. II C. III D. IV E. V

The Correct Answer is: B The growth plate is the most fragile part of the bone prior to bone maturation and thus is usually the first structure disrupted when force is applied. Statistically, Type II fractures are most common - those that involve both the growth plate and a chip fracture of the metaphysis.

Upon testing a patient for function of the hip flexors, which muscle is considered the primary muscle responsible for most flexion? A. Gracilis B. Iliopsoas C. Rectus femoris D. Sartorius E. Vastus intermedius

The Correct Answer is: B The iliopsoas muscle is the primary hip flexor muscle. It originates at T12 and L1-5 vertebrae and intervertebral disks as well as the iliac fossa of the pelvis and connects to the femur at the lesser trochanter. The gracilis muscle is considered a secondary hip adductor. Rectus femoris does help with hip flexion, but in a secondary role to the iliopsoas. Rectus femoris is also involved in knee extension. Sartorius is also involved in hip flexion, but in a secondary role. The vastus intermedius muscle is one of the four quadriceps muscles and is involved with knee extension and is not involved in hip flexion.

Which peripheral nerve is involved in the most common compression neuropathy in the upper extremity? A. Axillary B. Median C. Radial D. Sciatic nerve E. Ulnar

The Correct Answer is: B The median nerve is commonly compressed as it passes through the carpal tunnel in the wrist. This syndrome is most often diagnosed in middle aged or pregnant female patients. The axillary nerve passes through the axilla and is often compressed when patients use crutches improperly and bear weight on the axillary area. The ulnar nerve is second only to the median nerve and can be compressed as it passes through the cubital tunnel at the elbow or as it passes through the humeral and ulnar heads of the flexor carpi radialis muscle. Compression of the radial nerve (and its branches) as it passes through the radial tunnel on the lateral side of the elbow is often confused with lateral epicondylitis. The sciatic nerve is associated with the lower extremities and pain is often elicited as a result of a lumbar disk herniation causing nerve root impingement.

A 48-year-old man presents to the clinic for a routine employment physical. The patient is asymptomatic. A pre-employment CBC reveals hemoglobin of 13.2g/dl, hematocrit of 39.5%, and MCV of 60.6 fL. Subsequent iron studies, hemoglobin electrophoresis, and sickle cell screening are "normal." After the labs are reviewed, the patient states "they are always normal." What is the most likely diagnosis? A. Anemia of chronic disease B. Alpha thalassemia minor C. Beta thalassemia major D. Iron deficiency anemia E. Sickle cell disease

The Correct Answer is: B The patient has a mild anemia with pronounced microcytosis consistent with alpha thalassemia minor. Alpha thalassemia minor hemoglobin electrophoresis reveals normal results. Patients with anemia of chronic disease (A) and iron deficiency anemia (D) have abnormal iron studies; while patients with beta thalassemia major (C) and sickle cell disease (E) have abnormal hemoglobin electrophoresis results.

A 26-year-old female patient presents to the emergency department complaining of left eye discharge, without significant pain. She has no known drug allergies. Physical exam reveals the following: A gram stain reveals gram-negative intracellular diplococci. Which of the following is the most appropriate clinical intervention? A. Ceftriaxone 1 gram IM and ophthalmologic follow-up in 48 hours B. Ceftriaxone 1 gram IM and emergent referral to ophthalmologist C. Ciprofloxacin 500 mg twice daily for 10 days and ophthalmogic follow-up in 48 hours D. Ciprofloxacin 500 mg twice daily for 10 days and emergent referral to ophthalmologist E. Emergent referral to ophthalmologist

The Correct Answer is: B The patient has gonococcal conjunctivitis that is an ophthalmologic emergency due to potential risk of corneal perforation (C). Aggressive antibiotic therapy for gonorrhea (i.e., ceftriaxone) with immediate ophthalmology evaluation is the best course of action.

A 29-year-old woman presents to the clinic with a complaint of severe diarrhea occurring over the last 3 to 4 days. Upon examination, the patient displays poor skin turgor and has a temperature of 100.2°F. In the supine position, the patient's blood pressure is 88/64 mm Hg and her heart rate is 112 beats/min. Upon standing, her heart rate further increases to 126 beats/min. Which of the following accounts for the further increase in the patient's heart rate upon standing? A. decreased systemic vascular resistance B. decreased venous return C. increased preload D. increased myocardial contractility E. increased peripheral vasodilation

The Correct Answer is: B The patient is displaying signs of hypovolemia likely because of her chronic diarrhea. Upon standing, most of her low blood volume pools in the veins of her lower extremities because of the effects of gravity. As a result, even less blood returns to the heart, which leads to a decrease in both stroke volume and cardiac output as well as orthostatic hypotension. This elicits the baroreceptor reflex, which attempts to increase and maintain arterial blood pressure by raising the heart rate.

A 22 year-old male presents with a 2-week history of an upper respiratory infection that hasn't improved after taking amoxicillin for 6 days. He notes persistent sore throat, intermittent fever, and a worsening nonproductive cough. Physical examination reveals bilateral diffuse crackles. What is the most appropriate antibiotic to initiate after discontinuing the amoxicillin? A. Amoxicilin and clavulanic Acid B. Azithromycin C. Cephalexin D. Ciprofloxacin E. Trimethoprim/Sulfamethoxazole

The Correct Answer is: B The patient most likely has atypical pneumonia that responds best to macrolide antibiotics (B). Antibiotics that inhibit cell wall synthesis (A, C) are generally ineffective against these atypical organisms that are either intracellular or lack a cell wall.

A 76 year-old man with long-standing asthma presents to the clinic complaining of increased use of his albuterol and 2-3 nighttime awakenings over the last month. Which of the following is the most appropriate therapy? A. Beclomethasone via a pressurized meter dose inhaler B. Fluticasone dry powder inhaler C. Montelukast orally D. Prednisone orally E. Salmeterol dry powder inhaler

The Correct Answer is: B The patient requires inhaled corticosteroid therapy for moderate-persistent asthma. Fluticasone (B) dry powder inhaler is a preferred agent in this geriatric patient who is at increased risk of having poor inhaler technique. Beclomethasone (A) via a pMDI is prone to deposition in the oropharynx, decreased efficacy, and increased side effects. These concerns can be mitigated through the use of a spacer. Montelukast (C) and salmeterol (E) are not preferred agents for this stage of asthma.

A 62 year-old woman with a 40 pack year smoking history presents with a history of progressive dyspnea of several months duration. She denies hemoptysis, cough, fever, or orthopnea. Sputum production is positive. On physical exam, she shows evidence of accessory muscle use when breathing, sits in a slightly bent forward position, uses pursed lips, and you notice she is very thin. Auscultation reveals decreased vesicular breath sounds with prolonged expiration. Which of the following is the most likely diagnosis? A. CHF B. COPD C. Lung cancer D. Pulmonary emboli E. Pulmonary fibrosis

The Correct Answer is: B The patient scenario is consistent with COPD (B) based on the history of progressive dyspnea and physical exam findings of obstructive lung disease (i.e., pursed lip breathing and decreased breath sounds with prolonger expiratory phase. The patient lacks physical exam findings of pulmonary edema due to CHF (A). She is at increased risk of lung cancer (C), but lacks red flag symptoms at this time. Pulmonary embolism (D) typically occurs in patients with venous stasis, hyper coagulable state, and vascular wall injury. Pulmonary fibrosis (E) would present with sign of restrictive lung disease (e.g., shallow breaths with rapid expiration)

A 43-year-old woman presents to the outpatient clinic complaining of right eye redness, photophobia, and pain. She notes some blurred vision and denies the presence of discharge. On physical exam her visual acuity is 20/20 left eye, and 20/60 right eye. Her right eye has circumcorneal injections and the pupil is 3 mm and responds poorly to light. Her left pupil is 5 mm and responds well. Fluorescein staining of the eye is unremarkable and intraocular pressures are normal. Which of the following treatment regimens should be prescribed? A. Homatropine 5% solution four times daily B. Homatropine 5% solution four times daily and prednisolone 1% solution every 1 or 2 hours while awake C. Prednisolone 1% solution every 1 or 2 hours while awake D. Prednisolone 1% solution every 1 or 2 hours while awake and sulfacetamide 10% solution three times a day E. Prednisone 60 mg by mouth once daily

The Correct Answer is: B The patient's presentation of acute uveitis is best treated with topical corticosteroids and cycloplegics (B) once infectious causes (e.g., HSV) have been ruled out. The addition of a cycloplegic helps reduce pain. Antibiotic drops (D) aren't indicated for acute uveitis.

It is September and the radio is flooded with public service announcements recommending people get their flu shots early; flu is not yet endemic. Your patient, an 18 year old female, has come to see you in your family practice clinic with complaints of "flu-like" symptoms. She hoped the symptoms would resolve on their own, but it is now a week later and she is still experiencing them, and requests a rapid flu test. Which of the following is true regarding your patient and the rapid influenza tests? A. It is best she waits at least a week before having you perform a rapid test, since a false negative may result if seen too soon from onset of symptoms. B. You may warn your patient that given this time of year, the low prevalence in the community, and her delay in testing, the results are virtually useless and you would not recommend testing. C. After obtaining a specimen to run the rapid test, you inform you patient that you will have her results in two to three days. D. The greatest cost benefit would be achieved if antibiotics were prescribed once you receive a positive rapid test for her.

The Correct Answer is: B The rapid tests vary in terms of sensitivity and specificity. Research indicates that sensitivities are approximately 50% to 70%, while specificities are approximately 90% to 95%. Specimens to be used with rapid tests generally should be collected as close as is possible to the start of symptoms and usually no more than four to five days later in adults. In very young children, influenza viruses can be shed for longer periods; therefore, in some instances, testing for a few days after this period may still be useful. Most importantly, the positive and negative predictive values vary considerably depending upon the prevalence of influenza in the community. False-positive (and true-negative) influenza test results are more likely to occur when disease prevalence is low, which is generally at the beginning and end of the influenza season, as is the case here. False-negative (and true-positive) influenza test results are more likely to occur when disease prevalence is high, which is typically at the height of the influenza season. When disease prevalence is relatively low, the positive predictive value (PPV) is low and false-positive test results are more likely. By contrast, when disease prevalence is low, the negative predictive value (NPV) is high, and negative results are more likely to be true.

The term "nursemaid's elbow" refers to which of the following physical conditions? A. A fracture of the humerus B. A subluxation of the radial head C. Inflammation at the lateral epicondyle D. Inflammation of the medial epicondyle E. Olecranon bursitis

The Correct Answer is: B This is the most common elbow injury in children under the age of 5. The injury generally occurs when the child's arm is forcefully pulled when the elbow is in an extended position and the forearm is pronated. Fibers of the annular ligament that encircle the radial neck become trapped between the radius and ulna. On presentation, children hold their arm in slight flexion and pronation. There is no fracture associated with nursemaid's elbow and the olecranon bursa is unaffected. Nursemaid's elbow is an acute injury, while medial and lateral epicondylitis (also known as golfer's elbow and tennis elbow respectively) are generally chronic conditions.

A 71-year-old male who has a history of hypertension presents with a new finding of atrial fibrillation. He is independent, drives his own car, and tends to his daily activities without assistance. He is currently having no symptoms, and his heart rate is 90, with a blood pressure of 146/76. Given this clinical scenario, what is the best pharmacologic anticoagulation treatment for this patient? A. No anticoagulation B. Aspirin C. Aspirin and warfarin D. Aspirin and clopidogrel E. Warfarin and clopidogrel

The Correct Answer is: B This patient has a lower CHADS2 score of 1 (HTN) and would have adequate risk reduction with the therapy of aspirin alone (B). Higher risk patients with a CHADS2 score of 2 or higher qualify for adding warfarin (C), and the use of aspirin and clopidogrel (D) is not indicated. No anticoagulation (A) would put the patient at risk for CVA, and warfarin and clopidogrel (E) would create an over anticoagulated environment and increase risk for bleeding.

A 17-year-old female presents to your family practice office for the annual physical examination required by her cheerleading coach. Upon examination you note that her joints are more flexible than anticipated. You also note her long thin fingers. You listen to her heart and hear no murmurs. Her blood pressure is 105/65 mmHg, pulse 60 beats/min and regular, respirations of 15 breaths/min, and temperature 98.7˚F. As you are examining her she tells you that her "joints sprain and strain easily." Furthermore, you obtain family history and she tells you that some connective tissue disorder runs in her family. Before you can medically clear her you should do which of the following? A. This patient does not require any further evaluation. B. Obtain an echocardiogram. C. Perform and EKG. D. Perform a chest radiograph. E. Refer her to a rheumatologist.

The Correct Answer is: B This patient has clear signs and symptoms that are suspicious for Marfan syndrome. The complications of Marfan syndrome include cardiovascular issues, especially valvular and aortic disease. An echocardiogram is an appropriate, non-invasive initial first-step to begin your investigation to rule out significant valvular and/or aortic root abnormalities.

An 84-year-old male is admitted to the hospital for the chief complaint of syncope. The history provided states that the patient was in his normal state of health and feeling well when he had fallen, with an apparent loss of consciousness for 25-30 seconds. He has a history of hypertension and arthritis, and is well managed on medications that include lisinopril and acetaminophen. During his time on the telemetry unit it's noted that the patient has periods of sinus bradycardia in the 30s, followed by normal sinus rhythms that fluctuate in the 60 to 120 range. During several of the bradycardia episodes the patient becomes symptomatic with shortness of breath, lightheadedness, and dizziness. His blood pressure during these episodes is measured at 88/56. Given this clinical scenario, what is the most likely diagnosis for this patient? A. Ventricular tachycardia B. Sick sinus syndrome C. First-degree AV block D. Wolff-Parkinson-White syndrome E. Premature atrial contractions

The Correct Answer is: B This patient is exhibiting signs and symptoms related to sick sinus syndrome (B). There is no evidence in the clinical scenario to suggest any ventricular ectopy (A), and the patient's age and characteristics of the arrhythmia are not consistent with Wolff-Parkinson-White syndrome (D), or other skipped beat abnormalities (C and E).

A 18 year-old man present to the clinic due to a recent positive tuberculosis screening test. Which of the following steps should be taken to avoid neurotoxicity associated with his prophylactic isoniazid (INH) treatment? A. Perform an initial screening mental status exam B. Supplement with folic acid C. Supplement with pyridoxine (vitamin B6) D. Take medication on a full stomach E. Take the medication three times daily

The Correct Answer is: C INH leads to the development of peripheral neuropathy in 2% of patients secondary to pyridoxine deficiency. This can be prevented through the coadministration of pyridoxine (vitamin B6) (C).

A 22-year-old African American male presents to the emergency department with shortness of breath, which started 2 hours prior to arrival. He does not have a history of pulmonary disease that he is aware of, and he states that in the past at random events he has had similar episodes. He does nothing to get the episodes to stop, and he also states that he feels his chest pounding at the same time of the shortness of breath. He has no medical history that he is aware of, and he takes no medications or any illicit drugs. On examination he is alert, awake, and oriented. His vital signs show T 99.0, P 142, R 18, and BP 132/82. His chest x-ray is negative for any acute cardiopulmonary disease, and his electrocardiogram has an irregularly irregular rhythm and a rate of 142 with visible delta waves. Based on the information provided, what is the most likely diagnosis for this patient? A. Torsade de pointes B. Atrial fibrillation with Wolff-Parkinson-White syndrome C. Atrial fibrillation with re-entrant tachycardia D. Junctional tachycardia E. Atrio-ventricular re-entrant tachycardia

The Correct Answer is: B This patient most likely has Wolff-Parkinson-White (WPW) syndrome (B), based on the explanation of the delta wave findings on EKG. In addition, the patient is also having atrial fibrillation with a rapid ventricular response. Patients that present with a new finding of WPW can often present with atrial fibrillation, along with symptoms such as those described above. There is nothing noted that gives the history of ventricular tachycardia (D), or re-entrant tachycardia (C and E). Torsade de points (A) is an ventricular tachycardia without evidence of a noticeable rhythm.

A 69-year-old female presents with her family to her primary care provider. The family is concerned that the patient is increasingly forgetful and has difficulty remembering the names of people and places involved in her life for many years. The symptoms have worsened over the past five months. She often gets lost while driving and is found in unusual parts of the house. She has not had any physical complaints, has had no weight changes, and denies recent illnesses. Which of the following is the most likely explanation for these symptoms? A. Delirium B. Dementia C. Depression D. Hydrocephalus E. Hypothyroidism

The Correct Answer is: B This patient represents a deterioration of mental functioning over time, with an absence of physical illness symptoms. The decreased cognition is causing an impairment of activities of daily living. There is no indication of altered consciousness. The onset was not rapid, nor associated with recent medical conditions or other physical symptoms. This is most consistent with dementia. The other etiologies among the answer choices must be ruled out, although this patient does not exhibit the classic findings for any of the other diagnoses.

A 64-year-old African American female presents to the clinic for evaluation of her hypertension, which she has had for several years. In the past she had been taking hydrochlorothiazide and lisinopril, with little effect on her blood pressure management. At today's visit, she has no complaints and feels well. Her vitals show T m 96.6, P 85, R 18, BP 191/99. She has no jugular venous distention seen on the neck exam, her lungs are clear, and cardiac exam has a regular rate and rhythm without murmur or gallop. Her abdomen is soft, non-tender, and a bruit is appreciated at the mid-abdomen just a few centimeters below the epigastric region. There is no fullness or enlargement of the abdominal aorta on palpation. Based on the history and clinical findings, what is the most appropriate procedure for this patient? A. Hepatic venous angiography B. Renal artery angiography C. Cardiac catheterization D. Lower extremity arteriography E. Venous duplex ultrasound of lower extremities

The Correct Answer is: B To appreciate the extent of this patient's renal artery stenosis by history and physical exam, the patient would benefit the best by renal artery angiography (B). This test will help determine the extent of stenosis, as well as map the surrounding arterial supply. Hepatic venous arteriography (A) and cardiac catheterization (C), and lower extremity arteriography (D) would not be indicated given the physical exam findings. Venous duplex ultrasounds would not yield the appropriate information given this clinical scenario.

A 33 year-old man with long-standing, recurrent, abdominal pain is diagnosed with anemia secondary to vitamin B12 deficiency. Which of the following gastrointestinal disorders most likely explains his diagnoses? A. Achalasia B. Crohn's disease C. Duodenal ulcer D. Pancreatitis E. Ulcerative colitis

The Correct Answer is: B Vitamin B 12 complexed with intrinsic factor produced by gastric parietal cells is absorbed in the ileum, which is commonly affected by Crohn's disease (B). Achalasia (A) can lead to malnutrition and anemia secondary to vitamin B12, folate, and iron, but presents as difficulty swallowing, chest pain, and heartburn. Duodenal ulcer (C) and ulcerative colitis (E) may result in chronic blood loss and the development of iron deficiency anemia. Pancreatitis (D) causes malabsorption of fat soluble vitamins and may also lead to vitamin B12 deficiency in rarer instances.

A 59-year-old female patient with no complaints is undergoing routine physical examination in your family practice office and has a hypochromic, microcytic anemia that was not present on her physical examination last year. Which of the following is the most important cause to rule out in this patient? A. acute pathology secondary to aplastic anemia B. chronic anemia secondary to colon cancer C. Falconi's anemia D. iron deficiency anemia E. medication-induced anemia

The Correct Answer is: B While there are many causes of hypochromic, microcytic anemia, high on the differential is blood loss, commonly occult and long standing in nature. This can be from heavy menstruation or gastrointestinal loss. This later is commonly caused by an occult colon cancer. This patient should initially receive a sensitive screen such as an immunochemical fecal occult blood test (IFOBT) and, given her age, a colonoscopy should be strongly considered.

You are performing a mental status exam on a 19-year-old male patient who was brought to the Emergency Department because of bizarre behavior. As you are speaking with him he keeps repeating words that rhyme with words that either you or he say, but make no sense and are unrelated to your questions. How will you document this behavior in his chart? A. Circumstantiality B. Clanging C. Flight of ideas D. Perseveration E. Tangentiality

The Correct Answer is: B Word association based on rhyme is called clang association (B) and may be seen in psychotic disorders. Circumstantiality (A) is a disturbance in fluency where the speaker meanders on many side topics before returning to the topic at hand. Flight of ideas (C) is a rapid transition from thought to thought, leading to the speaker losing track of the original idea. Perseveration (D) is a fixed focus returning again and again to the same thought. Tangentiality (E) refers to a disturbance in continuity where the speaker shifts from one thought to another that may be only vaguely related.

You are evaluating a 53-year-old post-menopausal patient during her routine annual examination in a primary care office. You notice that the patient has not been properly assessed for risk for osteoporosis, and does have some complaints that are of concern for osteoporosis. Based on this history, and the standard of care, what test is used to confirm osteoporosis in this patient? A. Bone scan B. CT scan C. DEXA scan (dual energy x-ray absorptionmetry) D. MRI scan E. X-ray

The Correct Answer is: C A DEXA scan is the best test to determine bone density and is commonly used as a screening tool for those at risk of developing osteoporosis. This test is fast, reproducible and exposes patients to a relatively low dose of radiation. The results are reported as Z and T scores. Z scores indicate how the patients bone density compares to peers and T scores compare the patient to young, health individuals. Z and T scores are presented in the form of standard deviations below the comparison group. If the T score is 0 to greater than -1 the test is interpreted as normal. T scores of -1 to -2.5 are indicative of someone with osteopenia and a T score lower than -2.5 is consistent with osteoporosis. None of the other imaging options presented are ideal for determining bone density, but they each play important roles in other aspects of patient care. Bone scans are best suited for imaging studies designed to look for occult fractures, tumors, inflammatory or infectious processes within the bones, or metabolic bone diseases. CT is useful for a detailed examination of bone when looking for fractures or lesions and intra-articular pathology. MRI is helpful in the evaluation of spinal column pathology and various soft tissue injuries involving the muscles, tendons, ligaments, and cartilage. MRI is also useful in the evaluation of stress and occult fractures, osteomyelitis and early osteo necrosis. Radiographs (x-ray) are useful for most initial evaluation of musculoskeletal pathology, but are somewhat limited in their usefulness in visualizing some conditions. Low bone density can be perceived on x-ray, but only after the bone loss has been rather extensive. It is not a great early screening tool for osteoporosis.

What types of connective tissue are injured in a sprain? A. Bones and muscles B. Fascia and joint capsules C. Ligaments and joint capsules D. Muscles and tendons E. Tendons and bones

The Correct Answer is: C A sprain involves injury to those tissues that give support to joints - ligaments and joint capsules. Injury to muscles, tendons, and fascia would all be classified as a strain. Injuries to bone would be classified as a fracture.

A 64-year-old female presents with acute onset of severe right knee pain. She denies any trauma and her history is only significant for diabetes mellitus. On physical examination her knee has significant edema and erythema, with warmth on palpation. After an aspiration of the synovial fluid the results are: Color: opaque yellow fluid WBC: 90,000/mcL PMN: 90% Considering your suspected diagnosis what is the best treatment for this patient? A. IV analgesics and observation B. IV fluids C. Hospitalization and IV antibiotics D. PO antibiotics and follow-up in 3 days E. MRI for further evaluation

The Correct Answer is: C Acute bacterial septic arthritis is associated with white blood cell counts commonly > 50,000/mcl with 90% or more polymorphonuclear cells. Compromised immunity, such as diabetes mellitus, increases the risk of septic arthritis. The effective treatment of septic arthritis requires appropriate antibiotic therapy together with drainage of the infected joint. Hospitalization is always necessary to prevent sepsis and administer IV antibiotics (C). IV analgesics (A) can be used but are not a necessary treatment option. IV fluids (B) may help but would not treat the septic joint. Oral antibiotics would not work as quickly and the patient should be monitored until the infection is controlled, sending them home on PO antibiotics would not be recommended (D). Imaging tests (E) generally add little to the diagnosis of septic arthritis.

A 23 year-old woman seeks guidance on foods, medications, and supplements that can help increase the amount of iron supplement. Which of the following is the best advice to provide this patient? A. Co-administer with a calcium supplement B. Co-administer with proton pump inhibitors C. Co-administer with vitamin C D. Taking with a basic beverage E. Taking with food

The Correct Answer is: C An acidic achieved through presence of additional acids (C) (e.g., ascorbic acid) increases absorption of iron. Food (E), other heavy metals (A), and basic stomach environments (B, D) negatively affect the absorption of iron.

In taking the family history of your 25-year-old male patient, you discover that he has numerous relatives with breast and ovarian cancers. In the past, his mother received genetic counseling and testing for the BRCA1 and BRCA2 gene mutations and was found to be positive for a mutant allele. What is his risk for developing this genetic cancer? A. His chance is 0%, because this is not transmitted to men. B. 25% C. 50% D. 75% E. 100%

The Correct Answer is: C BRCA1 and BRCA2 gene mutations are expressed in the cells of breast and other tissue, where they help repair damaged DNA, or destroy cells if DNA cannot be repaired. If the BRCA1 or BRCA2 itself is damaged, damaged DNA is not repaired properly and this increases risks for certain cancers. These genes are inherited in an autosomal dominant manner so if his mother is positive (and his father is not), his risk is 50%, and he is also at increased risk for not only breast cancer, but also prostate, pancreatic, and other cancers.

A 64-year-old African American female presents to the clinic for evaluation of her hypertension, which she has had for several years. In the past she had been taking hydrochlorothiazide and lisinopril, with little effect on her blood pressure management. At today's visit, she has no complaints and feels well. Her vitals show T m 96.6, P 85, R 18, BP 191/99. She has no jugular venous distention seen on the neck exam, her lungs are clear, and cardiac exam has a regular rate and rhythm without murmur or gallop. Her abdomen is soft, non-tender, and a bruit is appreciated at the mid-abdomen just a few centimeters below the epigastric region. There is no fullness or enlargement of the abdominal aorta on palpation. Based on the history and clinical findings, what is the most appropriate next test for this patient? A. Computed tomography of the abdomen B. Nuclear exercise stress test C. Renal duplex ultrasound D. Echocardiogram E. Electrocardiogram

The Correct Answer is: C Based on the information of the presentation, the patient most likely has renal artery stenosis, which is causing uncontrolled hypertension. The most appropriate test for confirmation of this is a renal duplex ultrasound (C). This test will have a higher specificity and sensitivity than the other listed tests (A, B, D, and E), as well as determine flow.

Which condition is suggested by urethritis, arthritis, and conjunctivitis? A. chlamydial infection B. gonococcal infection C. reactive arthritis D. tertiary syphilis

The Correct Answer is: C Both chlamydia and gonorrhea infections can result in urethritis. Gonococci can disseminate to the joints and cause septic arthritis. Chlamydia is typically asymptomatic but can cause chronic conjunctivitis in adolescents and young adults. Reactive arthritis (also known as Reiter syndrome) is a result of an untreated chlamydia infection, and although typically characterized, in texts, by the triad of urethritis, arthritis, and conjunctivitis, all of the symptoms may not be present or not identified at the time of presentation. Tertiary syphilis is characterized by neurologic and cardiovascular disease, gumma, auditory and ophthalmic involvement, and cutaneous lesions.

A 45-year-old male comes into your family practice office for his second follow-up appointment since being diagnosed with type 2 diabetes. He does not complain of any symptoms. He is currently taking metformin 1000 mg BID and his HgA 1c at this visit 7.6%. You ask him about his diet and if he is regularly monitoring his blood glucose. He says that he does not really watch what he eats, but he does check his glucose levels daily. He tells you that on average his fasting plasma glucose (FPG) is usually around 88 mg/dl and his postprandial glucose (PPG) is around 180 mg/dl. What would be the BEST next appropriate step in management for this patient? A. Nothing, his levels are within normal limits. B. Consider switching to basal insulin therapy to control his FPG. C. Add a dipeptidyl peptidase 4 (DPP-4) inhibitor to control his PPG. D. Add a sulfonylurea to control his PPG. E. Increase his metformin to 1500 mg twice daily.

The Correct Answer is: C DPP-4 inhibitors modulated glucagon-like peptide-1 (GLP-1). Their mechanism of action is thought to result from increased incretin levels, especially GLP-1. GLP-1 inhibits glucagon release which, in turn, results in increased insulin secretion, delays gastric emptying, and decreases serum glucose levels. The class is particularly appropriate to utilize in patients who have near-normal HbA 1C and elevated postprandial serum glucose because they work only when food enters the gut and have little to any chance of hypoglycemia.

A 62-year-old male diabetic patient presents to your family practice office with a fever and cough, and after a physical examination you perform a chest x-ray, which reveals a right middle lobe pneumonia. You prescribe levofloxacin 750 mg PO for five days and give the patient his first dose in the office prior to sending him home. Within five minutes, he develops generalized urticaria, shortness of breath with a pronounced wheeze, and becomes hypotensive. While your staff calls 911, what is the most important agent to administer first? A. cimetidine B. dipenhydramine C. epinephrine D. methylprednisolone E. none of the above

The Correct Answer is: C Early recognition of an anaphylactic reaction is mandatory, since death occurs within minutes to hours after the first symptoms. Mild symptoms such as pruritus and urticaria can be controlled by administration of 0.3 to 0.5 mL of 1:1000 (1.0 mg/mL) epinephrine SC or IM, with repeated doses as required at 5- to 20-minute intervals for a severe reaction.

A 23-year-old female presents to the clinic for evaluation of a pre-employment physical examination with evidence of a first-degree AV block on ECG. She is otherwise healthy and without any medical history, is not on any medications, and is symptom free. Based on this history, what is the treatment for this patient? A. Beta-blockers B. Definitive electrophysiology study C. No treatment, only monitoring D. Ablation therapy E. Calcium channel blockers

The Correct Answer is: C First-degree AV block needs no immediate therapy. The treatment is to monitor the patient for any changes that may occur if a new disease presents itself.

A 7-year-old is diagnosed with an acute case of hematogenous osteomyelitis accompanied with fever and leukocytosis. Based on your knowledge of the disease, which bone is most likely to present with the infection? A. Feet B. Hands C. Long bones D. Pelvis E. Vertebrae

The Correct Answer is: C Fortunately, hematogenous osteomyelitis is not common in children, but when it does occur it primarily is found in the long bones. The femur, tibia and humerus are the most typical locations for osteomyelitis in children. The highly vascular metaphysis of long bones contribute to the potential for hematogenous spread of the implicated pathogen. Osteomyelitis can occur at any of the locations mentioned in the answer choices given, but at a significantly lower rate than in the long bones. The rate of occurrence at several selected locations is given below: Feet - 9% Femur 25% Hands - 6% Humerus - 13% Pelvis - 8% Radius/ulna - 6% Tibia/fibula - 28% Vertebrae - 2%

A 46-year-old female presents with pain to her left wrist. She complains that it is painful and swollen as she points to the volar aspect of the wrist on the radial side. On examination, there is a small, soft bump on the dorsum of her wrist with a jelly-like consistency. What is the most likely diagnosis? A. Cancerous tumor B. Fracture C. Ganglion cyst D. Hematoma E. Lipoma

The Correct Answer is: C Ganglion cysts commonly occur on the dorsal or volar aspect of the wrist. They result when a joint capsule or tendon sheath is damaged, allowing synovial fluid to escape producing a one-way valve, which allows fluid into the cyst, but not back out. The accumulating fluid forms the ganglion cyst. These cysts may or may not be tender and can fluctuate in size depending on activity level of the affected extremity. Cancerous tumors would tend to be much more firm, but also may be relatively pain free. Fractures would generally be exquisitely tender and if the bump is due to a displaced bone, it would be much more firm than a ganglion cyst. Hematomas are generally associated with acute trauma and would be tender and ecchymotic in many situations. Lipomas are benign fatty tumors that are more commonly seen on the thenar eminence than the dorsum of the wrist and their size does not change based on activity level.

A 53 year-old man with a past medical history of coronary artery disease, hypertension, diabetes, and allergic rhinitis is admitted to the hospital secondary to a non-ST elevation MI. On his second hospital day he develops severe thrombocytopenia with a platelet count of 17,000. Which of the following medications most likely caused his thrombocytopenia? A. Atenolol B. Erythromycin C. Heparin D. Loratadine E. Metformin

The Correct Answer is: C Heparin (C) directly causes the production of heparin/PF4 antibodies that affect platelets, which leads to decreased platelet numbers, but an increased thrombosis risk. Atenolol (A), erythromycin (B), loratadine (D) and metformin (E) are not associated with thrombocytopenia.

An 11-month-old African-American male presents to the pediatric office with lethargy, jaundice and splenomegaly. A CBC reveals hemoglobin of 8.0 mg/dl and a hematocrit of 25%. Peripheral smear appearance is available below. Which of the following treatments will reduce hemolysis and increase hemoglobin levels for this patient? A. Allogeneic stem cell transplant B. Folic acid supplementation C. Hydroxyurea D. Prophylactic penicillin E. Pneumococcal vaccine

The Correct Answer is: C Hydroxyurea directly reduces hemolysis and increases levels of fetal hemoglobin and reduces complications and transfusion frequency. Sickle cell disease can be cured in 80% of individuals who receive a suitable transplant (A), but this procedure carries great risk and donors are difficult to identify. Folic acid supplementation (B) and pneumococcal vaccine (E) are indicated for preventive therapy of all patients with sickle cell disease, and prophylactic antibiotics (D) are used to prevent recurrent infections.

Of the following sexual practices, which poses the greatest risk of HIV transmission when practiced with an infected partner but without the use of a reliable barrier method of prophylaxis? A. insertive anal intercourse B. insertive vaginal intercourse C. receptive anal intercourse D. receptive fellatio with ejaculation E. receptive vaginal intercourse

The Correct Answer is: C In unprotected intercourse with an infected partner, receptive anal intercourse carries a risk of HIV transmission between 1:100 and 1:30. Insertive anal intercourse, receptive vaginal intercourse, and fellatio with ejaculation each carry a risk of about 1:1000. Insertive vaginal intercourse carries a risk of 1:10,000.

A 73-year-old female with type 2 diabetes, hypertension, and hyperlipidemia presents to the outpatient clinic complaining of left ear pain, and a yellowish-green, foul-smelling discharge that began about 3 weeks ago. On physical examination, the patient is afebrile and examination reveals a markedly edematous left ear canal draining purulent, green discharge. The tympanic membrane is unable to be visualized. Which of the following is the most likely diagnosis? A. Auricular cellulitis B. Cerebrospinal fluid leakage C. Malignant otitis externa D. Otitis externa E. Serous otitis media with perforation

The Correct Answer is: C Malignant otitis externa (C) is a more serious form of otitis externa (D) that most commonly occurs in patients with diabetes and is most commonly caused by pseudomonas. The case scenario describes the location as the external auditory canal without significant involvement of the auricle (A). Serous otitis media with perforation (E) and CSF leak (B) would present with a clear drainage.

A 73-year-old female with type 2 diabetes, hypertension, and hyperlipidemia presents to the outpatient clinic complaining of left ear pain, and a yellowish-green, foul-smelling discharge that began about 3 weeks ago. On physical examination, the patient is afebrile and examination reveals a markedly edematous left ear canal draining purulent, green discharge. The tympanic membrane is unable to be visualized. Which of the following is the most likely causative agent for this patient's diagnosis? A. Escherichia coli B. Moraxella catarrhalis C. Pseudomonas aeruginosa D. Staphylococcus aureus E. Streptococcus pneumoniae

The Correct Answer is: C Malignant otitis externa is most commonly caused by pseudomonas (C). E coli (A) and S aureus (D) are less common causes of otitis externa, while S pneumoniae (E) and M catarrhalis (B) are common etiologies of acute otitis media.

What absolute tissue pressure generally is used as a guideline for diagnosing compartment syndrome? A. 10 mm Hg B. 20 mm Hg C. 30 mm Hg D. 40 mm Hg E. 50 mm Hg

The Correct Answer is: C Many trauma surgery services use an absolute tissue pressure of approximately 30 mm Hg as the threshold for diagnosing compartment syndrome. Based on the entire clinical picture, patients with numbers in that range or higher will likely require surgical decompression with a fasciotomy, while lower numbers will probably be managed with a more conservative approach.

Which type(s) of Salter-Harris fractures can generally be treated with closed reduction and cast immobilization? A. Type I B. Types I and II C. Types I, II, and III D. Types I, II, III, and IV E. Types I, II, III, IV, and V

The Correct Answer is: C Minimally displaced Salter-Harris types I, II, and III fractures generally can be treated with immobilization only. Types IV and V involve the cartilage of both the articular surface and the growth plate. To ensure proper alignment and a congruous joint surfaces open reduction and internal fixation is usually necessary.

A patient presents to your family practice office with classic renal colic with his pain being a 6 on a 1 to 10 scale, with 10 being the worst pain imaginable. You refer your patient for a stat computed tomography (CT) scan of the kidneys. Your radiologist calls and advises that the CT reveals that your patient has a 4 mm stone at the ureterovesical junction (UVJ) on the left side without evidence of hydronephrosis. What should you advise your patient? A. Go directly to the emergency department for admission. B. Go directly to the emergency department for hydration and pain management. C. Return to the office, administer ketorolac IM, give a prescription for an opioid, increase fluids, strain their urine, go to the emergency department if the pain worsens, and return in 48 hours for a reevaluation. D. Go home, take four Motrin IB tablets every eight hours for pain, and go to the emergency department if the pain worsens. E Refer the patient for immediate lithotripsy.

The Correct Answer is: C Nonsteroidal anti-inflammatory drugs (NSAIDs) in general, and ketoralac specifically more so than other, causes constriction of the renal afferent arteriole, reducing pressure on a kidney stone and providing significant pain relief. Generally speaking a stone of less than 5 mm, especially at the UVJ, will pass relatively rapidly (within 48 hours) and the patient only needs to strain their urine to collect the stone and receive analgesia.

A 61-year old female patient who is a smoker is undergoing a routine physical examination in your family practice. She is otherwise asymptomatic but her urinalysis reveals microscopic hematuria. Your next definitive step would include which of the following? A. Refer her for a spiral CT scan of the kidneys to ascertain where her renal calculi may be and to rule out hydronephrosis. B. Repeat her urinalysis in the morning after asking the patient to hydrate with at least eight glasses of water in the interim. C. Send her urine for cytology and refer her to a urologist to rule out bladder cancer. D. Send her urine for a C&S and depending upon the result, start her on antibiotics E. Start a course of levofloxacin to resolve her occult urinary tract infection.

The Correct Answer is: C Painless hematuria must always include bladder cancer in the differential diagnosis. Without another reasonable explanation—something that is not the case in this patient—bladder cancer must be ruled out, beginning with cytology and subsequent referral to an urologist.

A 18 year-old man presents to the clinic with a lifelong history of transfusion dependent anemia and iron chelation therapy. Which of the following hemoglobin electrophoresis findings is most likely present? A. Decreased HbA2 and increased HbF levels B. Decreased HbA2 and HbF levels C. Increased HbA2 and normal HbF levels D. Increased HbA2 and decreased HbF levels E. Normal HbA2 and HbF levels

The Correct Answer is: C The patient history is most consistent with beta thalassemia major that presents with increased HbA 2 and/or HbF levels. Alpha thalassemia is typically a less severe anemia and presents with a normal hemoglobin electrophoresis (E), since all adult hemoglobin are alpha-containing and affected equally. Beta thalassemia minor also typically presents with increased HbA 2 (C) and can typically be distinguished from beta thalassemia major by the clinical less sever clinical presentation.

A 64-year-old postmenopausal female presents for an annual examination. She is 5'0" and weighs 92 pounds; compared to her examination 3 years ago she has lost an inch in height. After performing a dual-energy x-ray absorptiometry (DXA) scan the diagnosis of osteoporosis is confirmed and she is started on appropriate treatment. How often should she be recommended to follow-up with a DXA bone density scan? A. Every six months B. Every year C. Every two years D. Every ten years E. No need to follow-up

The Correct Answer is: C Recommendations for patients diagnosed with osteoporosis without a fracture is every 2-3 years (C). This guideline is approved based on age, risk factors, or previous fractures. A post-menopausal woman at age 64 without a presenting fracture can be followed every 2-3 years with bone density imaging (DXA scan). Medications and other treatment options may need earlier or more frequent follow-up depending on the treatment plan. DXA is quite accurate and delivers negligible radiation. Performing a DXA at intervals less than two years has not shown of benefit (A and B). Every 10 years is too far of a time frame to follow osteoporosis (D). Follow-up is needed to see any changes in the bone density (E).

A 16-year-old girl is referred for a sports physical. Her blood pressure is 170/92 mm Hg. Urinalysis (UA) reveals 2+ protein. The girl's mother reports multiple episodes of urinary tract infections (UTIs) throughout childhood that were never investigated. The most likely diagnosis is A. obstructive uropathy B. orthostatic proteinuria C. chronic reflux nephropathy D. nephrotic syndrome E. exercise-induced proteinuria

The Correct Answer is: C Retrograde flow of urine from the bladder damages the renal interstitium, causing inflammation and fibrosis. If untreated, irreversible damage to the kidneys will occur. Because this is a tubulointerstitial process, the urinalysis will be negative for protein in the early stages of damage. Most damage is done before age 5, but if undetected, glomerular damage will occur and protein will appear in the urine eventually. Hypertension develops as the GFR decreases.

A 23 year-old woman presents to the clinic complaining of abrupt onset of severe pallor, fatigue and dyspnea on exertion. Initial CBC reveals a hemoglobin of 9.6 mg/dL and an MCV of 87 fL. Which of the following best describes the cause of the abnormality pictured in the the patient's peripheral smear? (note photo taken from figure 29-13 in Harrison's) A. Absent or non-functioning spleen B. Failure of nuclear maturation C. Intravascular hemolysis D. Lead intoxication E. Presence of uremia

The Correct Answer is: C Schizocytes are present on the peripheral smear and result from intravascular hemolysis. Howell-Jolly bodies result from the lack of removal of nuclear material due to an absent or non-functioning spleen (A). Failure of nuclear maturation (B) produces macrocytosis, lead intoxication (D) and thalassemia cause basophilic stippling, and uremia (E) is associated with Burr cells.

A 14-year-old boy is brought to your primary care office as a new patient. He is new to the area and having a little bit of difficulty adjusting to school. He denies feeling depressed or being bullied. You wish to screen him for suicidality. Which is the best first question to ask? A. "Do you have a gun at home?" B. "Do you have friends who have committed suicide?" C. "Do you sometimes feel life is not worth living?" D. "Do you sometimes want to jump off a bridge" E. "Have you considered what your death might do to those around you?"

The Correct Answer is: C Suicide risk should be routinely assessed. This is best done by asking a general question such as in (C), then following up with more specific questions about means, such as (A), or plans, such as (D). Having friends who have committed suicide (B) may increase risk, but is not helpful as a screening question. Questions should focus on the patient, not on family or friends as in (E).

A 43 year-old woman with a past medical history of HIV infection presents to the clinic due to a 3-month history of dry, non-productive cough and progressive dyspnea. The patient is subsequently prescribed trimethoprim-sulfamethazole. What is the mechanism of action of this sulfamethazole? A. Binds to the 30S ribosomal subunit to inhibit protein synthesis B. Binds to the 50S ribosomal subunit to inhibit protein synthesis C. Inhibits dihydropteroate synthase and folate production D. Inhibits DNA replication by binding to DNA gyrase and topoisomerase IV E. Inhibits the transpeptidation reaction

The Correct Answer is: C Sulfamethoxazole inhibits dihydropteroate synthase and folate production (C). Tetracycline binds to the 30S ribosomal subunit to inhibit protein synthesis (A). Macrolide antibiotics bind to the 50S ribosomal subunit to inhibit protein synthesis (B). Ciprofloxacin inhibits DNA replication by binding to DNA gyrase and topoisomerase IV (D). Beta lactam antibiotics inhibit the transpeptidation reaction leading to cell wall destruction (E).

Radiculopathy due to nerve root compression occurs most commonly at which nerve root within the brachial plexus? A. C5 B. C6 C. C7 D. C8 E. T1

The Correct Answer is: C The C7 nerve root is affected the most often (approximately 45-60%). This radiculopathy can result from foraminal encroachment of the spinal nerve, cervical disk herniation, tumor, and multiple sclerosis. C7 radiculopathy can present with weakness in the triceps, which cause elbow extension, and finger flexion and extension. C6 is another common site of radiculopathy. C6 radiculopathy can present with weakness in the biceps, brachioradialis, and wrist extensor muscles. Cervical radiculopathy at the C5, C8, and T1 are less common, but still possible. C5 radiculopathy can present with deltoid and biceps muscle weakness. C8 radiculopathy can present with finger flexor weakness and T1 radiculopathy with finger abduction weakness

A 78-year-old obese male smoker presents with a gradual progression of fatigue and pallor over the last few months. Initial CBC results show a hemoglobin of 10.4 mg/dL, hematocrit of 32%, an MCV of 74 fL, and a reticulocyte count of 0.1%. Iron studies reveal an elevated ferritin, decreased serum iron and TIBC. Which of the following is the most likely cause of this patient's anemia? A. Alpha thalassemia minor B. Beta thalassemia C. Anemia of chronic disease D. Hemolytic anemia E. Iron deficiency anemia

The Correct Answer is: C The patient most likely has an underlying chronic disease resulting in anemia, as evidenced by his adequate iron stores and inability to properly use that iron for RBC production. Iron studies in patients with thalassemia (A and B) are typically normal. Iron deficiency anemia (E) is confirmed with low ferritin, serum iron, and increased TIBC.

An otherwise healthy 48-year-old male patient presents to your family practice clinic for a complete physical examination. He takes no medications. His vital signs, CBC, CMP, and TSH are normal. His fasting lipid panel reveals a total cholesterol of 280 mg/dL, LDL-C of 190 mg/dL. HDL-C of 38 mg/dL, and triglycerides of 151 mg/dL. What is your next step in his management? A. Counsel him on target life style changes and recheck his lipid panel in three months and begin therapy if not to goal at that time. B. Redraw his fasting lipid panel today, counsel him on target life style changes and begin therapy immediately if his follow up lipid panel is not to goal. C. Counsel him on target life style changes, begin therapy with a statin at this time, and recheck his lipid panel in three months and adjust therapy. D. Counsel him on target life style changes, begin therapy with a nicotinic acid derivative at this time, and recheck his lipid panel in three months and adjust therapy. E. Counsel him on target life style changes, begin therapy with a fenofibrate at this time, and recheck his lipid panel in three months and adjust therapy

The Correct Answer is: C The National Cholesterol Education Program (NCEP) Adult Treatment Panel III (ATP-III) and subsequent revisions generally recommends that healthy male patients reach the following goals for cholesterol: low-density lipoprotein (LDL) of less than 100 mg/dL, high-density lipoprotein (HDL) of greater than 40 mg/dl in men and greater than 45 mg/dL in women, and triglycerides of less than 150 mg/dL. While counseling this patient on lifestyle changes is also important, it would be almost impossible for this patient to reduce his levels to goal without medication. A better strategy would be to combine lifestyle changes and initiate medication concurrently. Statins remain the first-line therapy for treatment of dyslipidemias.

A 66-year-old female is admitted to the hospital with a new onset of atrial fibrillation. Her past medical history has hypertension, diabetes mellitus type II, hypercholesterolemia, and rheumatoid arthritis. Her vital signs show a blood pressure of 136/78, pulse of 89, respirations 18, and oxygen saturation of 96%. Her lungs are clear to auscultation, and an irregularly irregular rhythm is appreciated. There is no edema on peripheral examination. Given this clinical scenario, what is the most appropriate test in determining if an intracardiac thrombus is present? A- Transthoracic echocardiography B- Electrocardiogram C -Transesophageal echocardiography D- Cardiac catheterization E- Magnetic resonance arteriography

The Correct Answer is: C The best and most appropriate test to evaluate for an intra-atrial thrombus is the transesophageal echocardiogram (C). This test has the better sensitivity and specificity when compared to the transthoracic echocardiogram (A). Electrocardiograms (B), cardiac catheterizations (D), and magnetic resonance imaging (E) do not give any specific information related to the evaluation of an intracardiac thrombus.

A 33-year-old woman presents with an itchy vaginal discharge for the past 2 days. She has been healthy other than a recent sinus infection for which she took a 10-day course of amoxicillin. Her husband is her only sexual partner and he has no symptoms. On examination, the vulva is noted to be slightly erythematous and swollen with some evidence of excoriation. Discharge is white and clumpy. Provided the most likely diagnosis is confirmed on microscopy, first-line therapy is A. metronidazole 500 mg i po bid for 1 week B. metronidazole 500 mg 4 tablets po at HS × 1 night C. fluconazole 150 mg i po × 1 day D. rocephin 250 mg IM × 1 dose

The Correct Answer is: C The clinical presentation is consistent with vulvovaginal candidiasis. The recent oral antibiotic use increased her risk for developing the infection. The white clumpy discharge and relatively benign bimanual examination support the diagnosis, which is confirmed by 10% potassium hydroxide wet mount of the secretions. Treatment for an uncomplicated case may include topical or oral antifungals. Oral fluconazole in the one dose regimen is effective, convenient, and likely to increase compliance. The metronidazole regimens are appropriate for bacterial vaginosis and trichomoniasis, respectively. Rocephin is an option for gonococcal infection and would likely worsen the candidiasis. (Nyirjesy, 2008, p. 642) Nyirjesy P. Vulvovaginal candidiasis and bacterial vaginosis.

A 32-year-old female professional golfer presents with a 3-week history of pain along her thumb and down her wrist. She denies any trauma and states that it is aggravated with any movement of her wrist and thumb. Her physical examination is unremarkable. Considering the suspected diagnosis you perform the following examination: This test requires her to cup her thumb in a closed fist and ulnar deviate, which reproduces her pain. Which test was performed? A. Apley's B. Phalen's C. Finkelstein's D. Tinel's E. Empty Can

The Correct Answer is: C The diagnosis of De Quervain tenosynovitis is supported by a history of pain in the location of the radial aspect of the wrist with a painful range of motion of the thumb and occasional local tenderness over the distal portion of the radial styloid. Further confirmation of the diagnosis may be provided by a positive Finkelstein test (C), in which the patient grasps the thumb in the palm of the hand and the examiner ulnar deviates the thumb and hand. This stretches the tendons over the radial styloid and produces sharp pain along the involved tendons. A Tinel's (D) and Phalen's (B) sign are positive in carpal tunnel syndrome. The Apley's compression test (A) is used to evaluate meniscus tears in the knee. The Empty Can Test (E) is used to assess the rotator cuff supraspinatus muscle.

A 56-year-old African American female is currently on heparin therapy for a new diagnosis of atrial fibrillation, and warfarin therapy is being started. She has a past history of hypertension, hypercholesterolemia, diabetes, and end stage renal disease. All of her chronic medical problems are stable, and the patient now has a rate controlled atrial fibrillation at 77 beats per minute. Given this clinical scenario, what is the most appropriate international normalized ratio (INR) to remain therapeutic while being administered warfarin? A. 0-1.0 B. 1.0-2.0 C. 2.0-3.0 D. 3.0-4.0 E. >4.0

The Correct Answer is: C The ideal target INR for a patient with non-valvular atrial fibrillation while on warfarin is 2.5 (C). Data suggests that anticoagulation with an INR >2.0 (D and E) not only reduces the risk of ischemic stroke, it also shows a reduction in the severity of stroke. However, higher INR levels >3.0 increase a patient's risk of side effects of warfarin, the most common being bleeding. Both choices A and B would not adequately anticoagulate the patient, and thus still put the patient at risk for development of thrombus formation.

A 24-year-old male patient will be undergoing cisplatin-based chemotherapy for treatment of his germ cell cancer. Which of the following antiemetic regimens is most effective in this situation? A- Dexamethasone B- Dronabinol (Marinol®) C- Palonosetron (Aloxi®) D- Prochlorperazine E- Dexamethasone and palonosetron

The Correct Answer is: E Combining 5-HT3 antagonist with glucocorticoids is the most effective therapy for prevention of acute chemotherapy-induced nausea and vomiting (CINV). Dronabinol (B) is ineffective in the treatment of CINV, while prochlorperazine (D) plays a limited role in pregnant patients.

A 28-year-old female presents with a gradual progression of fatigue and pallor over the last few months. Initial CBC results show a hemoglobin of 10.4 mg/dL, hematocrit of 32%, an MCV of 112 fL, and a reticulocyte count of 0.1%. Which of the following is the most likely pathophysiologic mechanism responsible for her anemia? A. Acute blood loss B. Defective bone marrow/stem cell function C. Defective DNA production D. Defective hemoglobin production E. Increased destruction of red blood cells

The Correct Answer is: C The patient most likely has an underlying vitamin B 12 or folate deficiency resulting in macrocytosis (MCV of 112 fL). Defective DNA production (C) results in failure of RBC maturation and macrocytosis (elevated MCV). Acute blood loss (A) would present more acutely and with normal hemoglobin, hematocrit, and MCV until hemodilution occurs and lowers the hemoglobin concentration and hematocrit. Defective bone marrow/stem cell function (B) tends to produce normocytic red blood cells. Microcytic, hypochromic anemia results from defective hemoglobin production (D). Hemolysis (E) is less likely than vitamin B 12 and folate deficiency, but can be confirmed or ruled out through the assessment of the reticulocyte count (increased reticulocyte will increase the MCV and be inconsistent with a nutritional deficiency) and further hematologic labs as necessary.

A 15-year-old male presents complaining of a sore throat, headache, and mild cough that started 8 days ago and has progressed to include a worsening cough and increasing fatigue. His chest x-ray reveals bilateral hilar infiltrates and a CBC is normal. Which of the following diagnostic tests will most likely confirm the suspected diagnosis? A. Acid-fast bacilli smear and culture B. Blood culture C. PCR testing of sputum D. Sputum culture E. Sputum gram stain

The Correct Answer is: C The patient most likely has atypical pneumonia secondary to mycoplasma pneumoniae which is best confirmed by PCR testing of sputum (C), oropharyngeal or nasal secretions and isn't detected through standard cultures (B, D) or staining techniques (A, E).

A 22 year-old male presents with a 2-week history of an upper respiratory infection that hasn't improved after taking amoxicillin for 6 days. He notes persistent sore throat, intermittent fever, and a worsening nonproductive cough. Physical examination reveals bilateral diffuse crackles. What is the most likely causative agent of this patient's symptoms? A. Streptococcal pneumoniae B. Klebsiella pneumoniae C. Mycoplasma pneumoniae D. Pneumocystis jiroveci E. Staphlococcal aureus

The Correct Answer is: C The patient presentation is consistent with atypical pneumonia most commonly caused by viruses, mycoplasma pneumonia (C), chlamydia pneumonia, or legionella. The presentation of streptococcal pneumoniae (A) is more acute with productive cough and fever being early symptoms. Klebsiella pneumoniae (B) also presents with fever and a cough classically noted to be productive of "currant jelly" sputum. Pneumocystis jiroveci (D) causes pneumonia of insidious onset in immunocompromised patients. Staphlococcal (E) pneumonia also presents more acutely with productive cough.

A 68-year-old man presents to the outpatient clinic complaining of decreased hearing in his left ear. The following is seen on otoscopic evaluation. Which of the following is the most likely diagnosis? A. Acute otitis media B. Cerumen impaction C. Cholesteatoma D. Chronic otitis media E. Tympanic membrane perforation

The Correct Answer is: C The photo depicts a classic cholesteatoma (C) effecting the pars flaccida. The localized nature of the findings and lack of inflammation make otitis media (A and D) unlikely. Any perforation in the TM (E) would be secondary to the cholesteatoma and the visible TM is inconsistent with a diagnosis of impacted cerumen (B).

A 24-year-old female comes into the clinic complaining of a severe sore throat. She was seen three days ago at an urgent care facility, and was given amoxicillin. She states that the pain is worse, she is unable to drink fluids, and is now having difficulty swallowing. She talks with a muffled voice. A physical exam reveals a markedly swollen and erythemic right tonsil and tonsillar pillar, with the uvula deviating to the left. The patient has extreme difficulty opening her mouth. What is the most likely diagnosis? A. Tonsillitis B. Uvulitis C. Peritonsillar abcess D. Tonsillar cellulitis E. Diphtheria

The Correct Answer is: C The physical exam is highly suspicious for peritonsillar abcess, which must be considered first. Tonisllar cellulitis, or phlegmon, is swelling and enlargement of the tonsil and peritonsillar tissue, without the presence of fluctuant abcess. Uvulitis can exist with a peritonsillar abcess or tonsillitis, but isolated uvulitis usually includes symmetric swelling and erythema as a result of irritation (snoring), allergy (angioedema), or infection from upper respiratory pathogens. Diphtheria is a tonsillitis, with a characteristic gray pseudomembrane on the tonsils and upper airway, caused by corneybacterium diphtheriae.

A 51-year-old female presents to her primary care provider for her annual physical. She is a healthy white female and a non-smoker. She has mild HTN, but an otherwise negative health Hx. In addition, FHx is negative and ROS is negative. Her LMP was 6 months ago. Her last mammogram was 3 years ago. When should she have her next mammogram? A. In 2 years B. In 1 year C. Now D. Only if self breast exam reveals abnormality E. Only if provider breast exam reveals abnormality

The Correct Answer is: C The preponderance of data strongly supports the benefits of a screening mammography. New analyses of older randomized studies have suggested that screening may not work. While the design defects in some older studies cannot be retrospectively corrected, most experts, including panels of the American Society of Clinical Oncology and the American Cancer Society, continue to believe that screening conveys substantial benefit.

Upon testing a patient for function of the hip abductors, which muscle is considered the primary muscle responsible for most abduction? A. Biceps femoris B. Gluteus maximus C. Gluteus medius D. Gluteus minimus E. Vastus medialis

The Correct Answer is: C The primary mover in the motion of hip abduction is the gluteus medius muscle. Gluteus minimus does play a supporting role in that motion. Biceps femoris is one of the three hamstring muscles and contributes to the motions of knee flexion (primary muscle) and hip extension (secondary muscle). Gluteus maximus is the primary mover for hip extension and vastus medialis is one of the four quadriceps muscles responsible for knee extension, but no hip movements.

A 78-year-old female trips and falls on an outstretched hand and now presents with right wrist pain. On examination, the patient complains of pain on palpation to the right radial side of the wrist near the anatomical snuffbox. Based on this clinical presentation, what is the most likely fracture that this patient has? A. Capitate B. Lunate C. Scaphoid D. Trapezium E. Trapezoid

The Correct Answer is: C The scaphoid or navicular bone of the wrist, is the most commonly fractured carpal bone. This injury occurs most commonly in young men. The lunate is the second most commonly fractured carpal bone and the most commonly dislocated carpal bone. The capitate is the largest of the carpal bones and it is not known as a common isolated fracture site. Fractures of the trapezium and trapezoid are relatively uncommon as well.

Which rotator cuff muscle is most commonly injured? A. Infraspinatus B. Subscapularis C. Supraspinatus D. Teres Major E. Teres Minor

The Correct Answer is: C The supraspinatus is involved in abduction and external rotation of the shoulder joint. It is often injured in repetitive overhead activities such as swimming and throwing sports. While all muscles of the rotator cuff can be injured, the supraspinatus has been shown to sustain the most frequent injuries. The infraspinatus and teres minor are also involved in abduction and external rotation, while the subscapularis and teres major assist in internal rotation. The teres major is not considered a rotator cuff muscle.

A 67-year-old, 220-pound female presents to your family practice office for a presurgical clearance examination for a total hip replacement in one week. Past medical history includes hypertension, atrial fibrillation, a prior percutaneous transluminal coronary angioplasty (PTCA) with drug-eluting stent seven months ago, and type 2 diabetes mellitus . Her medication include: lisinopril, metoprolol ER, a baby aspirin, clopidogrel, and metformin. Her blood pressure is 128/78 mm&thinsp;Hg, pulse is 72 beats/min, temperature 98.6˚F, and respirations of 17 breaths/min. Her heart and lung sounds are normal. The abdominal examination is benign. Her x-ray and EKG are normal, as are all required labs. Which of the following is recommended for this patient prior to her surgery? A. Discontinue metoprolol one week prior to surgery. B. Discontinue metformin one week prior to surgery. C. Discontinue the aspirin and clopidogrel five to seven days prior to surgery. D. Reschedule the surgery until patient has reduced their weight by 7%.

The Correct Answer is: C The timing and consideration of discontinuing anti-platelet agents is complicated. Generally speaking, for minor procedures (such as a tooth extraction) in patients with risk, discontinuation is not necessary. But for major and/or bloody procedures, such as a total joint replacement, discontinuation is necessary. When considering major surgeries, most anti-platelet therapies should be stopped five to seven days prior to the surgery. Consultation with the patient's cardiologist may be beneficial.

A 56-year-old male was admitted to the regular medical floor with a diagnosis of sepsis due to urinary tract infection. While on the floor he became acutely unstable and had a blood pressure of 70/40. A medical emergency code was called, which revealed the rhythm shown. There was no palpable pulse on exam, and no evidence of heart sounds on auscultation. The patient was not responding to any stimulus during this time, and there did not appear to be any spontaneous respirations noted. Based on this clinical scenario, what would be the most appropriate medical procedure to perform on this patient? A. Emergent cardiac catheterization B. Synchronized cardioversion C. Unsynchronized cardioversion D. Temporary pacemaker insertion E. No intervention, the patient has not survived

The Correct Answer is: C This patient is exhibiting ventricular fibrillation with hypotension. Because this patient is unstable in this clinical setting, immediate unsynchronized cardioversion (C) is warranted. Once corrected, consideration for cardiac catheterization (A) is reasonable since patients may have native coronary disease that caused the fibrillation. Unsynchronized cardioversion (C) would not initiate a cardiac response, and temporary pacemaker (D) must first have an established electrical rhythm to appropriately pace the patient. The patient still has a low blood pressure, therefore no intervention (E) would prove to be fatal.

You are treating a 60-year-old man with a history of angina. He has been on the therapeutic lifestyle change (TLC) diet for 12 weeks (with solid effort). This patient has no other medical conditions and takes nitroglycerin as needed and daily enteric-coated aspirin. His fasting lipid panel from last week demonstrates the following: What is the most appropriate treatment at this time? A. prescribe colestipol B. prescribe ezetimibe C. prescribe simvastatin D. prescribe niacin E. no pharmacological treatment

The Correct Answer is: C This patient's coronary heart disease risk factors and failed TLC diet warrant pharmacological treatment based on his LDL level. Although there are no absolute guidelines for the selection of lipid-modifying medications, an HMG-CoA reductase inhibitor is preferred.

A 22-year-old man (refer to Figure 4-2) is being evaluated for extremity enlargement unlike anyone in his family. Over the past 2 years, he has noticed that his rings no longer fit and his feet are so wide that he cannot find shoes to fit. He has always been tall for his age, greater than the 95th percentile throughout his teenage years. He has very coarse facial features, macroglossia, and a very deep voice. What is the most likely cause of this patient's condition? A. adrenal neoplasm B. multinodular goiter C. pituitary macroadenoma D. Rathke cleft cyst E. testicular neoplasm

The Correct Answer is: C This patient's signs and symptoms are consistent with acromegaly, which is caused by an increased secretion of GH. These are almost always caused by pituitary macroadenomas. The tumors may be locally invasive into the cavernous sinus but are typically not malignant.

A 73-year-old man is brought into your office by his adult children with a concern of memory loss. They report their father's memory has been declining since the death of their mother a few months ago but are now concerned because he is losing weight, sleeping during the daytime, and is not keeping up with current events like he usually does. This type of behavior is most associated with which of the following? A. Pick disease B. Creutzfeldt-Jakob disease C. depression D. Alzheimer disease E. vitamin B12 deficiency

The Correct Answer is: C This patient's symptoms are most consistent with situational depression over the loss of his spouse. Transient memory problems can be a component of depression as a result of decreased attention and interest. Dementia is a progressive impairment of higher cognitive function, and initially, the patient's social graces are preserved. It has many causes, of which Pick disease, Creutzfeldt-Jakob disease, and Alzheimer disease are irreversible. Vitamin B 12 deficiency can cause reversible form of cognitive impairment, in which the elderly are susceptible, so serum analysis of vitamin B 12 should be performed in diagnostic evaluations of dementia in this population.

Your patient is a 42-year-old female diagnosed with depression. She has taken two different serotonin reuptake inhibitors (SSRIs) at the usual dosage, each for 3 weeks, and reports again that there have been no ill effects but that she doesn't feel that this medication has helped either. What is the most likely problem with her therapy? A. She has a genetic resistance to the medication B. The dosage was too low C. The medication was discontinued too soon D. The wrong diagnosis was made E. The wrong medication was given

The Correct Answer is: C Treatments for depression generally require from 2 to 6 weeks of therapy in before effects can be evaluated (C). Too low a dose (B) is another common problem, but an increase in dose should not be considered until the medication has had time to take effect. The effect of these agents can be highly variable from one individual to another, but no specific genetic resistance (A) is known. There is always a possibility of making the wrong diagnosis (D), but there is no indication of this from the information given, and SSRIs are the first line therapy for depression (E), best combined with counseling.

A 52-year-old male bus driver presents to the clinic with a chief complaint of intense, shooting pains in his left cheek, each lasting for only a few seconds. He avoids touching certain parts of his face and has started to chew food only on the right side of his mouth because he is afraid he will set off an attack of pain. In between attacks, the patient feels well. What is the most likely diagnosis? A. cluster headache B. tension-type headache C. trigeminal neuralgia D. giant cell arteritis E. dental abscess

The Correct Answer is: C Trigeminal neuralgia is characterized by sharp, brief pain often described as "shooting, jabbing, electric shock, or stabbing." The history given for cluster headache (typically ipsilateral ocular headaches with tearing, and lasting for 2 hours) and tension-type headache is not at all like this patient's. The history for temporal arteritis is generally different, as it typically includes ocular symptoms, but it may be worth getting a sedimentation rate just to be sure. This location pain pattern is different than that of a focal dental problem.

Q 4.9: A 14-year-old female patient presents to your family practice clinic having received a 1% total body surface area first and second degree burn to the left forearm. Of the following, what would you recommend for your patient? A. Deroof any blisters, apply bacitracin topically, and prescribe pain medication, with follow-up in 48 hours, B. Deroof any blisters, apply silver sulfadiazine topically, and prescribe pain medication, with follow-up in 48 hours. C. Leave any blisters intact, apply bacitracin topically, and prescribe pain medication, with follow-up in 48 hours. D. Leave any blisters intact, apply silver sulfadiazine topically, and prescribe pain medication, with follow-up in 48 hours, E. Refer to the emergency department immediately.

The Correct Answer is: C Unless a critical surface (face, genitalia or hands) is involved, first and second degree burns may be treated in the outpatient setting. Blisters may be left intact as a physiologic dressing, and deroofed after they rupture. The patient requires tetanus prophylaxis and a topical antibiotic cream, usually either silver sulfadiazine or, preferably, bacitracin. Sulfadiazine may permanently stain skin, so use it cautiously in potentially exposed skin areas for cosmetic reasons.

A 66-year-old female patient presents to your family practice clinic for a complete physical exam. She is a smoker and you counsel her at length regarding smoking cessation. She is a well- controlled hypertensive taking lisinopril, along with an 81 mg enteric aspirin. You review her testing which reveals the following: • Comprehensive metabolic panel is normal. • Complete blood count is normal. • Urinalysis is normal, except for trace blood. • Fasting lipids are to goal. • Immunochemical fecal occult blood testing (IFOBT) is negative for occult blood. Of the following, what is a next appropriate step to take? A. Schedule a colonoscopy to rule out colon cancer. B. Refer her to pulmonary medicine for pulmonary function testing. C. Refer her to a urologist to rule out bladder and renal cell carcinoma. D. Stop her aspirin. E. Send out her urine for culture and sensitivity, and in the interim start her on an empiric antibiotic.

The Correct Answer is: C Unless the patient has another reason to have hematuria, consideration should be made of the possibility of bladder or renal cell cancers, especially in a smoker. After ruling out other causes, the standard of care is to refer this patient to a urologist to rule out these potentially life threatening cancers.

A 49-year-old male presents to the clinic with symptoms of nausea, occasional vomiting, vague epigastric pain, fatigue, and weight loss of 35 lbs. over the past few months. On exam you find a palpable abdominal mass. Which of the following diagnostic tests is the best initial test to obtain? A- Barium upper GI series B- Abdominal CT C- Upper endoscopy D- PET scan E- Flexible sigmoidoscopy

The Correct Answer is: C Upper endoscopy allows for a biopsy, which is highly sensitive in detecting gastric carcinoma, the suspected diagnosis. An upper GI series cannot distinguish a benign from a malignant lesion. An abdominal CT would be most useful once gastric carcinoma has been diagnosed, to help in preoperative evaluation. A PET scan would be most useful in detecting metastasis. A flexible sigmoidoscopy would not allow for visualization of the stomach.

What types of connective tissue are injured in a strain? A. Bones and muscles B. Fascia and joint capsules C. Ligaments and joint capsules D. Muscles and tendons E. Tendons and bones

The Correct Answer is: D A strain involves injury to the muscles and tendons that are responsible for active movement of various body parts. Fascia is a part of the muscle-tendon unit, so injury to fascia would be considered a strain as well. Injury to ligaments and joint capsules would be considered a sprain and damage to bone would be classified as a fracture.

An avulsion fracture at the base of the fifth metatarsal is commonly called which of the following? A. Bennett fracture B. Boxer's fracture C. Chauffer's fracture D. Jones fracture E. Lisfranc fracture

The Correct Answer is: D An avulsion fracture at the base of the fifth metatarsal, usually secondary to plantar flexion and inversion is called a Jones fracture. Also called a ballet or dancer's fracture, it is the most common metatarsal fracture. The fracture occurs at the proximal diaphysis. A Bennett fracture is an oblique fracture of the first metacarpal near the carpometacarpal joint. A boxer's fracture is a fracture of the fifth metacarpal. This is the most common fracture of the hand. A chauffer's fracture is an oblique fracture through the base of the radial styloid in the forearm. A Lisfranc fracture is actually a fracture and dislocation involving the tarsometatarsal joints.

An 8-month-old male presents to the clinic due to irritability, fatigue and parental concerns about developmental delays. He is fifth percentile for weight and 33rd percentile for height. An office based hemoglobin level is 8.4 mg/dL. The infant is prescribed iron supplementation, to begin today. Which of the following tests should be performed in 5-7 days to confirm response and adherence to iron supplementation? A. Ferritin B. Hemoglobin C. Hematocrit D. Reticulocyte count E. Serum iron

The Correct Answer is: D An elevated reticulocyte count 1 week after initiation of iron supplementation confirms the presence of IDA and efficacy of therapy. The hemoglobin (B), hematocrit (C), serum iron (E) and ferritin (A) will improve subsequently to the increase in the reticulocyte count. After one month of iron supplementation the patient's hemoglobin should increase by 1-2 mg/dL and the hematocrit should increase by 3-6%.

A 16-year-old male soccer player is complaining of pain to the right foot that has been getting progressively worse for the last 2 months. He states it hurts the most when he has all of his weight on his right foot as he plants to kick the ball. Most of the pain appears to be on weight bearing. You are concerned that this patient may be developing a stress fracture. Based on the patient's history and patient presentations, which bone is the most affected by stress fractures in the foot? A. Calcaneus B. Fifth metatarsal C. First metatarsal D. Second metatarsal E. Talus

The Correct Answer is: D Any bone that is exposed to repetitive stress can have a stress fracture, but the long and thin metatarsal bones of the foot are the most commonly affected bones. Of the metatarsals, the second metatarsal has the highest number of stress fractures. These weight bearing bones can be particularly vulnerable to stress fracture if the patient is involved in long distance running, especially if he/she is wearing improper footwear for that activity or footwear that has lost most of its shock absorbing abilities. Some young female athletes may be training so hard that they become amenorrheic which can contribute to osteopenia resulting in weaker bones. Older patients with osteoporosis will also have a higher risk of stress fracture. Initially stress fractures of the metatarsals may present with a small area of localized pain and the dorsal forefoot may demonstrate a fairly diffuse area of swelling. If the stress fracture is not treated early, some patients will experience an audible pop or crack as the incomplete stress fractures progresses to a complete break. All types of fractures occur more easily in long thin bones like the metatarsals, than thicker bones like the calcaneus and talus.

A 3 year-old male presents to the clinic with lethargy and fatigue. An initial CBC reveals a hemoglobin of 10.1 mg/dL and an MCV of 72 fL. Peripheral smear results are shown below. Which of the following best describes the cause of the abnormality pictured in the patient's peripheral smear? (note photo taken from figure 29-12 in Harrison's) A. Absent or non-functioning spleen B. Failure of nuclear maturation C. Intravascular hemolysis D. Lead intoxication E. Presence of uremia

The Correct Answer is: D Basophilic stippling is noted in the peripheral smear and is associated with lead intoxication (D) and thalassemia. Howell-Jolly bodies result from the lack of removal of nuclear material due to an absent or non-functioning spleen (A). Failure of nuclear maturation (B) produces macrocytosis, intravascular hemolysis (C) leads to the creation of schizocytes, and uremia (E) is associated with Burr cells.

What nerve is most commonly injured in a mid- or distal humeral shaft fracture? A. Axillary B. Median C. Peroneal D. Radial E. Ulnar

The Correct Answer is: D Because of the radial nerves proximity to the humerus, mid and distal shaft fractures with significant displacement can cause a radial nerve injury. Median and ulnar injuries are more commonly associated with forearm injuries. Axillary nerve injuries are most common in anterior shoulder dislocations and peroneal nerve damage occurs as a result of lower leg insult.

A 43 year-old man presents to the clinic seeking counseling and medication to help quit smoking. Which of the following factors would be a contraindication to prescribing him bupropion? A. Allergy to sulfa B. Depression C. Decreased libido D. Seizure disorder E. Use of nicotine replacement therapy

The Correct Answer is: D Bupropion is contraindicated in situations where patients are at risk of seizures (D). Bupropion is indicated for the treatment of depression (B) and doesn't typically cause libido concerns (C). Bupropion is commonly prescribed with nicotine replacement therapy (E) during tobacco cessation efforts.

Your 36-year-old obese female patient complained of a single episode of right upper quadrant pain after eating fast food (a double cheeseburger, fries, and a vanilla shake) last week. The symptoms have resolved and have not reoccurred when she presents to your family practice. Your psychical examination is completely normal, as is blood work, including a complete blood count, comprehensive metabolic panel, acute viral hepatitis panel, amylase, and lipase. You suspect cholecystitis and order an abdominal sonogram. The radiologist makes note that the gall bladder appears thickened with an associated calcification with a single, large gallstone. Of the following, what is the most appropriate next step? A. Counsel the patient on diet, exercise, and lifestyle changes and revisit the issue should symptoms reoccur. B. Refer the patient for an abdominal computed tomography (CT) scan. C. Refer the patient to a gastroenterologist. D. Refer the patient to a surgeon for a laparoscopic cholecystectomy due to the risk of gall bladder cancer. E. Re-evaluate the patient for symptomatology in four to six weeks.

The Correct Answer is: D Calcification of the gall bladder, frequently referred to as a porcelain gall bladder, and single, solitary, large, gall stone are a common presentation of gall bladder cancer on imaging. Especially in the face of a symptomatic patient, cholecystectomy with an associated biopsy should be a strong consideration.

Chronic NSAID use is a known cause of peptic ulcer disease. Which of the following NSAIDs is the least likely to lead to ulcer formation? A. Aspirin B. Ibuprofen C. Naproxen D. Celecoxib E. Diclofenac

The Correct Answer is: D Celecoxib is the only selective NSAID listed in the choices. Coxibs preferentially inhibit COX-2, the principle enzyme involved at sites of inflammation, while sparing COX-1. COX-1 is involved in mucosal cytoprotection in the stomach and duodenum. All the other choices listed are nonselective NSAIDs, and inhibit both COX-1 and COX-2 enzymes.

Which of the following is the most common cause of hypoparathyroidism? A. Familial hypoparathyroidism B. Idiopathic hypoparathyroidism C. Severe magnesium depletion D. Surgical removal of the parathyroid E. Iron deposition in the parathyroid

The Correct Answer is: D Choice D, surgical removal of the parathyroid glands, is the correct answer. Surgery for head and neck cancer, thyroidectomy, and parathyroidectomy are the most common causes of hypoparathyroidism. Choices A, B, C, and E are all causes of hypoparathyroidism that occur more infrequently.

A 41-year-old alcoholic male, who lives primarily on the streets, appears pale, cachectic, and mildly icteric. He is complaining of several weeks of increasing fatigue. Laboratory findings note an elevated MCV of 128. What other physical finding would most support the diagnosis for megaloblastic anemia? A.Decreased vibration and position sense B.Dementia C. Difficulty with balance D. Glossitis E. Parethesias

The Correct Answer is: D Features of folate deficiency are similar to vitamin B12 deficiency. However, there are none of the neurologic abnormalities associated with vitamin B12. Glossitis is the only non-neurologic finding in the PE that would support folate deficiency. Alcoholism and poor dietary intake also support the diagnosis of folate deficiency.

A toddler became infected with Giardia lamblia at day care and his two elder siblings and both parents have also developed foul-smelling diarrheal stools. Of the following treatments, which is most appropriate for his mother, who is 24 weeks pregnant? A- Albendazole B- Furazolidone C- Metronidazole D- Paromomycin E- Tinidazole

The Correct Answer is: D First-line treatments for giardiasis are albendazole (A), metronidazole (C), and tinidazole (E). Furazolidone (B) is also effective, but causes gastrointestinal side effects. Paramomycin has lower efficacy, but is considered safe in pregnancy, unlike the others.

A 72-year-old female presents with headache, jaw claudication, scalp tenderness, and visual changes in her left eye for the past week. She also reports a 4-month history of pain and stiffness in her shoulders and hips. She identifies the pain being worse in the morning and aggravated with getting in and out of the car along with difficulty brushing her hair. She also reports malaise and a 10-pound weight loss over the past few months. Her blood work shows an erythrocyte sedimentation rate (ESR) of 74 mm/h. What is the most likely cause of her jaw claudication? A. Raynaud's phenomenon B. Systemic lupus erythematosus C. Fibromyalgia D. Giant cell arteritis E. Migraine

The Correct Answer is: D Giant cell (temporal) arteritis (D) is characterized by headache, jaw claudication, polymyalgia rheumatica, visual abnormalities, and a markedly elevated ESR. Giant cell arteritis is a systemic panarteritis affecting medium-sized and large vessels in patients over the age of 50. Giant cell arteritis is also called temporal arteritis because that artery is frequently involved. About 50% of patients with giant cell arteritis also have polymyalgia rheumatica. The classic symptoms suggesting that a patient has arteritis are headache, scalp tenderness, visual symptoms (particularly amaurosis fugax or diplopia), jaw claudication, or throat pain. Of these symptoms, jaw claudication has the highest positive predictive value. Polymyalgia rheumatica is a clinical diagnosis based on pain and stiffness of the shoulder and pelvic girdle areas, frequently in association with fever, malaise, and weight loss. Raynaud's phenomenon (A) affects the digits. SLE (B) is also an autoimmune disorder but mostly occurs in a younger patient population with different symptoms. Fibromyalgia includes multiple trigger point locations, not consistent with this clinical scenario of jaw claudication (C). A migraine (E) does not usually present with this presentation of symptoms.

A 28-year-old patient who is a fire department paramedic presents for a routine physical examination to your family practice office. They are asymptomatic, but their PPD is positive. Suddenly, they relay that they have tested positive on their PPD in the past, "about five years ago," and that they were treated at that time with nine months of INH. What should your next step be in treating them? A. Begin treatment for resistant tuberculosis (TB). B. Nothing, inasmuch as they have already received a full course of therapy. C. Order a chest x-ray and if normal, follow the patient with chest x-rays every two to three years or sooner if they become symptomatic D. Order a chest x-ray, and if normal, clear the patient cautioning them to immediately contact you should they develop any signs or symptoms consistent with TB. E. Refer the patient to ID or Pulmonary for definitive treatment of resistant TB.

The Correct Answer is: D Health care workers (HCWs) with positive PPD test results should have a chest radiograph as part of the initial evaluation of their PPD test; if negative, repeat chest radiographs are not needed unless symptoms develop that could be attributed to TB. However, more frequent monitoring for symptoms of TB may be considered for recent converters and other PPD-positive HCWs who are at increased risk for developing active TB (e.g., HIV-infected or otherwise severely immunocompromised HCWs). Regardless of whether the patient completes treatment for latent TB infection, serial or repeat chest radiographs are not indicated unless the patient develops signs or symptoms suggestive of TB disease.

An 8-month-old male presents to the clinic due to irritability, fatigue and parental concerns about developmental delays. He is fifth percentile for weight and 33rd percentile for height. An office based hemoglobin level is 8.4 mg/dL. Which of the following is the most likely diagnosis? A. Alpha thalassemia minor B. Beta thalassemia minor C. Folate deficiency D. Iron deficiency anemia E. Sideroblastic anemia

The Correct Answer is: D Infants with poor nutrition are at greatest risk of iron deficiency anemia between the ages of 6 and 24 months. Alpha thalassemia minor (A) and beta thalassemia minor (B) present with a markedly microcytic, but mild anemia. Folate deficiency (C) due to inadequate nutrition in the absence of co-morbid disease is rare. Sideroblastic anemia (E) is far less common than iron deficiency anemia and typically diagnosed based on the finding of ringed sideroblasts visible on peripheral smear.

A 35-year-old nulliparous female has had increasing and heavy flow with her menstrual cycles over the last six months. She is being followed by her gynecologist, who has diagnosed several small uterine fibroids. She presents in clinic today with complaints of tachypnea on exertion, tachycardia, and occasional palpitations. There is no family history of cardiac problems or hypertension. She denies symptoms of infectious etiology and has no fever. On physical examination, you note a mild tachycardia of 110 after she has been sitting for a few minutes. No other physical findings are noted. What would be the next most useful diagnostic study to help you confirm the diagnosis? A. CBC B. Chest x-ray C. EKG D. Serum ferritin E. Transthoracic ECHO

The Correct Answer is: D Iron deficiency develops in stages. The first is depletion of iron stores. There is no anemia or change in RBC. The serum ferritin will become abnormally low and lead to symptoms of anemia, such as easy fatigability, tachycardia, palpitations, and tachypnea on exertion. Iron deficiency anemia is more frequently seen in women, especially those with heavy menstrual cycles, as they are unable to absorb enough iron in their diet to maintain stores—leading to depletion, signs of anemia, and progression if not treated.

A 19 year-old woman presents to the clinic complaining of fever, and productive cough. Physical exam reveals a temperature of 102F, decreased breath sounds of her right middle lobe. She is prescribed levofloxacin. What is the mechanism of action of levofloxacin? A. Binds to the 30S ribosomal subunit to inhibit protein synthesis B. Binds to the 50S ribosomal subunit to inhibit protein synthesis C. Inhibits dihydropteroate synthase and folate production D. Inhibits DNA replication by binding to DNA gyrase and topoisomerase IV E. Inhibits the transpeptidation reaction

The Correct Answer is: D Levofloxacin inhibits DNA replication by binding to DNA gyrase and topoisomerase IV (D). Tetracycline binds to the 30S ribosomal subunit to inhibit protein synthesis (A). Macrolide antibiotics bind to the 50S ribosomal subunit to inhibit protein synthesis (B). Sulfamethoxazole inhibits dihydropteroate synthase and folate production (C). Beta lactam antibiotics inhibit the transpeptidation reaction leading to cell wall destruction (E).

Which physical complaint is the leading cause of lost work time and disability for patients under the age of 45? A. Ankle pain B. Hip pain C. Knee pain D. Low back pain E. Shoulder pain

The Correct Answer is: D Low back pain is the most common causes of lost time from work and disability. Factors that can lead to back problems include repeated or heavy lifting and twisting or use of vibrating equipment along with poor fitness levels, smoking, poor job satisfaction and other psychosocial factors. Fortunately, 80% of patients experience significant recovery within one month. All of the other types of pain can occur based on multiple factors, but none of them occur with a frequency that rivals low back pain.

A 68-year-old female with a past medical history of diabetes, dyslipidemia, hypertension, and gout presents with a gradual progression of fatigue and pallor over the last few months. Initial CBC results show a hemoglobin of 10.4 mg/dL, hematocrit of 32%, an MCV of 112 fL, and a reticulocyte count of 0.1%. Which of the following medications should be considered as a potential cause of her anemia? A. Allopurinol B. Atorvastatin C. Lisinopril D. Metformin E. Nicotinic acid

The Correct Answer is: D Metformin is associated with vitamin B 12 deficiency and may rarely be a medication cause for macrocytic anemia. Nicotinic acid (E) administration may help reduce the risk of vitamin B 12 deficiency secondary to nicotinic acid deficiency. Allopurinol (A), atorvastatin (B), and lisinopril (C) are not strongly associated with vitamin B 12 or folate deficiency.

A 63 year-old woman developed severe thrombocytopenia after undergoing her first treatment of a cisplatin-based chemotherapy regimen. Which of the following agents may be administered to prevent thrombocytopenia if the patient continues her cisplatin-based regimen? A- Darbepoetin alfa (DARB) B-Erythropoietin (EPO) C-Filgrastim (G-CSF) D-Oprelvekin (IL-11) E- Sargramostim (GM-CSF)

The Correct Answer is: D Oprelvekin, or recombinant interleukin-11, is a megakaryocyte growth factor that promotes platelet production. Erythropoietin (B) and darbepoetin alfa (A) are both stimulators of red blood cell production, while filgrastim (C) and sargramostim (E) stimulate granulocyte production.

An 8-month-old male presents to the clinic due to irritability, fatigue and parental concerns about developmental delays. He is fifth percentile for weight and 33rd percentile for height. An office based hemoglobin level is 8.4 mg/dL. Which of the following is the most common side effect the infant may experience after starting oral iron supplementation therapy? A. Anaphylaxis B. Clay-colored stools C. Flushing D. GI upset E. Melena

The Correct Answer is: D Oral iron commonly causes GI upset. Anaphylaxis (A) commonly occurs with parenteral iron therapy. Stools will darken and not be clay colored (B), which occurs in the absence of bile production. Flushing (C) is a common side effect of niacin. Dark stools will test falsely positive for blood on hemoccult tests and not be true melena (E) with blood present in the stool.

A 17-year-old female distance runner with no significant PMH complains that she has diffuse, aching anterior knee pain that is worsened when she walks up or down stairs or when she squats down. There has been no acute trauma, but she has been increasing her running mileage. No effusion is present. What is the probable diagnosis? A. Anterior cruciate ligament tear B. Medial meniscal tear C. Osteoarthritis of the knee joint D. Patellofemoral syndrome E. Posterior cruciate ligament tear

The Correct Answer is: D Patellofemoral syndrome is a common condition in active adolescents due to repetitive stresses on the patellofemoral joint. This can be exacerbated by altered patellofemoral tracking due to growth and development in adolescents. Other biomechanical issues that may contribute to this problem include poor flexibility, weakness of the vastus medialis muscle, which results in a more lateral tracking of the patella, and excessive foot pronation. This condition does not produce an effusion. Anterior and posterior ligament tears would be highly unlikely given that there was no history of an acutely traumatic event and because ligaments are fairly vascular structures, an effusion would be expected with any cruciate ligament tear. Meniscal tears in a young patient are generally associated with a traumatic twisting event and while not as vascular as a ligament, meniscal tears generally lead to the gradual development of an effusion. Older patients may experience a degenerative tear with minimal trauma, but that is unlikely in our 17-year-old patient. Osteoarthritis is generally associated with gradual wear and tear after many years or the process can be accelerated after a significant acute trauma to a joint. Neither scenario is applicable to this patient.

A 33 year-old man with cystic fibrosis is admitted to the hospital with pneumonia. You order a sputum culture and sensitivity. What is the most likely organism causing this patient's pneumonia? A. Klebsiella pneumoniae B. Legionella C. Mycoplasma pneumoniae D. Pseudomonas aeruginosa E. Streptococcus pneumoniae

The Correct Answer is: D Patients with Cystic Fibrosis commonly develop chronic infection and subsequent pneumonia from pseudomonous aeruginosa (D).

A 63-year-old female presents with deep bone pain in both legs. Laboratory evaluation reveals Bence-Jones proteinuria and a monoclonal spike on serum electrophoresis. Which of the following vaccines should this patient receive as part of her management? A. Diphtheria B. Human Papilloma Virus (HPV) C. Pertussis D. Pneumococcal E. Varicella

The Correct Answer is: D Patients with multiple myeloma are at risk from infections, especially from encapsulated organisms such as Haemophilus influenzae and pneumococcus. Pneumococcal vaccines should be administered, but the patient response is decreased based on their current immunodeficiency. DTaP (A and C), HPV (B), and varicella (E) vaccines are appropriate for this patient based on general immunization guidelines for adults, but not of increased necessity based on her diagnosis.

Your patient is a 4-year-old boy whose father brings him to your primary care office concerned about behavioral problems. He cannot seem to sit still in school. He seems to have a great deal of difficulty focusing for any length of time. Which of the following is required in order to make the diagnosis of attention deficit/hyperactivity disorder (ADHD) according to the DSM-IV-TR? A. He exhibit more symptoms of inattention than hyperactivity B. He must be at least 10-years-old C. He behaves aggressively towards others on at least some occasions D. Some impairment from the symptoms is present in two or more settings E. Symptoms of inattention be present on a daily basis for 3 months

The Correct Answer is: D The diagnosis of ADHD requires symptoms of both hyperactivity/impulsivity and of inattention (D). It does not require more or more prominent symptoms (A) of one or the other. Aggression (C) is seldom involved, although the impulsivity may manifest as lack of concern for others. The symptoms must be present before the age of seven (B) and must have been present for at least 6 months (E).

A 72-year-old female presents with a 4-month history of pain and stiffness in her shoulders and hips. She identifies the pain being worse in the morning and aggravated with getting in and out of the car along with difficulty brushing her hair. She also reports malaise and a 10-pound weight loss over the past few months. Her blood work shows an erythrocyte sedimentation rate (ESR) of 74 mm/h. What is the most likely diagnosis? A. Gouty arthritis B. Bursitis C. Fibromyalgia D. Polymyalgia rheumatica E. Septic arthritis

The Correct Answer is: D Polymyalgia rheumatica (D) is a clinical diagnosis based on pain and stiffness of the shoulder and pelvic girdle areas, frequently in association with fever, malaise, and weight loss. Because of the stiffness and pain in the shoulders, hips, and lower back, patients have trouble combing their hair, putting on a coat, or rising from a chair. Elevated ESRs are present in most cases being > 50 mm/h. The age group most affected is over 50 years old. Fibromyalgia (C) is not specific for only the shoulders and hips. Bursitis (B) would usually be specific to one location. Gouty arthritis (A) is usually monoarticular and is not commonly seen in the hips and shoulders on presentation. Septic arthritis (E) would be joint specific and more acute.

A 65-year-old male presents with pain and swelling to his right knee without any history of injury. He has had this type of pain and swelling before, and does recall that he had fluid drained out of the knee several years ago. On examination the patient has a swollen, tender knee with a palpable effusion. There is decreased range of motion to the joint secondary to the effusion. An arthocentesis is performed, and the analysis of the fluid reveals calcium pyrophosphonate crystals. Based on these findings, what is the most likely diagnosis? A. Septic arthritis B. Acute synovitis C. Hemarthoma D. Pseudogout E. Occult fracture

The Correct Answer is: D Pseudogout is also known as Calcium Pyrophosphate Deposition Disease (CPDD) and most commonly affects patients over the age of 65. The knee joint is most commonly affected with the wrist joint second most common. An examination of synovial fluid aspirated from the affect joints, will reveal calcium pyrophosphate crystal. Ankle joints are rarely affected by pseudogout but are fairly common locations for gout caused by urate crystal deposition. The first metatarsal phalangeal joint is the classic location for gout (sometimes referred to as podagra), but it is not a typical pseudogout location. The glenohumeral joint of the shoulder is affected by pseudogout much more than gout. Distal interphalangeal joints of the hand are rare locations for either gout or pseudogout, but are classic locations to see signs and symptoms of osteoarthritis. When pseudogout does affect the hands it is generally seen in the metacarpophalangeal joints.

A 5-week-old male infant presents with a 1-week history of vomiting which occurs shortly after feeding. The mother describes the vomiting as forceful and the vomitus is occasionally blood streaked; the infant has not had diarrhea. You note that the infant appears slightly dehydrated and has lost weight since a routine check at 2 weeks. Which of the following is the most likely diagnosis? A. peptic ulcer disease B. viral gastroenteritis C. Hirschsprung disease D. pyloric stenosis E. intussusception

The Correct Answer is: D Pyloric stenosis usually presents with forceful/projectile vomiting between 2 and 4 weeks of age. There is a 4:1 male predominance; dehydration and failure to thrive may develop. Peptic ulcer disease can occur at any age and commonly affects more men but is more common from 12 to 18 years of age. Intussusception is more common in men but presents with colicky abdominal pain with subsequent development of vomiting and bloody diarrhea.

A 56-year-old male who works in construction climbing ladders has developed pain to the right foot for several days. You have seen and examined the patient a few days after the patient started complaining of pain to the foot. Your initial x-rays of the foot are negative for fracture. He continues to have pain, and decreased ability to bear weight. Based on this clinical scenario, how many days after the initial examination should another x-ray be ordered to look for a stress fracture? A. 1-2 days B. 5-7 days C. 10-12 days D. 14-30 days E. 180 days

The Correct Answer is: D Radiographic evidence of stress fractures is not immediately apparent after the initial onset of symptoms. Estimates of anywhere from 14 to 42 days before visible signs of a stress fracture can be detected on X-ray have been put forth with a commonly utilized range of 14 to 30 days. Any answer choice above that included numbers less than 14 days would simply be inaccurate due to the gradual appearance of the classic x-ray findings of a stress fracture. After 180 days from the time of stress fracture onset, the fracture should be nearly healed if managed properly. If a stress fracture is highly suspected in light of negative x-rays, imaging with a bone scan should help make the diagnosis. MRI scans may help confirm the diagnosis, but they are rarely utilized in the work-up of suspected stress fractures.

A 72-year-old female presents with a several month history of symmetric pain to the hands, wrists, and feet. She also is noticing a change to the appearance of the joints of the hands and feet, with a more "nobby" appearance than before. When the patient wakes up in the morning, the joints are particularly stiff and red at times. Out of all the areas that are painful, it is the hands that are the most painful. Based on this history, what physical exam findings would you expect to find on the hands that are classic findings of this disease? A. Radial deviation and subluxation of the DIP joints B. Radial deviation and subluxation of the MCP joints C. Radial deviation and subluxation at the PIP joints D. Ulnar deviation and subluxation at the MCP joints E. Ulnar deviation and subluxation at the PIP joints

The Correct Answer is: D Rheumatoid arthritis is a systemic autoimmune disease that is incurable and affects woman more than men by a 3 to one ratio. It can damage any joint, but it most often affects joints of the hands, wrists, knees, feet and ankles. The classic appearance of hands that have been afflicted with progressive rheumatoid arthritis is that they demonstrate ulnar deviation and subluxation at the MCP joints. Involvement of the PIP joints is not uncommon, but DIP joints are generally not as significantly affected. Radial deviation is not a typical characteristic of rheumatoid arthritis of the hands.

A 15-year-old female comes into your office with multiple symptoms of an eating disorder. What is the single feature that would lead you to diagnose anorexia rather than bulimia or binge eating disorder? A. Binge eating behaviors B. Compensatory measures such as purging C. Preoccupation with body weight D. Refusal to maintain normal body weight E. Strict focus on dietary intake

The Correct Answer is: D The diagnosis of anorexia requires that the patient refuse to maintain a normal body weight (D). All the other behaviors mentioned (A, B, C, E) may be shared between anorexia and bulimia. People with binge eating disorder (not fully recognized in the DSM IV-TR) display binge eating without compensatory measures, resulting in obesity.

A 76-year-old male presents to the hospital with a hot, swollen, and painful right knee that appears to have an effusion. After your examination, you decide to perform an arthrocentesis. The fluid aspirate is turbid, cloudy, and most definitely looks like it is infectious material. Based on this clinical scenario, what would be most likely bacterial organism present in an elderly septic arthritis? A. Hemophilus influenzae B. Neisseria gonorrhoeae C. Pseudomonas aeruginosa D. Staphylococcus aureus E. Streptococcus pneumonia

The Correct Answer is: D Septic arthritis is an infection within a joint space. The infection can be caused by multiple pathogens including bacteria, viruses, fungi and mycobacteria, but in older adults the most common organism implicated is Staphylococcus aureus. These bacteria can get into the joint space through multiple mechanisms including direct inoculation after a penetrating injury, hematogenous spread from an infection elsewhere in the body, or from extension from a contiguous bone infection. All of the bacteria mentioned as possible answers are possible causes of septic arthritis, but there prevalence varies within different patient populations. Hemophilus influenza and Streptococcus pneumonia are commonly seen in children less than five years of age that develop septic arthritis. Neisseria gonorrhoeae is most common in sexually active adolescents and young adults. Pseudomonas aeruginosa is a common cause of septic arthritis in IV drug abusers.

A 42-year-old woman presents complaining of shortness of breath, three days of fever as high as 103F, and a cough productive of green sputum. On physical examination, you hear crackles in her lungs. A chest radiograph reveals a consolidation in the left lower lobe. What tactile fremitus findings do you expect to observe in the left lower lobe region? A. Absent B. Decreased C. Hyperresonant D. Increased E. Normal

The Correct Answer is: D Tactile fremitus is increased (D) by processes that cause consolidation within the lung parenchyma and decreased (B) by processes that insulate the lung parenchyma from transmitting vibration (e.g., pneumothorax, hemothorax) or airway obstruction.

A 7-year-old girl is brought in by her mom for evaluation of a rash. She has had a fever for a few days and woke up this morning looking like she had been slapped on both cheeks. Other than supportive care, which instruction below represents the best patient education for this patient? A. She should remain out of school because she is contagious until the rash resolves. B. She may return to school but stay out of physical education class to avoid splenic injury. C. It spreads by the fecal-oral route, so she should wash her hands after using the bathroom. D. She may resume normal activities as her energy level improves.

The Correct Answer is: D The "slapped cheek" appearance to the rash is consistent with a Parvovirus B19 etiology for erythema infectiosum. It is a droplet infection that is no longer contagious once the rash breaks out. It generally has a benign course and patients recover fully with supportive care. Splenic involvement is not typically a part of the course, so she may resume activities as she feels able.

A 23-year-old woman presents to the outpatient clinic complaining of chronic congestion and obstruction of her right nasal passage. Upon physical exam you visualize the following. Which of the following is the most likely diagnosis? A. Chronic sinusitis B. Inverted papilloma C. Juvenile angiofibroma D. Nasal polyp E. Squamous cell carcinoma

The Correct Answer is: D The appearance of this pale, edematous, mucosally covered mass is consistent with a nasal polyp (D). Juvenile angiofibromas (C) typically present as epistaxis in adolescent males. The symptoms and appearance of this polyp are inconsistent with the presentation of inverted papilloma (B), chronic sinusitis (A), and squamous cell carcinoma (E).

A 23-year-old patient presents to the Emergency Department complaining about a bloody left eye that occurred after being struck with a tennis ball. The patient denies any decreased vision or photophobia. A photo of the left eye reveals the following (see below). Which of the following is the most likely diagnosis? A. Episcleritis B. Hyphema C. Pinguecula D. Subconjunctival hemorrhage E. Uveitis

The Correct Answer is: D The classic appearance of a bright red, flat collection of blood is consistent with a subconjunctival hemorrhage (D). A hyphema (B) is an internal hemorrhage within the anterior chamber of the eye. The patient lacks conjunctival injection seen in episcleritis (A) and unveitis (E). A pinguecula (C) is a small nodular conjunctival growth.

A 5-year-old boy presents to urgent care complaining of painful lesions in his mouth that have made eating difficult the past 2 days. The mother confirms he has been unable to eat for 48 hours, but has been able to sip water. On physical exam he has a temperature of 102.6 F; numerous small vesicles and ulcers on the buccal mucosa and tongue, inflamed gingiva; and tender anterior cervical adenopathy. Which of the following is the most likely diagnosis? A. Aphthous ulcers B. Hand, foot, and mouth disease C. Herpangina D. Herpes simplex gingivostomatitis E. Viral pharyngitis

The Correct Answer is: D The classic presentation of initial herpes simplex infection (D) includes multiple small, painful vesicles or ulcers on the mucousa with gingival involvement, fever, and adenopathy. (A), (B), and (C) all present with ulcers, but typically involve an isolated area (A), or the tonsils and posterior pharynx (B and C).

A 5-year-old boy presents to urgent care complaining of painful lesions in his mouth that have made eating difficult the past 2 days. The mother confirms he has been unable to eat for 48 hours, but has been able to sip water. On physical exam he has a temperature of 102.6º F; numerous small vesicles and ulcers on the buccal mucosa and tongue; inflamed gingiva; and tender anterior cervical adenopathy. Which of the following is the most likely causative organism? A. Coronavirus B. Coxsackie virus A16 C. Group A beta-hemolytic strep D. Herpes simplex 1 E. Rhinovirus

The Correct Answer is: D The classic presentation of initial infection with herpes simplex virus 1 (D) includes multiple small, painful vesicles or ulcers on the mucousa with gingival involvement, fever, and adenopathy. Coxsakie virus A16 (B) causes hand, foot, and mouth disease. Coronavirus (A) and rhinovirus (E) cause viral pharyngitis.

A 36-year-old female presents with increasing fatigue, appears pale, and states that her hands and feet feel numb. She is a strict vegan and avoids all animal products. What diagnostic study would be most useful in diagnosing this patient? A. Direct bilirubin B. Folate C. Thyroid stimulating hormone D. Total iron binding capacity E. Vitamin B12

The Correct Answer is: E A diagnosis of vitamin B12 deficiency is confirmed by an abnormally low vitamin B12 serum level. Early deficiencies are noted with a serum level of <170, and patients with symptoms usually have levels <100 pg/mL. Other abnormalities may include elevated serum lactate dehydrogenase and a modest increase in indirect bilirubin.

You are evaluating a 32-year-old male who has possibly injured his Achilles tendon while planting his foot into the soft ground while playing touch football. When evaluating a patient with this suspected injury, you have the patient plantar flex his ankle as part of the exam. Based on your knowledge of anatomy, what two plantar flexing muscles attach into the Achilles tendon? A. Anterior tibialis and soleus B. Flexor hallucis longus and flexor digitorum C. Flexor hallucis longus and gastrocnemius D. Gastrocnemius and soleus E. Peroneus longus and peroneus brevis

The Correct Answer is: D The gastrocnemius and soleus muscles form a common tendon at their distal insertions known as the Achilles tendon. This the strongest and thickest tendon in the body and it attaches at its distal end onto the calcaneus bone of the foot. Contraction of the gastrocnemius and soleus muscles are responsible for plantar flexion of the foot and ankle. The anterior tibialis muscle is one of the important dorsiflexors of the foot and ankle. Flexor hallucis longus and flexor digitorum are both involved in plantar flexion, but are not as strong as the gastrocnemius and soleus muscles and do not attach to the Achilles tendon. The flexor hallucis longus is primarily responsible for plantar flexion of the great toe and flexor digitorum produces plantar flexion of the remaining toes. The peroneus longus and peroneus brevis muscles do contribute to plantar flexion, but are not connected to the Achilles tendon. They are also the primary evertor muscles of the foot and ankle.

A 25 year-old HIV positive white male presents with dyspnea and a nonproductive cough. The patient's temperature is 103.5 degrees F, respirations 40, and pulse 140. Physical exam reveals a thin, dyspneic male, height 5'10" and weight 150 pounds, in moderate respiratory distress with minimal bibasilar crackles and peripheral cyanosis. CXR reveals a diffuse interstitial infiltrate and his LDH is elevated. Your most likely diagnosis is: A. Streptococcal pneumoniae B. Klebsiella pneumoniae C. Mycoplasma pneumoniae D. Pneumocystis jiroveci E. Staphlococal aureus

The Correct Answer is: D The insidious onset of atypical pneumonia symptoms in a patient with HIV is consistent with pneumocystis jiroveci (D). Streptococcal pneumoniae (A), Klebsiella pneumonia (B), and Staphlococal aureus (E) present as "typical" pneumonia with the acute onset of fever and productive cough.

A 22-year-old male presents to the Emergency Department with severe pain in his right arm and is in distress without relief after being given narcotic pain medication. He had a right short arm cast placed for a fractured distal radius 12 hours ago. His pain began 4 hours ago, without any associated trauma. On removal of the cast his arm is noticeably swollen, firm, and tender to any palpation or movement. Based on high clinical suspicion of the suspected diagnosis what is the most appropriate treatment option? A. Apply a short arm splint B. Emergent MRI for evaluation C. IV antibiotics D. Emergent surgical consult for a fasciotomy E. IV fluids and narcotics with observation

The Correct Answer is: D The key to early detection of compartment syndrome is a high index of clinical suspicion. Initiate medical management and an emergent surgical consult while the surgical fasciotomy is being arranged. This includes supplemental oxygen and supporting the blood pressure in the hypotensive patient. Place the affected limb at the level of the heart; elevation higher than the heart reduces the arteriovenous pressure gradient. Associated open or closed fractures are common. The patient with compartment syndrome will initially complain of severe pain, often difficult to control even with narcotic pain medications. The pain typically starts within a few hours of the injury, but may develop up to 48 hours after the event. Clinically, the compartment is commonly swollen, firm, and tender to squeezing by the examiner. Removing constrictive casts or dressings, you would not want to reapply a cast (A). An MRI (B) would take too long when time is critical. IV antibiotics would not correct compartment syndrome (C). IV fluids and pain medication (E) can be given while waiting for surgery but are not used to treat compartment syndrome.

The two major risk factors for human immunodeficiency virus (HIV) infection in American women are intravenous drug use and which of the following? A. history of blood transfusion B. needle stick injuries C. pelvic inflammatory disease D. sexual contact with an infected male E. use of oral contraceptives

The Correct Answer is: D The major risk factors for HIV infection in American women are intravenous drug use (33%) and heterosexual contact with an infected partner (65%). Thanks to universal blood donor screening using the HIV ELISA, antigen, and viral load testing, the risk for any person contracting HIV from a screened unit of blood is only 1:1,000,000. The risk for any person following a needle-stick injury is about 1:300 with deeper sticks, hollow bore needles, visible blood on the needle, and advanced stage of disease in the source increasing the risk. HIV infection puts a woman at increased risk for gynecologic complications such as pelvic inflammatory disease. Unlike the use of latex condoms, the use of oral contraceptives does not protect against HIV transmission, but is not, per se, a risk factor for HIV infection.

A 26 year-old man presents to the clinic after developing a severe productive cough, fever, chills, night sweats and weight loss. Which is the most definitive diagnostic test for determining the presence of Mycobacterium tuberculosis as the causative agent of his active pulmonary tuberculosis and yields results in less than 24 hours? A- Acid-fast bacilli smear of sputum B- Blood culture C- Mycobacterial culture of sputum D -Nucleic acid amplification of sputum E- Tuberculin skin test

The Correct Answer is: D The most definitive diagnostic tests for confirmation of tuberculosis are mycobacterial culture (C) which requires at least 2 weeks to grow mycobacterium, and nucleic acid amplification (D) of a respiratory specimen that can be completed in a matter of hours. AFB smear (A) has a sensitivity of only 40-60%, but positive results help establish a presumptive diagnosis. Tuberculin skin testing (D) is effective in screening patients for latent TB. Routine blood cultures (B) are ineffective in diagnosing TB.

A 56-year-old right hand dominant male presents with swelling to the right elbow. He explains to you that he has had this type of swelling before and was diagnosed with bursitis. Based on this description, where would the most likely site for the elbow bursitis be located? A. Capitellum B. Lateral epicondyle C. Medial epicondyle D. Olecranon E. Radial head

The Correct Answer is: D The olecranon bursa is very vulnerable to injury and chronic inflammation due to its superficial location on the extensor side of the elbow. Acute falls, chronic pressure, and other inflammatory processes such as rheumatoid arthritis or gout may contribute to olecranon bursitis. The medial and lateral epicondyles are known to be associated with chronic tendonopathies. The radial head is a common location for subluxation or fracture, while the capitellum at the distal end of the humerus is sometimes associated with fractures and articular cartilage damage.

A 32-year-old woman comes to your office for contraception. She has divorced her husband two weeks ago and is now dating several new boyfriends. She states she has started a new company just over a week ago and has sunk all of her money into it. Although she knows that businesses like hers have not done well lately, she is certain the market is about to shift in her favor. She states she must hurry and get back to work, that she hasn't even had time to sleep for over a week. She speaks very rapidly and is pacing back and forth, seemingly unable to sit still. Conversation is difficult as she is easily distractible. She denies use of drugs or alcohol. She also denies any history of depression. What is her most likely diagnosis? A. Bipolar disorder B. Delusional disorder C. Hypomanic episode D. Manic episode E. Neurosis

The Correct Answer is: D The patient described is exhibiting behaviors consistent with a manic episode (D). Her symptoms have been present for over a week and are impairing her ability to function, so it is not hypomania (C). As she has no clear history of depression, it is likely that she will have a depressive episode in the future, which will allow for a diagnosis of bipolar disorder (A). She does exhibit grandiosity, but there are no clear delusions (B). Neurosis (E) is a more general term for cognitive distress including some form of anxiety.

A 48-year-old man presents to the clinic for a routine employment physical. The patient is asymptomatic. A pre-employment CBC reveals hemoglobin of 13.2g/dl, hematocrit of 39.5%, and MCV of 60.6 fL. Subsequent iron studies, hemoglobin electrophoresis, and sickle cell screening are "normal." After the labs are reviewed, the patient states "they are always normal." Which of the following is the most appropriate management for this patient? A. Bone marrow aspirate B. Ferrous sulfate daily C. Folic acid supplements D. Reassurance E. Refer to an oncologist

The Correct Answer is: D The patient has a mild anemia with pronounced microcytosis consistent with alpha thalassemia minor and requires reassurance. Alpha thalassemia minor hemoglobin electrophoresis reveals normal results. Patients with alpha thalassemia are often inappropriately advised to take iron supplements (B) and don't require iron, folic acid (C) or vitamin B 12 supplements unless a coexisting deficiency is confirmed. Referral to an oncologist (E) and a bone marrow aspirate (A) aren't necessary for this patient.

A 48-year-old female presents with a gradual progression of fatigue, pallor, and dyspnea on exertion over the last few months. Initial CBC results show a hemoglobin of 10.2 mg/dL, hematocrit of 31%, an MCV of 74 fL, and a reticulocyte count of 0.1%. Which of the following is the most likely pathophysiologic mechanism responsible for her anemia? A. Acute blood loss B. Defective bone marrow/stem cell function C. Defective DNA production D. Defective hemoglobin production E. Increased destruction of red blood cells

The Correct Answer is: D The patient most likely has an underlying iron deficiency anemia (IDA), which requires confirmation and then evaluation for sources of chronic blood loss. IDA results from defective hemoglobin production. Acute blood loss (A) would present more acutely and with normal hemoglobin, hematocrit, and MCV until hemodilution occurs and lowers the hemoglobin concentration and hematocrit. Defective bone marrow/stem cell function (B) tends to produce normocytic red blood cells. Defective DNA production (C) results in failure of RBC maturation and macrocytosis (elevated MCV). Hemolysis (E) is less likely than IDA, but can be confirmed or ruled out through the assessment of iron studies, the reticulocyte count and further hematologic labs as necessary.

A 23-year-old man presents to the outpatient clinic for follow-up from a recent urgent care visit. He complains of sore throat, fever, fatigue, myalgias, and a rash that started 5 days ago, and have worsened since he was seen in the urgent care 3 days ago. The patient appears non-toxic with a temperature of 39.4 degrees Celsius. Physical exam reveals pharyngeal and tonsillar erythema without exudates, generalized lymphadenopathy, a morbilliform rash on his trunk, and no hepatosplenomegaly. A rapid strep screen and Monospot performed at the local urgent care were reportedly negative. Which of the following laboratory tests is most likely to confirm the expected diagnosis? A. Complete blood count B. Cytomegalovirus titer C. Group A beta-hemolytic strep culture D. HIV viral load E. Epstein-Barr virus titer

The Correct Answer is: D The patient presentation is consistent with acute retroviral syndrome, which is best confirmed during this initial presentation phase through direct testing for the HIV virus, such as an HIV viral load (D). The lack of tonsillar exudates, a negative monospot, and presence of generalized adenopathy make infectious mononucleosis (B and E) less likely. A CBC (A) may show lymphopenia and support the diagnosis, but it doesn't confirm the diagnosis.

A 45-year-old man presents to the clinic complaining of a lesion in the middle of his right lower eyelid margin that is obscuring his vision when he works on fly-tying or other detailed projects. On physical exam there is a 5 mm round lesion that lacks erythema or tenderness. The patient's visual acuity is 20/20. Which of the following interventions is most appropriate for this patient? A. Apply warm compresses four times daily until resolution B. Apply bacitracin ophthalmic ointment 500 units/gram three times a day C. Aspirate the lesion D. Excise the lesion E. Take amoxicillin-clavulanate 875/125 mg by mouth two times a day for 10 days

The Correct Answer is: D The patient's presentation is consistent with a chalazion that is best treated by surgical excision of the granulomatous tissue (D). Warm compresses (A), bacitracin ointment (B), and amoxicillin-clavulanate (E) are effective treatment options for hordeolum, or hordeoulum with associated preseptal cellulitis.

A 64-year-old female presents with acute onset of severe right knee pain. She denies any trauma and her history is only significant for diabetes mellitus. On physical examination, her knee has significant edema and erythema, with warmth on palpation. After an aspiration of the fluid the results show: Color: opaque yellow fluid WBC: 90,000/mcL PMN: 90% What is the most likely diagnosis? A. Osteoarthritis B. Knee dislocation C. ACL injury D. Fibromyalgia E. Septic arthritis

The Correct Answer is: E Acute bacterial septic arthritis (E) is found most commonly in a weight-bearing joint with acute onset of inflammatory monoarticular arthritis. Joint effusions are usually large with the fluid being opaque and white blood cell counts commonly > 50,000/mcl with 90% or more polymorphonuclear cells. Compromised immunity, such as diabetes mellitus, increases the risk of septic arthritis. Osteoarthritis (A), knee dislocation (B), ACL injury (C), and fibromyalgia (D) would not show an increased WBC count with opaque yellow fluid and 90% PMNs.

A 39-year-old male patient presents with low back pain with radiation to the right leg. On examination you place the right hip in a flexed position, and as you palpate between the iscial tuberosity and the greater tronchanter of the femur the patient complains of radiation of pain down his right leg. Based on this history and exam, which nerve was affected by this part of the examination maneuver? A. Femoral B. Peroneal C. Saphenous D. Sciatic E. Sural

The Correct Answer is: D The sciatic nerve does lie midway between the ischial tuberosity and greater trochanter and it can be palpated when the patient is in a hip flexed position. The gluteus maximus obscures the nerve from being effectively palpated when the leg is in an extended position. Tenderness of the sciatic nerve can be caused by a lumbar disk herniation, direct trauma, or spasm of the nearby pyriformis muscle. The femoral nerve is a deep structure that lies lateral to the femoral artery and is not considered to be palpable. The femoral nerve is responsible for the L1-3 dermatomes and for supplying motor function to the iliopsoas muscle. The peroneal nerve originates from the sciatic nerve and splits into the superficial and deep peroneal nerves, which are responsible for much of the sensory and motor nerve function in the lower leg. The saphenous nerve originates from the femoral nerve in the femoral triangle and runs down the medial aspect of the leg. The sural nerve has medial and lateral components that are found in the lower leg. The medial cutaneous sural nerve arises from the tibial nerve just below the knee and eventually connects with peroneal nerve to form the sural nerve. On the lateral side of the lower leg, the sural nerve arises from the common peroneal nerve just above the knee and eventual connects with the previously discussed medial branch to form the sural nerve.

A 56-year-old male was admitted to the regular medical floor with a diagnosis of sepsis due to urinary tract infection. While on the floor he became acutely unstable and had a blood pressure of 70/40. A medical emergency code was called, which revealed the rhythm shown. There was no palpable pulse on exam, and no evidence of heart sounds on auscultation. The patient was not responding to any stimulus during this time, and there did not appear to be any spontaneous respirations noted. A single, unsynchronized cardioversion shock is given, and the patient has restored blood pressure, and his mental status and responsiveness returns to baseline. Given this clinical scenario, what long-term medical therapy is indicated for this patient? A. Observation B. Permanent pacemaker insertion C. Long-term anticoagulation D. Implantable cardiac defibrillator E. Long-term antibiotics

The Correct Answer is: D This patient developed ventricular fibrillation with hypotension. When a patient has an event of this magnitude, prevention of further events will be a goal, to reduce sudden cardiac death. In this particular situation, the patient would benefit from an implantable cardiac defibrillator (D). This would be accomplished once the acute sepsis is resolved and the patient has been stabilized. The patient would not survive with observation alone (A), and once a rhythm is reestablished a permanent pacemaker (B) is not indicated. Long-term anticoagulation (C) or long-term antibiotic do not address the underlying rhythm problem.

A 44-year-old female presents to the emergency department with a right-sided headache. She states the headache is located on the right temple region, is non-radiating, and does not cause photophobia. She is otherwise healthy and has no reported medical problems. She only takes acetaminophen for the pain, which has minimal relief. On physical exam she is alert, awake, and oriented. Her vitals are T 98.8, P 78, R 18, and BP 128/76. Her head is normocephalic, atraumatic, and pupils are equal and reactive. She has tenderness to the right temporal area of the temporal artery, and there is no noted swelling, redness, or abnormalities noted. There is no tenderness to the cervical muscles. She exhibits a non-focal neurological exam. Based on the description, what is the most likely diagnosis of this patient? A. Migraine headache B. Tension headache C. Cluster headache D. Temporal arteritis E. Takayasu arteritis

The Correct Answer is: D This patient is exhibiting a case of temporal arteritis (D). She has the signs and symptoms that are classic in nature and do not fit into the realm of the other headaches (A, B, and C), all of which usually have different characteristics on history and physical exam. Takayasu arteritis (E) will typically not present in the temporal artery.

A 44-year-old female presents to the emergency department with a right-sided headache. She states the headache is located on the right temple region, is non-radiating, and does not cause photophobia. She is otherwise healthy and has no reported medical problems and has only taken acetaminophen for the pain, which has minimal relief. On physical exam she is alert, awake, and oriented. Her vitals are T 98.8, P 78, R 18, and BP 128/76. Her head is normocephalic, atraumatic, and pupils are equal and reactive. She has tenderness to the right temporal area of the temporal artery, and there is no noted swelling, redness, or abnormalities noted. There is no tenderness to the cervical muscles. She exhibits a non-focal neurological exam. Based on the clinical presentation, what test is the best test in confirming an accurate diagnosis? A. C-reactive protein B. Erythrocyte sedimentation rate (ESR) C. Computed tomography of head D. Temporal artery biopsy E. Complete blood count (CBC)

The Correct Answer is: D This patient is exhibiting a case of temporal arteritis. She has the signs and symptoms that are classic in nature and do not fit into the realm of the other headaches, all of which usually have different characteristics on history and physical exam. The best test to confirm the diagnosis is to biopsy the temporal artery (D) for pathologic confirmation. The three blood tests (A, B, and E) would only suggest the presence of the disease, but they are not specific enough to confirm. Computed tomography of the head (C) would not be indicated as temporal arteritis is an extracranial condition.

5 year-old girl presents to the clinic with her mother who is concerned about her daughter's energy level since starting full day kindergarten. CBC reveals a hemoglobin of 12.3 g/dl, hematocrit of 36%, MCV of 62 fL, and an MCHC of 34 g/dL. Serum ferritin levels are normal. What is the most likely hemoglobin electrophoresis results for this patient? A- Decreased HbA2 and increased HbF levels B- Decreased HbA2 and HbF levels C- Increased HbA2 and normal HbF levels D- Increased HbA2 and HbF levels E- Normal HbA2 and HbF levels

The Correct Answer is: E Alpha thalassemia presents with a normal hemoglobin electrophoresis, since all adult hemoglobin are alpha-containing and affected equally. Beta thalassemia major may present with increased HbA 2 and/or HbF levels (C and D). Beta thalassemia minor typically presents with increased HbA 2 (C).

An 84-year-old male is admitted to the hospital for the chief complaint of syncope. The history provided states that the patient was in his normal state of health and feeling well when he had fallen, with an apparent loss of consciousness for 25-30 seconds. He has a history of hypertension and arthritis, and is well managed on medications that include lisinopril and acetaminophen. During his time on the telemetry unit it's noted that the patient has periods of sinus bradycardia in the 30s, followed by normal sinus rhythms that fluctuate in the 60 to 120 range. During several of the bradycardia episodes the patient becomes symptomatic with shortness of breath, lightheadedness, and dizziness. His blood pressure during these episodes is measured at 88/56. Given this clinical scenario, what is the most appropriate treatment for this patient? A. Observation B. Increase in lisinopril C. Cardiac catheterization D. Permanent pacemaker insertion E. Beta blocker therapy

The Correct Answer is: D This patient is having sick sinus syndrome, in which the patient has developed an aberrancy wherein the heart is not able to regulate a steady rate and maintain an adequate blood pressure. Because of the patient having syncope, observation only (A) is not advised. Increasing the ACE inhibitor (B) is contraindicated given the patient experiencing hypotension. This is also true with beta-blocker therapy (E). The best treatment for this clinical situation would be permanent pacemaker insertion (D). Cardiac catheterization (C) is not indicated in this situation, and does not address the electrophysiological issue.

An 82-year-old male presents to the emergency department with a 2-day history of weakness, fatigue, and mild shortness of breath. His past medical history includes hypertension, arthritis, diabetes, and hypercholesterolemia. His medications include metoprolol, glyburide, simvastatin, and acetaminophen. On physical examination, the vital signs reveal T 97.7, P 40, R 15, and BP 84/60. His neck exam reveals no jugular venous distention, lungs are clear to auscultation, and cardiac has a regularly irregular bradycardia rhythm. There is no evidence of edema to the extremities. The patient is placed on a telemetry monitor, which reveals the image shown. Based on the information provided, what is the most appropriate therapy for this patient? A. Observation B. Start cardiopulmonary resuscitation C. Unsynchronized cardioversion D. Permanent pacemaker insertion E. Cardiac catheterization

The Correct Answer is: D This patient's telemetry strip is revealing complete heart block. Given this scenario it is unlikely that a rhythm such as this will convert back to a normal sinus rhythm. Therefore, a permanent pacemaker (D) is warranted. In this case, the patient would require immediate intervention due to his symptoms as well as his unstable hemodynamics. Patients left unattended in this setting may go into cardiac arrest due to the slow ventricular rate the patient has developed. The patient would not survive with observation alone (A). The patient has valid vital signs with some hemodynamics, so cardiopulmonary resuscitation (B) is not indicated and harmful. Unsynchronized cardioversion (C) and cardiac catheterization (E) are both not indicated.

A 12-year-old boy is being seen for concerns of development of breast tissue. Upon physical exam, he is noted to have a firm, slightly tender mass under the left areola. What is the most appropriate action at this time? A. referral to pediatric surgery for resection B. measurement of serum hCG C. measurement of testosterone and estrogen levels D. reassurance and observation

The Correct Answer is: D Type 1 idiopathic gynecomastia in adolescent men presents with a firm mass under the areola ("breast bud") typically during sexual maturation stages (SMR), stages II to III. This is a result of normal estrogen and androgen activity at the breast tissue level. Appropriate action is observation and to reassure the patient that the condition will likely resolve in 1 to 2 years.

Which of the following is the most common cause of sudden death in young athletes under 35 years of age? A. sthelosclerotic heart disease B. Brugada syndrome C. commotio cordis D. idiopathic hypertrophic cardiomyopathy E. Marfan syndrome

The Correct Answer is: D Undetected idiopathic hypertrophic cardiomyopathy is the most common cause of sudden death in otherwise healthy young athletes. A heart murmur, chest pain, or shortness of breath in this patient population should trigger an escalating level of investigation including consideration of at least both an EKG and an echocardiogram. Commotio cordis, while most common in adolescent athletes, remains extremely rare.

An 18-year-old female with no significant past medical history presents with a gradual progression of fatigue and pallor over the last three months. Initial CBC results show a hemoglobin of 9.8 mg/dL, hematocrit of 30%, an MCV of 116 fL, and a reticulocyte count of 0.1%.Which of the following dietary practices is most likely to contribute to her anemia? A. High protein B. Low carbohydrate C. Low fat D. Vegan E. Ovo-lacto vegetarian

The Correct Answer is: D Vitamin B 12 is derived from animal products and thus vegans are at risk for deficiency that results in macrocytic anemia. Patients who pursue a low carbohydrate (B), high protein (A) diet likely rely on animal sources of protein and/or fats to meet their caloric needs. Ovo-lacto vegetarians (E) obtain vitamin B 12 from eggs and dairy products.

A 17-year-old female patient presents to your family practice office complaining of a persistent sore throat for the past 2 weeks duration. She is extremely fatigued, and she tells you that her boyfriend has had similar symptoms for about a week. Her vital signs are a blood pressure of 116/72 mm&thinsp;Hg, pulse 88 beats/min, respirations of 16 beats/min, and temperature of 101.1˚F taken orally. Her white blood cell count (WBC) reveals 14,800 mcL with more than 10% atypical lymphocytes. On physical examination you find significant yellowish-grey pharyngeal exudate, cervical lymphadenopathy as well as an enlarged spleen. Her rapid strep is negative. What should be your treatment of this patient? A. Begin treatment with cephalexin. B. Send out a back-up throat culture and begin treatment with cephalexin pending those results. C. Send out a back-up throat culture and hold treatment pending those results. D. Draw a Monospot test and treat the patient supportively pending the results, insuring that they avoid contact sports until a definitive diagnosis has been made. E. Draw an Epstein-Barr virus (EBV) IgG level and treat the patient supportively if positive.

The Correct Answer is: D With this patient's presentation and a negative rapid strep test, other etiologies beyond group A beta-hemolytic strep should be seriously considered. In light of her symptoms including fatigue, her WBC, and that her significant other is having similar symptoms, infectious mononucleosis is highest on the differential diagnosis. A Monospot test would rule this possibility in or out. She should refrain from contact sports until her infection and any associated splenomegaly has resolved.

A fracture involving the medial epicondyle will most likely cause damage to which nerve? A. Axillary B. Median C. Peroneal D. Radial E. Ulnar

The Correct Answer is: E Because the ulnar nerve passes through the cubital tunnel, which is a groove on the posterior aspect of the medial epicondyle, any fractures involving the medial epicondyle can also cause damage to the ulnar nerve. The median nerve is most susceptible to injury at the carpal tunnel. Branches of the radial nerve can become entrapped on the lateral side of the elbow and the associated symptoms are often confused with lateral epicondylitis. Radial nerve injuries are more commonly associated with humeral shaft fractures. The axillary nerve is significantly proximal to the medial epicondyle and the peroneal nerve is in the leg.

Mr. Addley is a 69-year-old, recently retired accountant who comes to you in your family practice, accompanied by his wife, for a routine follow-up of his blood pressure. During his visit he comments that he is concerned about his memory. He notes that he is having some difficulty remembering where he left his car keys, the day of the week, and names of people immediately after being introduced to them. When his wife is asked, she has noticed nothing regarding any change in his mental status or his ability to perform activities of daily living. You perform a mini-mental state exam (MMSE) and he scores 25 out of 30. What is the most appropriate working diagnosis for Mr. Addley? A. mild to moderate depression B. mild Alzheimer's disease C. moderate Alzheimer's disease D. Lewy body dementia E. mild cognitive impairment

The Correct Answer is: E Because your patient notices symptoms but family members, friends, and coworkers do not, and his MMSE is slightly below normal, your patient meets the diagnostic criteria for Mild Cognitive Impairment, or MCI. Metabolic causes should be ruled out before making a definitive diagnosis. This may or may not progress to Alzheimer's and there are no FDA-approved medications for MCI, although some clinicians do utilize acetylcholinesterase-inhibitors after discussions with patients and their families.

A 2 year-old girl presents to the clinic due to fever, and right ear pain. Physical exam reveals a temperature of 103.1 F and an erythematous and immobile right tympanic membrane. The patient is subsequently prescribed amoxicllin. What is the mechanism of amoxicillin? A. Binds to the 30S ribosomal subunit to inhibit protein synthesis B. Binds to the 50S ribosomal subunit to inhibit protein synthesis C. Inhibits dihydropteroate synthase and folate production D. Inhibits DNA replication by binding to DNA gyrase and topoisomerase IV E. Inhibits the transpeptidation reaction

The Correct Answer is: E Beta lactam antibiotics inhibit the transpeptidation reaction leading to cell wall destruction (E). Tetracycline binds to the 30S ribosomal subunit to inhibit protein synthesis (A). Macrolide antibiotics bind to the 50S ribosomal subunit to inhibit protein synthesis (B). Sulfamethoxazole inhibits dihydropteroate synthase and folate production (C). Ciprofloxacin inhibits DNA replication by binding to DNA gyrase and topoisomerase IV (D).

A 73 year-old woman with a past medical history of diabetes and hypertension presents to the clinic complaining of progressive fatigue. An initial CBC reveals a hemoglobin 10.1 mg/dL and MCV of 91 fL. Which of the following best describes the cause of the abnormality pictured in the patient's peripheral smear? (note photo taken from figure 57-5- in Harrison's) A. Absent or non-functioning spleen B. Failure of nuclear maturation C. Intravascular hemolysis D. Lead intoxication E. Presence of uremia

The Correct Answer is: E Burr cells are present on the peripheral smear and are associated with uremia. Howell-Jolly bodies result from the lack of removal of nuclear material due to an absent or non-functioning spleen (A). Macrocytosis results from failure of nuclear maturation (B) commonly secondary to vitamin B 12 or folate deficiency. Intravascular hemolysis (C) creates schizocytes, and lead intoxication (D) or thalassemia cause basophilic stippling.

A 21-year-old female patient presents with a large area of swelling localized to the front of the knee, between the patella and the skin after a fall directly on her knee. She has been stating that her knee is becoming more difficult to move due to the swelling and pain. She is otherwise healthy with no other medical problems. Her exam reveals a tender, fluctuant area just anterior to the patella without warmth. She does not appear to have any effusion, and her ligament stability testing is normal. Based on these findings, what is the most likely diagnosis? A. Anterior cruciate ligament tear B. Medial meniscal tear C. Pes anserine bursitis D. Posterior cruciate ligament tear E. Prepatellar bursitis

The Correct Answer is: E Bursae are synovial fluid filled sacs that facilitate the reduction of friction between adjacent structures. They may be found between the skin and various bony prominences or between tendons, ligaments and bone. They can become injured by an acute direct impact or gradual repetitive stress such as what might occur if someone was required to do extensive kneeling on the job. They may also be sites of infection with staphylococcus aureus and streptococcus species being the most common pathogens. The knee contains two primary bursae. The prepatellar bursa lies directly below the skin and above the patella. It is often acutely injured with a fall or other traumatic impact. A localized swelling can occur quickly and this fluid accumulation is not intracapsular as occurs with a joint effusion after an anterior cruciate ligament tear, meniscal tear or posterior cruciate tear. Meniscal injuries tend to produce a smaller effusion than cruciate ligament tears. The mechanism of falling on a bent knee can be consistent with a posterior cruciate ligament injury, but given the lack of an effusion and the presence of a well contained anterior knee swelling, this becomes less likely. Pes anserine bursa are located on the anterior medial aspect of the tibia, just below the tibial plateau. This location is the insertion of the gracilis, sartorius, and semitendinosus muscles. Pes anserine bursitis is commonly associated with early osteoarthritis in the medial compartment of the knee.

A 57-year-old woman with a history of rheumatic fever is seen complaining of dyspnea while vacuuming her apartment, which has been worsening over the last few months. On physical exam, a possible opening snap, loud S 1 , and a very soft diastolic rumbling murmur is auscultated. When the patient is placed in the left lateral decubitus position, the murmur is accentuated, and heard best at the apex. With inspiration, the murmur does not increase in amplitude. Which of the following is the most appropriate next diagnostic study? A. Chest x-ray B. Transesophageal echocardiogram C. Holter monitor D. Treadmill exercise stress test E. Transthoracic echocardiogram

The Correct Answer is: E Choice E, transthoracic echocardiogram, is a simple, sensitive and non-invasive diagnostic tool, which can evaluate for the presence of valvulopathy in the setting of a patient with a diastolic murmur and a history of rheumatic fever. Choice B is also useful for an evaluation for the presence of valvulopathy, but is more invasive than choice E; therefore, choice E is the more appropriate study. Choice A might be able to give evidence of cardiomegaly, but would not be sensitive enough to detect valvulopathy. Choice C is a useful diagnostic tool for the evaluation of patients complaining of palpitations, but incorrect for this patient who has no symptoms. Choice D, although a useful diagnostic tool for the evaluation of exercise tolerance and in patients complaining of chest pain, does not allow direct visualization of the heart valves to evaluate for valvulopathy.

A 32 year-old obese woman presents to the clinic complaining of swelling, warmth, and redness of her left lower extremity. Doppler ultrasound confirms a deep venous thrombosis is the greater saphenous vein. Which of the following best describes the most likely pathophysiology underlying her thrombophilia? A- Antithrombin deficiency B- Increased plasma prothrombin C- Protein C deficiency D- Protein S deficiency E- Resistance to activated protein C

The Correct Answer is: E Factor V Leiden is a common genetic mutation that results in resistance to activated protein C. Increased plasma prothrombin concentration results from a prothrombin gene mutation (B), while deficiencies of antithrombin (A), protein C (C) and protein S (D) are rarer causes of prothrombotic states and are not directly associated with factor V Leiden mutation which is the most common cause of thrombophilia in this clinical scenario

Approximately what percentage of patients with a solid primary tumor elsewhere will end up with metastatic disease of the vertebrae during the clinical course of their cancer? A. 10% B. 20% C. 30% D. 40% E. 50%

The Correct Answer is: E Fifty percent of cancer patients will develop metastatic disease of the vertebrae at some point during the course of their illness. The highest percentages of cancers that lead to such spinal lesions are carcinomas of the breast, lung, prostate, colon, thyroid and kidney. This likely occurs through hematogenous spread. Pain is a common presenting symptom, but it may be found while still asymptomatic if routine screenings are done with bone scans, MRI or CT in patients with a known primary tumor elsewhere in the body. The pain is usually worse with weight bearing activities and better when lying down, but pain that persists through the night and prevents sleep needs to be evaluated for possible neoplasm. Some metastatic disease of the spine may present with neurological symptoms such as sensory or motor deficits following a spinal nerve root distribution or more generalized neurological deficits due to spinal cord compression or cauda equina syndrome.

Q 6.10: A 64-year-old female who has a history of injectable drug use presents with blood work that reveals leukocytosis with a left shift, and there is suspicion of osteomyelitis based on the patient's prior history. Based on this history, what bone would be most affected by hematogenous osteomyelitis in adults? A. Feet B. Long bones C. Pelvis D. Sternoclavicular bones E. Vertebrae

The Correct Answer is: E Hematogenous osteomyelitis accounts for about 20% of all cases of osteomyelitis in adults. It is more common in males and the prevalence is higher amongst those who are IV drug abusers, patients being treated with dialysis or who have sickle cell disease. Other conditions which may lead to sepsis (i.e. patients with central lines, urinary infections, and urethral catheterization) increase the risk of hematogenous osteomyelitis. Unlike children, the long bones are rarely affected in adults with the vertebrae being the most likely location for the bone infection to occur. Lumbar vertebrae are most often affected, followed by thoracic and cervical vertebrae. Osteomyelitis of the sternoclavicular bones and pelvic bones are not uncommon sites, but these tend to be most frequent amongst IV drug abusers. The feet do not tend to be significantly affected by hematogenous osteomyelitis as frequently as osteomyelitis caused by infected foot ulcers as are often seen in those with diabetes or peripheral vascular disease.

Which of the following statement or statements is/are true regarding influenza vaccination? A. The Advisory Committee on Immunization Practices (ACIP) recommends annual influenza vaccination for all persons six months of age and older without specific contraindications. B. In spite of the fact that the influenza vaccine is FDA category C, ACIP still recommends immunizing pregnant women or women who are planning to get pregnant during the upcoming influenza season. C. In spite of the manufacturing method of the current influenza vaccine using chicken embryos, ACIP recommends vaccination persons with known allergies to eggs if they can eat baked products manufactured in the US without having an allergic reaction. D. If an unvaccinated person develops a flu-like illness early in the influenza season, ACIP recommends that they receive an influenza vaccine once their febrile illness has resolved. E. All of the above is true.

The Correct Answer is: E Influenza (the flu) is a contagious respiratory illness caused by influenza viruses. It can cause mild to severe illness, and at times can lead to death, especially in certain populations. Some people, such as older people, young children, and people with certain health conditions including pregnancy, are at the highest risk for serious flu complications. The best way to prevent the flu is by getting vaccinated each year. The vaccine contains three strains (two of influenza A and one of influenza B) and is derived from chicken embryos and people with severe egg allergies should not be vaccinated. However, if a person is able to eat backed goods manufactured in the United States, according to the CDC, their potential level of allergy is so low that the benefit of vaccination outweighs any reaction. Once a vaccine-eligible patient has a viral-like illness and their fever resolves, he or she should receive the annual influenza vaccine. (Hay, et al., 2011, Chapter 9)

A 67 year-old man with chronic obstructive pulmonary disease is prescribed ipratropium bromide inhaler. Which of the following is a potential side effect associated with ipratropium? A. Bradycardia B. Diarrhea C. Salivation D. Sweating E. Urinary retention

The Correct Answer is: E Ipratropium is an anticholinergic agent that can directly cause tachycardia, ileus, dry mouth, flushed/warm/dry skin, delirium, and urinary retention (E). Answer choices A, B, C and D result from cholinergic agents.

A 23 year-old male with cystic fibrosis inquires about the availability of treatments that can help improve his lung function. Which of the following treatments is most effective at reversing the pulmonary effects of cystic fibrosis? A. Albuterol B. Azithromycin C. Inhaled hypertonic saline D. Inhaled levofloxacin E. Ivacaftor

The Correct Answer is: E Ivacaftor (E) is the only treatment that restores function of the CFTR protein in cystic fibrosis patients with a G551D mutation thereby reversing the effects of the disease, approximately 5% of all cystic fibrosis patients have the G551D mutation. Albuterol (A) and hypertonic saline (C) are indicated to improve lung function and mucous clearance. Azithromycin (B) and inhaled levofloxacin (D) are used to treat chronic infection/colonization with pseudomonous.

A 15-year-old male presents complaining of a sore throat, headache, and mild cough that started 8 days ago and has progressed to include a worsening cough and increasing fatigue. His chest x-ray reveals bilateral hilar infiltrates, CBC is normal and a nasal secretions test positive for mycoplasma pneumoniae by PCR. What is the most appropriate therapy? A. Amoxicillin B. Cefuroxime C. Clindamycin D. Ciprofloxacin E. Doxycycline

The Correct Answer is: E Mycoplasma pneumoniae is commonly treated with macrolides, doxycycline (E), or respiratory fluoroquinolones. Mycoplasma pneumonia doesn't respond to beta-lactam antibiotics (A, B) or non-respiratory fluoroquinolones (D).

A 38 year-old woman with severe-persistent asthma presents to the clinic complaining of nightly nighttime awakens and frequent use of her inhaled albuterol. Which of the following is an antibody that prevents IgE from binding to its receptor on mast cells and basophils and may be considered as a treatment for this patient? A. adlimumab B. daclizumab C. etanercept D. infliximab E. omalizumab

The Correct Answer is: E Omalizumab (E), is an antibody that prevents IgE from binding to its receptor on mast cells and is used in the treatment of allergic disease. Daclizumab (B) is anti-IL-2 antibody used to prevent acute rejection of organ transplants. Adlimumab (A), etanercept (C), and infliximab (D) bind TNF, thus inhibiting the action of TNF and are used in the treatment of disorders such as psoriasis and rheumatoid arthritis.

A 78-year-old woman with chronic, open-angle glaucoma (COAG) presents to the outpatient clinic complaining of fatigue, and near syncope. She just began eye drops for her glaucoma 3 days ago. Which of the following ophthalmic drop medications is likely causing her side effects? A. Bimatoprost B. Brimonidine C. Brinzolamide D. Dipivefrin E. Timolol

The Correct Answer is: E Open angle glaucoma is typically treated first-line with beta adrenergic blocking agents (E) or prostaglandin analogs (A). A common side effect of beta adrenergic agents is their systemic absorption and subsequent beta blockade sides effects (i.e. decreased pulse, blood pressure) leading to symptoms such as fatigue, pre-syncope, or syncope. Treatments (B), (C), and (D) are not typically first-line agents and lack the beta blockade side effects this patient is experiencing.

A 56-year-old insulin dependent diabetic has been under your evaluation for his diabetes for several years. The patient has a 3-year history of diabetic neuropathy to the right foot, and may have suffered an injury to the foot without knowing due to loss of sensation. The patient now presents with a tender, reddened, and swollen right foot for the last 10 days that is also warm to the touch. You suspect that this patient may have an acute case of osteomyelitis. Based on this history, what bacterial organism is most commonly the cause of osteomyelitis? A. Group A beta-hemolytic streptococci B. Hemophilus influenzae C. Mycoplasma D. Pseudomonas aeruginosa E. Staphylococcus aureus

The Correct Answer is: E Osteomyelitis is an infection in a bone and can occur in patients of all ages. The most common organism implicated in osteomyelitis across all age groups is Staphylococcus aureus. It can enter the bone through multiple mechanisms including by direct inoculation during an open fracture or during surgical intervention following a fracture (most common mechanisms for adults) or by hematogenous spread from another source (the usual cause in children). Hemophilus influenzae was a much more common organism in pediatric osteomyelitis in the past, but its prevalence is decreasing due to routine immunizations. Group A beta-hemolytic streptococci is the second most common organism found in osteomyelitis in children, while Pseudomonas aeruginosa is the second most common pathogen in adults. Mycoplasma induced osteomyelitis is relatively rare and usually confined to immunocompromised patients.

Which of the following is a type of primary malignant tumor of the bone? A. Enchondroma B. Leiomyoma C. Lipoma D. Melanoma E. Osteosarcoma

The Correct Answer is: E Osteosarcoma is a type of primary bone cancer. Fortunately this is a rare cancer, accounting for only 1% of the cancers diagnosed each year in the USA for patients of all ages. Osteosarcomas are more common in the pediatric population, accounting for 5% of all childhood cancers and 56% of all cancers of the bone in patients less than 20 years old. Osteosarcomas have a bimodal occurrence rate with the highest prevalence between ages 11-13 and over the age of 65. Enchondromas are common benign cartilaginous tumors that develop in the medulla (marrow cavity) of bone. A leiomyoma is another name for a benign uterine fibroid. A lipoma is a benign, soft, freely movable, generally nontender mass in the soft tissue sometimes referred to as a fatty tumor. These are generally inconsequential, but may be a marker for spina bifida if found in the lumbar region. Melanoma can be found in bone, but it would be considered a metastatic lesion and not a primary malignant bone tumor.

A 22-year-old patient with sickle cell disease presents to the emergency department complaining of chest pain, fever, and non-productive cough. On physical exam his temperature is 100.6˚F, BP is 144/88, pulse is 110, respiratory rate is 24, and pulse oximetry is 84%. CBC shows a WBC of 11,500, hemoglobin of 8.3%, and hematocrit of 28%. What is the most critical/emergent treatment for this patient? A. Acetaminophen B. Bone marrow transplant C. Furosemide D. Hydroxyurea E. Transfusion

The Correct Answer is: E Patients with sickle cell disease are prone to acute chest syndrome resulting from sickling of cells within the lung that typically presents with chest pain, tachypnea, cough, fever, and oxygen desaturation. Transfusion is indicated when the hematocrit drops below 30% or oxygen saturation drops below 90%. Bone marrow transplant is only an option in children (B) and not indicated for emergent therapy. Furosemide (C) would promote dehydration and further exacerbation of cell sickling. Hydroxyurea (D) is used prophylactically to reduce the frequency of sickle cell crises and/or acute chest syndrome. Acetaminophen (A) may be used for symptomatic treatment, but is less critical than transfusion and may not provide adequate pain management.

A 64-year-old, nonsmoking male presents to the clinic with complaints of headache and dizziness. Initial labs reveal hemoglobin of 20.1 g/dl, hematocrit of 60%, and platelet count of 567,000. Which of the following is the most likely diagnosis in this patient? A. Acute myeloid leukemia B. Chronic myeloid leukemia C. Chronic obstructive pulmonary disease (COPD) D. Multiple myeloma E. Polycythemia vera

The Correct Answer is: E Polycythemia vera often presents with complaints related to increased red blood cell mass, such as, headache, fatigue, and dizziness with elevated red blood cell mass with leukocytosis and/or thrombocytosis noted on CBC. Chronic hypoxia from conditions such as COPD (C) can also cause elevated RBC mass, but is unlikely in this non-smoker. The patient lacks white blood cell abnormality symptoms (e.g., recurrent infections, elevated white count) making acute myeloid leukemia (A), chronic myeloid leukemia (B), or multiple myeloma (D) unlikely diagnoses.

A 23 year-old patient being treated for tuberculosis presents to the clinic complaining of red discoloration of his contact lenses and staining of his eye. Which of the following medications can change the color of tears leading to this complication? A. Amoxicillin B. Clindamycin C. Doxycycline D. Metronidazole E. Rifampin

The Correct Answer is: E Rifampin (E) is an anti-tuberculosis medicine that changes urine, tears, feces, and sweat to a harmless orange color. Patients should be made aware of this effect prior to starting the medication and should be advised to discontinue contact lens usage while taking rifampin.

Your patient is a 42-year-old male who comes to you because, after a recent move to the country, he finds he has an excessive fear of snakes leading to extreme anxiety. He realizes that this fear has been present since childhood, but was not much of a problem before. What is your diagnosis? A. Conversion disorder B. Factitious disorder C. Generalized anxiety disorder D. Social phobia E. Specific phobia

The Correct Answer is: E Specific phobia (E) is the fear of a very specific object or situation that the individual knows to be excessive. Social phobia (D) involves performance, while generalized anxiety (C) involves anxiety without a known stimulus. Conversion disorder (A) and factitious disorder (B) are both somatoform disorders, involving somatic symptoms with a psychogenic cause.

A 64-year-old, nonsmoking male presents to the clinic with complaints of headache and dizziness. Initial labs reveal hemoglobin of 20.1 g/dl, hematocrit of 60%, and platelet count of 567,000. Which of the following is the most likely physical exam finding in this patient? A. Erythromelalgia B. Hepatomegaly C. Loss of vibratory sense D. Petechiae E. Splenomegaly

The Correct Answer is: E Splenomegaly is the most common physical exam finding in patients with polycythemia vera (PCV) and may be accompanied by hepatomegaly (B). Erythromelalgia (A) also occurs in patients with PCV due to increased red blood cell mass, but occurs less commonly than splenomegaly. Patients with PCV often have thrombocytosis, which most commonly contributes to thrombotic events, but less commonly may result in abnormal platelet function and petechiae (D) and increased bleeding risk. PCV is associated with elevated B 12 levels and not likely to result in the classic neurologic findings of B 12 deficiency including peripheral neuropathy assessed by vibratory sensation (C), ataxia and dementia.

A 16-year-old male high school wrestler presents to your family practice clinic with a fluctuant 3 cm by 3 cm abscess to his back superior and lateral to his scapula. Which of the following is the most appropriate treatment? A. amoxicillin-clavulanate orally for 10 days B. cephalexin orally for 10 days C. trimethoprim sulfamethoxazole orally for 10 days D. vancomycin intravenously for 10 days E. incision and drainage is likely to resolve the abscess without the need for medications

The Correct Answer is: E The causative agent of this abscess is most likely caused by community-acquired methicillin-resistant S aureus (caMRSA). Infectious Diseases Society of America (IDSA) guidelines issued in January of 2011 generally recommend incision and drainage alone for fluctuant abscesses in an otherwise immunocompetent patient.

A 23-year-old male with a recent diagnosis of Non-Hodgkin's lymphoma presents complaining of swelling of the neck and face, cough, and dyspnea on exertion. On exam you note dilated neck veins. Which of the following is the most likely diagnosis? A. Angioedema B. Carotid artery dissection C. Cushing's syndrome D. Myxedema E. Superior vena cava syndrome

The Correct Answer is: E The classic presentation signs and symptoms of superior vena cava syndrome (SVCS) are present in this patient. Although the most common cancer associated with SVCS is lung cancer, it also occurs in Non-Hodgkin's lymphoma. Angioedema (A), carotid artery dissection (B), Cushing's syndrome (C) and myxedema (D) can result in facial swelling, but should not lead to the presence of dilated neck veins.

A 64-year-old, nonsmoking male presents to the clinic with complaints of headache and dizziness. Initial labs reveal hemoglobin of 20.1 g/dl, hematocrit of 60%, and platelet count of 567,000. Which of the following laboratory finding confirms the suspected diagnosis? A. Bone marrow hypocellularity B. Elevated ferritin C. Elevated serum erythropoietin D. Presence of Philadelphia chromosome E. Presence of JAK2 mutation

The Correct Answer is: E The clinical presentation is most consistent with polycythemia vera (PCV), which, in the presence of a markedly elevated red blood cell mass, is confirmed by the presence of the JAK2 mutation. The patient's erythropoietin (C) and ferritin (B) are most likely decreased as a result of bone marrow overproduction of RBCs and use of iron stores to produce these RBCs. The bone marrow of patients with PCV is typically hypercellular (A) across all cell lines.

A 24-year-old woman comes to your office complaining of episodic chest pains that have been occurring over the past several months. She experiences shortness of breath with each occurrence and feels like "I'm going to die." She denies cough, fever, or sputum production and has noticed the episodes occur when she is in a crowd. Her current examination is normal and she feels well between episodes. You suspect panic disorder, but want to rule out the most likely organic causes. Which of the following tests will you include in your initial work-up? A. Arterial blood gases B. CBC C. Chest x-ray D. Sputum culture E. Urine toxicology screen

The Correct Answer is: E The differential diagnosis of panic disorder includes many cardiac, pulmonary, and endocrine conditions, but in a young woman without any symptoms or signs between episodes, drug use, as detected on a urine toxicology screen (E), is the most likely cause. An EKG might also be considered to rule out a cardiac cause. CBC (B) and sputum culture (D) might be helpful if infection is a consideration, but she has no fever or other signs of infection. Chest x-ray (C) and blood gases (A) would not be helpful without symptoms or signs suggesting particular etiologies.

What is the first type of motion that is lost as a person develops progressively worsening osteoarthritis of the hip joint? A. Abduction B. Adduction C. External rotation D. Flexion E. Internal rotation

The Correct Answer is: E The earliest sign of the development of osteoarthritis of the hip is often the loss of internal rotation. As the condition worsens, muscular contractures may develop which hold the affected limb in a flexed and externally rotated position, which has great consequence on the individual's gait and functioning level. Typically a person experiencing this type of osteoarthritis will eventually develop an antalgic gait where the time spent bearing weight on the affected limb is brief due to pain. Also, the gluteus medius (which is a hip abductor and helps stabilize the pelvis) may become weakened as the condition worsens, resulting in an abductor lurch as the trunk of the body sways out over the affected limb when attempting to walk.

A 58-year-old female presents to the outpatient clinic complaining of 1 week of rhinorrhea, nonproductive cough, and hoarseness. On physical exam she is noted to have erythematous nasal mucosa and decreased phonation without significant nasal discharge, sinus tenderness, pharyngeal erythema, or lymphadenopathy. Which of the following is the most likely causative organism for this patient's condition? A. Clostridium diphtheriae B. Group A streptococcus C. Moraxella catarrhalis D. Mycobacterium tuberculosis E. Rhinovirus

The Correct Answer is: E The presence of acute hoarseness associated with an upper respiratory infection is consistent with laryngitis, which may be caused by all of the organisms above (A-E), but most likely has a viral (E) etiology.

A 73-year-old, male with a 30 pack-year smoking history presents to the clinic with complaints of headache and dizziness. Labs reveal a hemoglobin of 21.3 g/dl, hematocrit of 63%, and platelet count of 498,000. He tests positive for the JAK2 mutation. Which of the following is the most appropriate treatment for the patient's suspected diagnosis? A. Allopurinol B. Anagrelide C. Diphenhydramine D. Hydroxyurea E. Phlebotomy

The Correct Answer is: E The first-line, and primary treatment for polycythemia vera is chronic phlebotomy. Patients who fail to improve or don't tolerate chronic phlebotomy can be treated with hydroxyurea (D), with anagrelide (B) being a third-line treatment. Allopurinol (A) may be necessary to treat hyperuricemia and gout. Diphenhydramine (C) may be used as symptomatic treatment for pruritus that doesn't improve with phlebotomy.

A 14-year-old girl presents 1 week after the neighbor's cat bit her hand. In the first 3 days after the bite she developed a shallow ulcer at the bite site. Because her parents knew the cat was up to date on shots, they treated the ulcer with topical antibiotics and did not seek medical care. Now, the patient has low-grade fever and headache and feels tired. Axillary nodes on the affected side are swollen. The ulcer on the hand is nearly healed. The best treatment option is A. doxycycline 100 mg bid × 21 days B. Augmentin 500 mg po bid × 10 days C. azithromycin 500 mg po qd × 7 days D. acyclovir 400 mg po bid × 10 days E. no therapy required

The Correct Answer is: E The history and course of illness are consistent with cat-scratch fever. It is caused by infection with Bartonella henselae. Cat scratch or bite transmits it to humans. Clinical course usually begins with papule or ulcer at the site within a few days of the bite. Fever, headache, and malaise develop 7 to 21 days later. Lymph drainage of the site may result in swollen, tender, and/or suppurative nodes. Clinical diagnosis is the norm but special cultures or biopsy is possible. The symptoms usually resolve spontaneously with no specific therapy required. Complications may include encephalitis or disseminated disease in immunocompromised patients.

A 17 year-old male is training for an Ironman triathlon and notes excessive coughing, chest tightness and wheezing when running. Which of the following is the most appropriate treatments for this patient? A. Albuterol B. Cromolyn C. Fluticasone D. Ipratropium bromide E. Salmeterol

The Correct Answer is: E The mainstay of treatment for exercise-induced asthma are beta-2 agonists. Due to the duration of physical activity this patient should be treated with a long-acting beta-2 agonist, salmeterol (E) instead of albuterol (A). If his symptoms are persistent then addition of a daily inhaled corticosteroid (C) is required, and daily ICS should be added if his use of salmeterol is frequent to avoid asthma-related death associated with the lone use of long-acting beta-agonists. Cromolyn (B) is an additional treatment options for exercise-induced bronchospasm, but is less commonly used compared to beta-2 agonists.

A 48-year-old female presents with a gradual progression of fatigue, pallor, and dyspnea on exertion over the last few months. Initial CBC results show a hemoglobin of 10.2 mg/dL, hematocrit of 31%, an MCV of 74 fL, and a reticulocyte count of 0.1%. After treatment for her anemia begins, which of the following diagnostic studies can be ordered first (earliest) to demonstrate a response to therapy? A. Bone marrow aspirate B. Ferritin C. Hemoglobin/hematocrit D. Iron binding capacity E. Reticulocyte count

The Correct Answer is: E The patient most likely has an underlying iron deficiency anemia (IDA) which requires confirmation and then evaluation for sources of chronic blood loss. IDA is treated with iron replacement (e.g., ferrous sulfate) and response to therapy can be assessed in 7-10 days via a reticulocyte count. The elevation of the reticulocyte count (after initial reticulocyte count was relatively low) at this time demonstrates the return of normal hemoglobin production (E). Bone marrow aspirate (A) would be an inappropriately invasive test to perform in this scenario. The patient's hemoglobin, hematocrit (C), MCV, and iron studies (B and D) will take a couple to a few months to return to normal.

A 65-year-old man presents to the clinic complaining of fatigue, weakness, paresthesias, and an unsteady gait. Physical exam reveals decreased vibratory sensation and lack of proprioceptive feedback, noted on Romberg testing. CBC reveals hemoglobin of 10.7 g/dl, hematocrit of 32.1% and MCV 117.6 fL. Which of the following is the most appropriate treatment? A. Erythropoietin B. Ferrous sulfate C. Folic acid D. Metformin E. Vitamin B12

The Correct Answer is: E The patient most likely has vitamin B 12 deficiency, as evidenced by the macrocytic anemia and neurological signs and symptoms. The patient should be treated with vitamin B 12 . Anemia secondary to chronic kidney disease (A) typically presents as a normocytic, normochromic anemia. Iron deficiency anemia (B) presents as a microcytic, hypochromic anemia. Folate deficiency (C) typically presents as a macrocytic anemia, but without the classic neurological symptoms consistent with vitamin B 12 deficiency.

An 11-month-old African-American male presents to the pediatric office with lethargy, jaundice and splenomegaly. A CBC reveals hemoglobin of 8.0 mg/dl and a hematocrit of 25%. Peripheral smear appearance is available below. Which of the following is the most likely diagnosis? A. Acute lymphocytic leukemia B. Alpha thalassemia minor C. Beta thalassemia minor D. G6PD deficiency E. Sickle cell disease

The Correct Answer is: E The patient's clinical presentation is most consistent with sickle cell disease and is confirmed by the presence of sickle cells and target cells on the peripheral smear. The patient lacks white blood cell abnormalities consistent with acute lymphocytic leukemia (A). Alpha thalassemia minor (B) is typically an asymptomatic, mild microcytic anemia with target cells present of peripheral smear. Beta thalassemia minor (C) typically presents with hematocrits between 28 and 40% and target cells on peripheral smear; however, the presence of sickle cells only occurs in the presence of combined sickle cell-beta thalassemia. G6PD (D) deficiency results in a hemolytic anemia with marked anemia and the presence of "bite" cells on peripheral smear.

A 36 year-old woman with no significant past medical history presents with gradual onset of dyspnea and fatigue leading to an episode of "fainting" this morning. Physical exam reveals increased jugular venous pressure, weak carotid pulses, clear lungs, and a loud S2. What is the most likely diagnosis? A. Aortic stenosis B. Cardiac tamponade C. Mitral Regurgitation D. Pulmonary fibrosis E. Pulmonary hypertension

The Correct Answer is: E The patients symptoms are due to decreased cardiac output resulting from decreased preload associated with pulmonary hypertension (E). Aortic stenosis (A) presents more commonly in geriatric patients who present with a murmur. Cardiac tamponade (B) can decrease cardiac output, but would lead to decreased heart sounds. Mitral regurgitation (C) would cause pulmonary edema and rales in conjunction with increased jugular venous pressure. Pulmonary fibrosis (E) is unlikely in this patient with normal lung sounds.

You are evaluating a patient who is having decreased sensation to his arm after he sustained a head on injury while playing football. He is stating that his right arm has decreased sensation that goes into the hand, but he is slightly vague on being descriptive. As you perform the physical examination, what part of the body would exhibit sensation for the C7 component of the brachial plexus? A. Lateral forearm B. Lateral upper arm C. Medial forearm D. Medial upper arm E. Third Finger

The Correct Answer is: E The sensory nerve emanating from the C7 level is responsible for sensation of the third finger. The lateral forearm is supplied by C6. The lateral upper arm is supplied by C5. The medial upper arm is supplied T1 and the medial forearm is supplied by C8.

The most commonly fractured long bone in both adults and children is which of the following? A. Femur B. Fibula C. Humerus D. Radius E. Tibia

The Correct Answer is: E The tibia is the most commonly fractured long bone in the body for both adults and children. The fractures are often the result of sporting activities in the young and may occur from a simple fall in the elderly - especially those with osteoporosis. Motor vehicle accidents are another common cause of tibial fractures. Open or complex tibial fractures are sometimes associated with compartment syndromes, infection and neurovascular compromise. The femur is the strongest of the long bones and generally only sustains fractures when exposed to extreme stress, such as that experienced in a motor vehicle collision or industrial accident. Fibular fractures commonly occur with a direct below to the lateral lower leg or with extreme ankle rotational forces or excessive inversion. A high percentage of ankle fractures involve the fibula, especially in older women. Humerus fractures are relatively rare in adults, but are the second most common fractures to occur at birth - behind only the clavicle in frequency. The radius is the most commonly fractured bone in the upper extremity, but still less common in frequency than the tibia. Falls on an outstretched arm are a common mechanism for the injury.

Active elbow extension is primarily controlled by which muscle(s)? A. Anconeus B. Biceps C. Brachialis D. Brachioradialis E. Triceps

The Correct Answer is: E The triceps are the primary muscles that produce active elbow extension. The anconeus muscle is known as a secondary elbow extensor. The biceps and brachialis muscles are primary muscles of elbow flexion and the brachioradialis is a secondary elbow flexor. (

A 64-year-old male presents with right knee pain and stiffness while walking and going up and down stairs for the past six months. He states the pain persists all day long and is relieved with rest. There is no history of trauma and his past medical history is unremarkable. An x-ray of the right knee is performed and the following is observed: What is being identified by the white arrow? A. Osteophyte B. Salter-Harris Type I fracture C. Torn anterior cruciate ligament D. Osteosarcoma E. Narrowed joint space

The Correct Answer is: E The white arrow is identifying a narrowed joint space (E) on the medial side of the knee joint seen in osteoarthritis. The black line depicts sclerosis of the bone in the medial compartment, which is evidence of cortical thickening. The white wedge is showing the formation of osteophytes (A) in the medial femur as a result of the osteoarthritis. Salter-Harris fractures (B) are identified in adolescents with open growth plates. Osteosarcoma (D) would show a boney lesion. Soft tissue injuries (C) would not usually be identifiable on x-ray and would need further imaging, such as a CT or MRI.

An 82-year-old male presents to the emergency department with a 2-day history of weakness, fatigue, and mild shortness of breath. His past medical history includes hypertension, arthritis, diabetes, and hypercholesterolemia. His medications include metoprolol, glyburide, simvastatin, and acetaminophen. On physical examination, the vital signs reveal T 97.7, P 40, R 15, BP 84/60. His neck exam reveals no jugular venous distention, lungs are clear to auscultation, and cardiac has a regularly irregular bradycardia rhythm. There is no evidence of edema to the extremities. The patient is placed on a telemetry monitor, which reveals the image shown. Based on the information provided, what is the most appropriate medical therapy for this patient? A. Digoxin B., Diltiazem C. Amiodarone D. Increase metoprolol E. Stop metoprolol

The Correct Answer is: E This patient has developed complete heart block as evidenced by the telemetry rhythm strip. While the patient will require a pacemaker as the definitive treatment, the pharmacologic intervention that must be done is to halt the use of beta blockers (E), which will contribute to the patient's slower rate. The other choices (A, B, C, and D) would all exacerbate the condition the patient is already in, and make the clinical situation worse and potentially fatal.

A 44-year-old female presents to the emergency department with a right sided headache. She states the headache is located on the right temple region, is non-radiating, and does not cause photophobia. She is otherwise healthy and has no reported medical problems and has only taken acetaminophen for the pain, which has minimal relief. On physical exam she is alert, awake, and oriented. Her vitals are T 98.8, P 78, R 18, and BP 128/76. Her head is normocephalic, atraumatic, and pupils are equal and reactive. She has tenderness to the right temporal area of the temporal artery, and there is no noted swelling, redness, or abnormalities noted. There is no tenderness to the cervical muscles. She exhibits a non-focal neurological exam. A biopsy of the temporal artery reveals inflammatory cells of the artery wall. Based on the history and biopsy results, what is the best initial treatment for this patient? A. Cyclosporine B. Azathioprine C. Methotrexate D. Ibuprofen E. Prednisone

The Correct Answer is: E This patient is exhibiting a case of temporal arteritis. She has the signs and symptoms that are classic in nature and do not fit into the realm of the other headaches, all of which usually have different characteristics on history and physical exam. In this instance, the use of oral steroids will be the best choice in management of the condition. Anti-inflammatories (C, D) and immunosuppressant agents (A, B) will not be as effective as oral prednisone (E), and the long-term outcomes are improved with steroids.

A 39-year-old male presents to the clinic with a concern for a sexually transmitted infection. He suspects his wife has been having an affair. He denies any penile pain or discharge. This is the third time he has been into the office with this concern in the past four months. All of the STD tests have been negative thus far. Upon further questioning, he is noted to be defensive, extremely sensitive, and quite secretive. He also has suspicions that his neighbor is throwing fallen leaves from a tree into his yard. This patient is exhibiting symptoms most consistent with which of the following personality disorders? A. Antisocial B. Borderline C. Narcissistic D. Obsessive compulsive E. Paranoid

The Correct Answer is: E This patient is exhibiting symptoms consistent with paranoid personality disorder. Antisocial personality disorder is characterized by selfishness, callousness, promiscuity and impulsive behavior, inability to learn from experience and legal problems. The clinical findings of borderline personality disorder include impulsiveness, unstable and intense interpersonal relationships, lack of self control, suicidal, aggressive behavior and a high drug abuse rate. Clinical findings of obsessive-compulsive disorder include being a perfectionist, egocentric, indecisive, with rigid thought patterns and a need for control. Narcissistic personality disorder presents with the clinical findings of grandiosity, a preoccupation with power, lacking interest in others, and excessive demands for attention.

A 17 year-old male notes acute onset of dyspnea, excessive non-productive coughing, "wheezing" and upper chest tightness when running. Symptoms usually resolve in a few minutes and he is able to resume running. He is prescribed albuterol with no improvement. Which of the following is the most likely diagnosis in the patient? A. Angioedema B. Bronchiectasis C. Cystic fibrosis D. GERD E. Vocal Cord Dysfunction

The Correct Answer is: E Vocal cord dysfunction (E) is commonly misdiagnosed as asthma or is a comorbidity in patients with asthma. The history of acute onset and rapid disappearance of symptoms is consistent with vocal cord dysfunction, and not typical of cystic fibrosis (C), or GERD (D). The presence of urticarial or swelling of the face or lips would support the life-threatening diagnosis of angioedema (A). Bronchiectasis (B) typically presents with a chronic productive cough and additional pulmonary symptoms based on the underlying cause and severity.

Which of the following would be the best type of exercise to do to strengthen bones and prevent osteoporosis? A. Ballistic stretching B. Recumbent bike C. Static stretching D. Swimming E. Weight training

The Correct Answer is: E Weight bearing and resistance exercises are the most effective at maintaining bone density. Bones respond favorably to regular impact loading exercises such as walking and strength training with either machines or free weights. Adding balance training exercises like those done in tai chi, yoga, or Pilates would be beneficial as well as part of a fracture prevention plan. Ballistic stretching is a type of flexibility exercise that uses bouncing movements to try and increase range of motion. Static stretching is a more gentle approach to stretching where the person stretches as far as they comfortably can and hold that position for several seconds. There are various schools of thought on the benefits and safety of each stretching method. While both likely have a role in maintaining flexibility and function, neither is likely to play a significant role in the prevention of osteoporosis. Exercising on a recumbent bike likely offers some strength and cardiovascular benefits, especially if utilized as part of a strenuous workout, but as a non weight bearing exercise (seeing as the person using the bike is sitting and leaning back with their weight fully supported), this is not the ideal exercise for osteoporosis prevention. Swimming is a wonderful exercise choice since it affords a challenging cardiovascular workout, uses all of the muscles of the body, improves range of motion and is well tolerated by people with various joint ailments, but because it is not a weight bearing exercise, it should be combined with weight training for optimal bone health.

A 16-year-old girl presents to the office complaining of a very sore throat, swollen lymph nodes, fever, and general malaise. Her examination reveals a temperature of 102.2°F, enlarged exudative tonsils, tender cervical lymphadenopathy, and borderline enlarged spleen. Rapid strep screen is negative. Which of the following laboratory findings best supports the most likely diagnosis? A. decreased white blood cell count B. increased monocytes on white cell differential C. thrombocytosis D. decreased levels of antibody to Epstein-Barr viral capsid antigen E. increased atypical lymphocytes on white blood cell differential

The Correct Answer is: E With a negative rapid strep screen, the most likely explanation for this presentation is acute infectious mononucleosis. The fever, fatigue, tonsillar hypertrophy, and splenomegaly are all classic symptoms and signs. Laboratory evaluation often includes an elevated total white blood cell count with increased atypical lymphocytes on differential. Platelets may be decreased. Initially, IgM antibodies for the Epstein-Barr virus, and viral capsid antigen (VCA) levels will be elevated. Later, the IgG levels increase and IgM normalizes.

A 60-year-old female presents to her primary care practitioner for her yearly check up. Her past medical history is significant only for a history of repeat kidney stones. She enjoys gardening outside, has no significant family history, and is not taking any medications. Routine mammograms have been normal. During the review of her systems, the patient describes feeling tired lately and has noticed muscle aches over the last few weeks. Initial lab values are shown below: Hemoglobin= 12.0 gm/dL Hematocrit = 36% BUN= 18 mg/dL Creatinine= 1.0 mg/dL Calcium= 12 mg/dL Phosphate= 2.0 mg/dL Intact PTH= 80 pg/mL TSH= 3.0 uU/L What is the most likely pathology associated with the findings in this patient? A. Solitary parathyroid adenoma B. Excessive Ingestion of calcium C. Parathyroid carcinoma D. Medullary thyroid cancer E. Chronic renal disease

The correct choice is A, solitary parathyroid adenoma. Primary hyperparathyroidism is caused by a solitary adenoma in 80% of patient cases. Choices B, D, and E do not cause primary hyperparathyroidism. Choice C can cause primary hyperparathyroidism, but with an incidence of less than 1%.

Patients with suspected familial hypercholesterolemia have serum cholesterol levels > 300 mg/dL and are at increased risk of atherosclerosis. Which of the following physical exam findings are nearly pathognomonic for familial hypercholesterolemia? A. Tendon xanthomas B. Lipomas C. Bouchard's nodes D. Carotid bruits E. Visceral obesity

The correct choice is A, tendon xanthomas. These are depositions of cholesterol rich substances that can present in any tendon as a mass-like lesion. They are most commonly found in the Achilles, patellar, and hand extensor tendons. Choice B, lipomas, are benign, soft, moveable subcutaneous tumors made from fat cells. Choice C, Bouchard's nodes, are painless nodules on the PIP joints, commonly seen in patients with osteoarthritis. Choice D, carotid bruits, may be heard with auscultation of the neck during the physical exam in patients with artherosclerosis of the carotid arteries. Choice E, visceral obesity, is a risk factor for diabetes and atherosclerosis, but is not pathognomonic of familial hypercholesterolemia.

A 29-year-old woman presents in July to your office with symptoms of palpitations, sore neck, and excessive sweating, despite using her air conditioner all the time. No surgical or trauma history is noted. She is currently not taking any medications. Vitals include the following: BP = 124/68, pulse = 110 beats per minute, respirations = 18 per minute, and temperature = 101 o F orally. Upon exam, her thyroid is mildly enlarged without nodules, and severely tender. No local erythema or heat is noted. Which of the following lab results would you expect in this patient? A. Serum total T4 level = 5.0 ug/dL B. Serum TSH level = 0.25 uIU/mL C. Sedimentation rate = 15 mm/hr D. Free thyroxine index = 8.0 E. Positive thyroid stimulating antibodies

The correct choice is B, Serum TSH level=0.25 uIU/mL. The reference range for TSH is 0.34 to 4.25 uIU/mL, and therefore the level in this patient is low. This patient is presenting with signs and symptoms of hyperthyroidism, most likely due to subacute thyroiditis. The leaking of thyroid hormone into the circulation causes anterior pituitary suppression and reduced TSH secretion. Choice A is seen in patients with hypothyroidism. Choice C is within the reference range for woman. Since subacute thyroiditis is an acute inflammatory disorder, patients with this disorder will commonly present with an elevated sedimentation rate. Choice D corresponds to a euthyroid situation. It is an estimate of the free thyroid hormone level in the plasma. This result is within the reference range. Choice E is not seen in subacute thyroiditis. They are commonly found in patients with Graves' disease.

A 50-year-old woman presents with a history of polyuria, polydipsia, muscle aches, bone pain, nausea, and constipation for the past few months. Her past medical history is significant for a pituitary adenoma, peptic ulcer disease, and kidney stones. You are considering a diagnosis of primary hyperparathyroidism. Which of the following lab values would you expect in this patient? A. Low serum calcium and elevated serum PTH B. Elevated serum calcium and PTH levels C. Elevated serum calcium and low serum PTH levels D. Low serum phosphate and PTH levels E. Elevated serum phosphate and calcium levels

The correct choice is B, elevated serum calcium and PTH levels. In primary hyperparathyroidism, excess PTH is secreted and stimulates a rise in serum calcium by increasing calcium release from bone, thereby reducing renal clearance of calcium and increasing calcium absorption through the intestine. In choice A, low serum calcium is not seen in hyperparathyroidism. In choice C and D, low serum PTH levels aren't seen in hyperparathyroidism. In choice E, elevated serum phosphate levels aren't seen in hyperparathyroidism. It typically presents with low serum phosphate levels.

A 35-year-old woman comes to see you with symptoms of anxiety, sweating, and tremors. She has no history of diabetes, liver or kidney failure, hormone deficiencies, or past surgeries. She states that she is not currently prescribed any medications and does not drink alcohol. She is concerned that she will not be able to continue to care for her husband, who has a long history of diabetes mellitus. She is asking you to give her some medication to stop her symptoms. Initial lab results are as follows: Plasma glucose = 54 mg/dL (70-110 mg/dL) TSH = 2.0 mIU/L (0.34-4.25 uIU/mL) Insulin = 35 uU/mL (2.0-20 uU/mL) C-peptide= 0.4 ng/mL (0.5-2.0 ng/mL) Her symptoms are relieved with the drinking of orange juice. What is the most likely cause of her hypoglycemia? A. Alimentary hypoglycemia B. Factitious hypoglycemia C. Beta cell insulinoma D. Congenital hyperinsulinism E. Reactive hypoglycemia

The correct choice is B, factitious hypoglycemia. This occurs when patients accidentally or on purpose self-administer insulin or an insulin secretagogue. This occurs most commonly among health care personnel, patients with diabetes or family members of those with diabetes, and people with a history of other factitious diseases. It can also happen secondary to a pharmacy error. Patients with this disorder will have increased measured insulin without the physiologic corresponding increase in C-peptide. Choice A, alimentary hypoglycemia, is a cause of hypoglycemia in patients with a history of gastrectomy. Choice C, beta cell insulinoma would present with elevated levels of both insulin and C-peptide. Choice D, congenital hyperinsulinism would have presented itself earlier than in a patient who is 35 years old. Choice E, reactive hypoglycemia, occurs after eating a meal and must be documented in this fashion.

Until now, an elderly patient with type 2 diabetes mellitus has been able to keep her plasma glucose and hemoglobin A1c under control with a diabetic diet and regular exercise. Her recent check-ups revealed that her hemoglobin A1c has been rising up. Her health care provider suggested that she would benefit from treatment with medication. Which of the following medications acts by reducing hepatic gluconeogenesis? A. Pioglitazone B. Metformin C. Acarbose D. Miglitol E. Saxagliptin

The correct choice is B, metformin. This medication is used as an adjunct to diet, to control hyperglycemia in patients with type 2 diabetes mellitus. It works well in patients who are obese and who may not be responding to sulfonylurea medications. It tends to improve both fasting and postprandial hyperglycemia. Choice A, pioglitazone, is a thiazolidinedione and acts to reduce plasma glucose by sensitizing peripheral tissues to insulin. Choice C, acarbose, and choice D, miglitol, are alpha-glucosidase inhibitors. They act to treat hyperglycemia by acting as competitive inhibitors of alpha-glucosidases in the intestinal brush border. This results in a delay in the absorption of carbohydrates and reduced postprandial glucose excursion. Choice E, saxagliptin, is a DPP-4 inhibitor that acts to prolong the action of endogenously released glucagon like peptide 1 (GLP-1) and glucose-dependent insulinotropic polypeptide (GIP), which increase the glucose-induced insulin release.

A 45-year-old patient presents to your office with a magazine ad for lipitor. She is wondering if she should be taking this medication. She heard on television that it may help to reduce her risk of death. Most of her friends are taking it, and she wants the best health care possible. She has no past medical history or family history of obesity, diabetes, hypertension, stroke, or heart attacks. She doesn't smoke and she exercises regularly. Her diet includes a wide variety of vegetables, fish, meats, and fruit. Her fasting lipid panel reveals the following results: Total cholesterol = 180 mg/dL Total triglycerides = 100 mg/dL HDL = 60mg/dL LDL = 140 mg/dL According to the 2004 revised NCEP ATP III guidelines, what plan should you recommend for this patient? A. Initiate strict dietary changes to lower total fat intake to between 10% and 20% daily B. Suggest preparing a food diary to look for patterns and choices C. Increase the amount of exercise to five days per week D. Start lipid lowering agent such as niacin E. Suggest starting a low carbohydrate diet

The correct choice is B, suggest preparing a food diary to look for patterns and choices. According to the NCEP ATP III guidelines, this patient does not have any risk factors for coronary heart disease. Her LDL level should be less than 160 mg/dL. All patients can benefit from observing their dietary habits and looking for opportunities to make changes to lower their fat intake. Dietary cholesterol should be less than 200 mg/day, and soluble fiber should be included in the diet. Patients can look at their eating patterns at home, look at the methods used for cooking and baking, and determine the frequency of eating out. Strict changes in her diet, choice A, are not necessary at this time. She already exercises regularly and her HDL is at an adequate level. Therefore, choice C, increasing the amount of exercise to five days per week, is not necessary. Choice D, start a lipid lowering agent such as niacin, is not recommended. Niacin is used to increase serum HDL and lower serum LDL. Her lipid levels are within the reference ranges. Choice E, suggest starting a low carbohydrate diet, may be associated with short-term weight loss, an increase in HDL, and a decrease in triglycerides. This is not something that this patient requires.

Which of the following is appropriate in the pharmacologic management of patients with hypoparathyroid tetany? A. Aggressive IV hydration B. Cinacalcet hydrochloride PO C. Calcium gluconate IV D. Pamidronate IV E. Calcitonin IV

The correct choice is C, calcium gluconate IV. In severe hypocalcemia, replacement calcium must be started promptly, as well as airway maintenance and magnesium and vitamin D replacement, as necessary. Choices A, aggressive hydration, B, cinacalcet hydrochloride, D, pamidromate, and E, calcitonin, are all possible treatment options for patients with hypercalcemia.

A patient presents with symptoms of palpitations, tremor, confusion, and sweating. She is also hungry. Her plasma glucose level is currently 54 mg/dL. Which of the following drugs would be particularly important to investigate during the patient history? A. Vancomycin B. Acetaminophen C. Ethanol D. Lisinopril E. Prednisone

The correct choice is C, ethanol. Ethanol can suppress gluconeogenesis and stimulate glucose utilization. Alcohol induced hypoglycemia occurs most commonly after a several day drinking binge with little food intake. Other medications that can be associated with hypoglycemia include insulin, insulin secretagogues, salicylates, sulfonamides, pentamidine, quinine, quinolone antibiotics, and beta 1 -adrenergic antagonists. The other choices are not associated with inducing hypoglycemia. Choice E, prednisone use, has been associated with hyperglycemia.

A 60-year-old female presents to her primary care practitioner for her yearly check up. Her past medical history is significant only for a history of repeat kidney stones. She enjoys gardening outside, has no significant family history, and is not taking any medications. Routine mammograms have been normal. During the review of her systems, the patient describes feeling tired lately and has noticed muscle aches over the last few weeks. Initial lab values are shown below: Hemoglobin = 12.0 gm/dL Hematocrit = 36% BUN = 10 mg/dL Creatinine = 0.7 ng/mL Calcium = 12 mg/dL Phosphate = 2.0 mg/dL PTH = 100 pg/mL TSH = 2.0 UIU/mL What is the most likely cause of her symptoms? A. Vitamin D deficiency B. Hypercalcemia of malignancy C. Primary hyperparathyroidism D. Factitious hypercalcemia E. Hyperthyroidism

The correct choice is C, primary hyperparathyroidism. This patient has the characteristic signs and symptoms of hypercalcemia, along with the lab findings of primary hyperparathyroidism with elevated calcium, low phosphate, and elevated PTH. Other common presenting features include bone pain, paresthesias, polyuria, and gastrointestinal symptoms. Females are affected three times as much as males, and it is typically found accidentally. Choice A, vitamin D deficiency, is unlikely since she enjoys gardening outside in the sun. Vitamin D deficiency can be seen is association with primary hyperparathyroidism, but is not the cause of this disorder. Choice B, hypercalcemia of malignancy, is unlikely with her history of normal mammograms. Serum PTH is typically normal in this disorder as well. Choice D, factitious hypercalcemia, is unlikely since she doesn't take any calcium supplements or other medications. Choice E, hyperthyroidism, can be a cause of hypercalcemia if the patient is suffering from thyrotoxicosis, but this patient's TSH is within the reference range.

A 50-year-old male is seen with a routine check-up. He is concerned about the possibility of developing diabetes mellitus. He has a negative family history of diabetes. He has no signs or symptoms of diabetes and he is not overweight. Without any risk factors for diabetes, what is the recommended screening protocol for this patient according to the American Diabetes Association (ADA)? A. screen all men over 25 years of age every five years B. screen all men over 35 years of age every two years C. screen everyone over 45 years of age every three years D. no screening is necessary without risk factors E. no screening is necessary without a family history of diabetes

The correct choice is C, screen everyone older than 45 years of age every three years. In addition, the ADA recommends screening for younger people if they are overweight and have at least one additional risk factor, such as positive family history, hypertension, and/or vascular disease. The other choices are not recommended by the ADA for screening the general population for diabetes mellitus.

Which of the following sets of disorders is commonly found in multiple endocrine neoplasia (MEN) 2A? A. Islet cell tumor; renal cell carcinoma; pheochromocytoma; B. Pheochromocytoma; medullary thyroid carcinoma; mucosal neuromas C. Medullary thyroid carcinoma; parathyroid hyperplasia; pheochromocytoma D. Parathyroid adenoma; islet cell hyperplasia; pituitary adenoma E. Visceral lipomas; Marfanoid features; retinal angiomas

The correct choice is C. The three primary features of MEN type 2A include medullary thyroid carcinoma, parathyroid hyperplasia or adenoma, and pheochromocytoma. In choice A, islet cell tumor and renal cell carcinoma are disorders found in various other MEN syndromes, but not in MEN type 2A. Choice B, pheochromocytoma, medullary thyroid carcinoma, and mucosal neuroma are disorders found in MEN 2B. Choice D, parathyroid adenoma, islet cell hyperplasia, and pituitary adenoma are found in MEN I. Choice E, visceral lipomas, Marfanoid features, and retinal angiomas are features found in various MEN syndromes, but not in MEN 2A.

Patients diagnosed with type 2 diabetes are encouraged to maintain their plasma glucose, to prevent or slow the initiation of chronic complications. What is the maximum level that is within ADA guidelines for a one to two hour postprandial plasma glucose in these patients? A. 70 mg/dL B. 100 mg/dL C. 120 mg/dL D. 180 mg/dL E. 220 mg/dL

The correct choice is D, 180 mg/dL. Patients are encouraged to adhere to lifestyle and medication treatment plans in order to help achieve this goal. Other ADA guidelines include maintaining the hemoglobin A1c < 7% or as close to normal (<6%) as possible, blood pressure < 130/80, and preprandial glucose between 90 and 130 mg/dL. (Powers et al., 2008, Chapter 338)

A patient presents with polydipsia and polyuria. Diabetes mellitus is ruled out with a normal plasma glucose and hemoglobin A1c. You are concerned that he may have hypothalamic diabetes insipidus. If you are correct, what treatment would you recommend to this patient? A. Lithium B. Indomethacin C. Metformin D. Desmopressin E. Fluid restriction

The correct choice is D, desmopression. Patients with hypothalamic diabetes insipidus can't secrete vasopression (or antidiuretic hormone), since the vasopressin producing neurons are dead. Vasopressin analog desmopressin is available in tablets, nasal solution, and parenteral solution for patients with this disorder. Choice A, lithium, can cause nephrotoxicity, and can be a cause of acquired nephrogenic diabetes insipidus. Choice B, indomethacin, is an anti-inflammatory medication used to treat conditions such as gout. Choice C, metformin, is an oral diabetic medication that acts by suppressing hepatic glucose production. Patients must drink sufficient fluids to maintain serum sodium levels since without ADH, they can't conserve water. Therefore, choice E, fluid restriction, is not correct.

A patient was recently diagnosed with type 1 diabetes mellitus. A treatment plan was initiated, with a combination regimen of insulin. Which of the following types of insulin works well with a rapidly acting insulin, such as insulin lispro, to provide 24-hour coverage for the patient? A-NPH insulin B-Regular insulin C- Insulin aspart D- Insulin glargine E- Humalog 75/25

The correct choice is D, insulin glargine. This is the only long acting insulin listed. The combination of a long acting insulin with a rapidly acting insulin provides physiologic insulin replacement to the patient. This regimen provides postprandial control after meals and basal coverage throughout the day and night. Choice A NPH insulin, can be used by itself in two or more injections throughout the day. Choice B, regular insulin, can be used instead of rapid acting insulin, and not in combination with it. Choice C, insulin aspart, is a type of rapidly acting insulin and would not be used in combination with another rapidly acting insulin. Choice E, Humalog 75/25, is a combination insulin preparation with 75% intermediate acting insulin and 25% insulin lispro.

An elderly women presents with a history of atrial fibrillation and falls. Her current review of symptoms reveals increased sweating and weight loss. A physical exam revealed multiple non-tender nodules on thyroid exam. Which of the following serum lab values is most consistent with this diagnosis? A. High TSH, high free T3, normal total T4 B. Normal TSH, low free T3, low total T4 C. Normal TSH, high free T3, high total T4 D. Low TSH, high free T3, high total T4 E. Low TSH, high free T3, low total T4

The correct choice is D, low TSH, high free T 3 , and high total T 4 . A diagnosis of toxic multinodular goiter must be made first. This disorder presents with symptoms of hyperthyroidism. In the elderly, patients present more frequently with symptoms of the complications of the disease including cardiovascular and cerebrovascular presentations. These may include atrial fibrillation, congestive heart failure, angina, apathy, and confusion. Other general symptoms of hypermetabolism may include weight loss, sweating, and tremor. Laboratory findings in various causes of hyperthyroidism include a reduced serum TSH, as the pituitary is responding to the increased serum levels of thyroid hormone in the circulation. These thyroid hormone levels feed back on the anterior pituitary, causing a suppression of TSH secretion. In choice A, high TSH, high free T 3 ,and normal total T 4 , high TSH is only seen in secondary causes of hyperthyroidism (e.g. anterior pituitary origin). This patient's history of multiple thyroid nodules indicates a thyroid cause for her symptoms. In choices B and C, the TSH is normal. This is seen in euthyroid patients. Choice E does include suppression of TSH, but the total T 4 is typically elevated in hyperthyroid states.


Kaugnay na mga set ng pag-aaral

Chapter 18 - Disorders of Aging and Cognition

View Set

APUSH AP Sample Questions Unit 4

View Set

AP Euro: The Industrial Revolution

View Set

LAW101 علاقة النظام بالعلوم الإجتماعية

View Set

Business Communications Final Exam Study Guide

View Set

Anatomy: Chapters 9 and 10 Homework

View Set

FAR Chapter 1: Segment Reporting

View Set